100% found this document useful (1 vote)
3K views190 pages

New SAT Math Problems Arranged by Topic and Difficulty Level Downloadable Full

Uploaded by

adel
Copyright
© © All Rights Reserved
We take content rights seriously. If you suspect this is your content, claim it here.
Available Formats
Download as PDF, TXT or read online on Scribd
100% found this document useful (1 vote)
3K views190 pages

New SAT Math Problems Arranged by Topic and Difficulty Level Downloadable Full

Uploaded by

adel
Copyright
© © All Rights Reserved
We take content rights seriously. If you suspect this is your content, claim it here.
Available Formats
Download as PDF, TXT or read online on Scribd

Legal Notice

This book is copyright 2015 with all rights reserved. It is illegal to copy,
distribute, or create derivative works from this book in whole or in part or
to contribute to the copying, distribution, or creating of derivative works
of this book.
New SAT Math Problems
arranged by Topic and
Difficulty Level

For the Revised SAT


March 2016 and Beyond
Dr. Steve Warner

© 2015, All Rights Reserved

iii
BOOKS FROM THE GET 800 COLLECTION FOR
COLLEGE BOUND STUDENTS
28 SAT Math Lessons to Improve Your Score in One Month
Beginner Course
Intermediate Course
Advanced Course
320 SAT Math Problems Arranged by Topic and Difficulty Level
320 SAT Math Subject Test Problems Arranged by Topic and
Difficulty Level
Level 1 Test
Level 2 Test
SAT Prep Book of Advanced Math Problems
The 32 Most Effective SAT Math Strategies
SAT Prep Official Study Guide Math Companion
SAT Vocabulary Book
ACT Prep Red Book – 320 ACT Math Problems with Solutions
320 AP Calculus AB Problems Arranged by Topic and Difficulty
Level
320 AP Calculus BC Problems Arranged by Topic and Difficulty
Level
555 Math IQ Questions for Middle School Students
555 Geometry Problems for High School Students

CONNECT WITH DR. STEVE WARNER

iv
Table of Contents
Introduction: The Proper Way to Prepare 7
1. Using this book effectively 7
2. The magical mixture for success 8
3. Practice problems of the appropriate level 9
4. Practice in small amounts over a long period of time 10
5. Redo the problems you get wrong over and over and
over until you get them right 10
6. Check your answers properly 11
7. Guess when appropriate 11
8. Pace yourself 11
9. Attempt the right number of questions 12
10. Use your calculator wisely 13
11. Grid your answers correctly 15

Problems by Level and Topic with Fully Explained Solutions 17


Level 1: Heart of Algebra 17
Level 1: Geometry and Trig 23
Level 1: Passport to Advanced Math 29
Level 1: Problem Solving and Data 37

Level 2: Heart of Algebra 42


Level 2: Geometry and Trig 47
Level 2: Passport to Advanced Math 54
Level 2: Problem Solving and Data 57

Level 3: Heart of Algebra 65


Level 3: Geometry and Trig 75
Level 3: Passport to Advanced Math 81
Level 3: Problem Solving and Data 87

Level 4: Heart of Algebra 92


Level 4: Geometry and Trig 98
Level 4: Passport to Advanced Math 109
Level 4: Problem Solving and Data 118

v
Level 5: Heart of Algebra 122
Level 5: Geometry and Trig 127
Level 5: Passport to Advanced Math 136
Level 5: Problem Solving and Data 143

Supplemental Problems – Questions 151


Level 1: Heart of Algebra 151
Level 1: Geometry and Trig 152
Level 1: Passport to Advanced Math 154
Level 1: Problem Solving and Data 156
Level 2: Heart of Algebra 158
Level 2: Geometry and Trig 159
Level 2: Passport to Advanced Math 161
Level 2: Problem Solving and Data 162
Level 3: Heart of Algebra 164
Level 3: Geometry and Trig 166
Level 3: Passport to Advanced Math 167
Level 3: Problem Solving and Data 168
Level 4: Heart of Algebra 169
Level 4: Geometry and Trig 170
Level 4: Passport to Advanced Math 172
Level 4: Problem Solving and Data 173
Level 5: Heart of Algebra 176
Level 5: Geometry and Trig 177
Level 5: Passport to Advanced Math 179
Level 5: Problem Solving and Data 180

Answers to Supplemental Problems 182

About the Author 187

Books by Dr. Steve Warner 188

vi
[Link]

I N T R O D U C T I O N
THE PROPER WAY TO PREPARE

his book is for the revised SAT beginning in March 2016. If


you are preparing for an SAT being administered before this date, then
this is not the right book for you. The PSAT being given in October 2015
will have the new format, so you can use this book to prepare for that test,
especially if you are going for a national merit scholarship.
There are many ways that a student can prepare for the SAT. But not
all preparation is created equal. I always teach my students the methods
that will give them the maximum result with the minimum amount of
effort.
The book you are now reading is self-contained. Each problem was
carefully created to ensure that you are making the most effective use of
your time while preparing for the SAT. By grouping the problems given
here by level and topic I have ensured that you can focus on the types of
problems that will be most effective to improving your score.

1. Using this book effectively


 Begin studying at least three months before the SAT
 Practice SAT math problems twenty minutes each day
 Choose a consistent study time and location
You will retain much more of what you study if you study in short bursts
rather than if you try to tackle everything at once. So try to choose about
a twenty minute block of time that you will dedicate to SAT math each
day. Make it a habit. The results are well worth this small time
commitment.
 Every time you get a question wrong, mark it off, no matter
what your mistake.
 Begin each study session by first redoing problems from previous
study sessions that you have marked off.
 If you get a problem wrong again, keep it marked off.

7
[Link]

Note that this book often emphasizes solving each problem in more than
one way. Please listen to this advice. The same question is not generally
repeated on any SAT so the important thing is learning as many
techniques as possible.
Being able to solve any specific problem is of minimal importance. The
more ways you have to solve a single problem the more prepared you will
be to tackle a problem you have never seen before, and the quicker you
will be able to solve that problem. Also, if you have multiple methods for
solving a single problem, then on the actual SAT when you “check over”
your work you will be able to redo each problem in a different way. This
will eliminate all “careless” errors on the actual exam. Note that in this
book the quickest solution to any problem will always be marked with an
asterisk (*).

2. The magical mixture for success


A combination of three components will maximize your SAT math score
with the least amount of effort.
 Learning test taking strategies that work specifically for
standardized tests.
 Practicing SAT problems for a small amount of time each day for
about three months before the SAT.
 Taking about four practice tests before test day to make sure you
are applying the strategies effectively under timed conditions.
I will discuss each of these three components in a bit more detail.
Strategy: The more SAT specific strategies that you know the better off
you will be. Throughout this book you will see many strategies being
used. Some examples of basic strategies are “plugging in answer
choices,” “taking guesses,” and “picking numbers.” Some more advanced
strategies include “trying a simple operation,” and “moving the sides of a
figure around.” Pay careful attention to as many strategies as possible and
try to internalize them. Even if you do not need to use a strategy for that
specific problem, you will certainly find it useful for other problems in
the future.
Practice: The problems given in this book, together with the problems in
the practice tests from the College Board’s Official Study Guide (2016
Edition), are more than enough to vastly improve your current SAT math
score. All you need to do is work on these problems for about ten to
twenty minutes each day over a period of three to four months and the
final result will far exceed your expectations.

8
[Link]

Let me further break this component into two subcomponents – topic and
level.
Topic: You want to practice each of the four general math topics
given on the SAT and improve in each independently. The five topics are
Heart of Algebra, Geometry and Trig, Passport to Advanced Math,
and Problem Solving and Data Analysis. The problem sets in this book
are broken into these four topics.
Level: You will make the best use of your time by primarily
practicing problems that are at and slightly above your current ability
level. For example, if you are struggling with Level 2 Geometry and Trig
problems, then it makes no sense at all to practice Level 5 Geometry and
Trig problems. Keep working on Level 2 until you are comfortable, and
then slowly move up to Level 3. Maybe you should never attempt those
Level 5 problems. You can get an exceptional score without them (higher
than a 700).
Tests: You want to take about four practice tests before test day to make
sure that you are implementing strategies correctly and using your time
wisely under pressure. For this task you should “The Official SAT Study
Guide (2016 Edition).” Take one test every few weeks to make sure that
you are implementing all the strategies you have learned correctly under
timed conditions.

3. Practice problems of the appropriate level


Roughly speaking about one third of the math problems on the SAT are
easy, one third are medium, and one third are hard. If you answer two
thirds of the math questions on the SAT correctly, then your score will be
approximately a 600 (out of 800). That’s right—you can get about a 600
on the math portion of the SAT without answering a single hard question.
Keep track of your current ability level so that you know the types of
problems you should focus on. If you are currently scoring around a 400
on your practice tests, then you should be focusing primarily on Level 1,
2, and 3 problems. You can easily raise your score 100 points without
having to practice a single hard problem.
If you are currently scoring about a 500, then your primary focus should
be Level 2 and 3, but you should also do some Level 1 and 4 problems.
If you are scoring around a 600, you should be focusing on Level 2, 3,
and 4 problems, but you should do some Level 1 and 5 problems as well.
Those of you at the 700 level really need to focus on those Level 4 and 5
problems.

9
[Link]

If you really want to refine your studying, then you should keep track of
your ability level in each of the four major categories of problems:
 Heart of Algebra
 Geometry and Trig
 Passport to Advanced Math
 Problem Solving and Data Analysis
For example, many students have trouble with very easy Geometry and
Trig problems, even though they can do more difficult algebra problems.
This type of student may want to focus on Level 1, 2, and 3 Geometry
and Trig questions, but Level 3 and 4 Heart of Algebra questions.

4. Practice in small amounts over a long period of time


Ideally you want to practice doing SAT math problems ten to twenty
minutes each day beginning at least 3 months before the exam. You will
retain much more of what you study if you study in short bursts than if
you try to tackle everything at once.
The only exception is on a day you do a practice test. You should do at
least four practice tests before you take the SAT. Ideally you should do
your practice tests on a Saturday or Sunday morning. At first you can do
just the math section. The last one or two times you take a practice test
you should do the whole test in one sitting. As tedious as this is, it will
prepare you for the amount of endurance that it will take to get through
this exam.
So try to choose about a twenty minute block of time that you will
dedicate to SAT math every night. Make it a habit. The results are well
worth this small time commitment.

5. Redo the problems you get wrong over and over and
over until you get them right
If you get a problem wrong, and never attempt the problem again, then it
is extremely unlikely that you will get a similar problem correct if it
appears on the SAT.
Most students will read an explanation of the solution, or have someone
explain it to them, and then never look at the problem again. This is not
how you optimize your SAT score. To be sure that you will get a similar
problem correct on the SAT, you must get the problem correct before the
SAT—and without actually remembering the problem.

10
[Link]

This means that after getting a problem incorrect, you should go over and
understand why you got it wrong, wait at least a few days, then attempt
the same problem again. If you get it right you can cross it off your list of
problems to review. If you get it wrong, keep revisiting it every few days
until you get it right. Your score does not improve by getting problems
correct. Your score improves when you learn from your mistakes.

6. Check your answers properly


When you go back to check your earlier answers for careless errors do
not simply look over your work to try to catch a mistake. This is usually a
waste of time. Always redo the problem without looking at any of your
previous work. Ideally, you want to use a different method than you used
the first time.
For example, if you solved the problem by picking numbers the first time,
try to solve it algebraically the second time, or at the very least pick
different numbers. If you do not know, or are not comfortable with a
different method, then use the same method, but do the problem from the
beginning and do not look at your original solution. If your two answers
do not match up, then you know that this a problem you need to spend a
little more time on to figure out where your error is.
This may seem time consuming, but that’s okay. It is better to spend more
time checking over a few problems than to rush through a lot of problems
and repeat the same mistakes.

7. Take a guess whenever you cannot solve a problem


There is no guessing penalty on the SAT. Whenever you do not know
how to solve a problem take a guess. Ideally you should eliminate as
many answer choices as possible before taking your guess, but if you
have no idea whatsoever do not waste time overthinking. Simply put
down an answer and move on. You should certainly mark it off and come
back to it later if you have time.

8. Pace yourself
Do not waste your time on a question that is too hard or will take too
long. After you’ve been working on a question for about 30 to 45 seconds
you need to make a decision. If you understand the question and think
that you can get the answer in another 30 seconds or so, continue to work
on the problem. If you still do not know how to do the problem or you are
using a technique that is going to take a long time, mark it off and come
back to it later if you have time.

11
[Link]

If you do not know the correct answer, eliminate as many answer choices
as you can and take a guess. But you still want to leave open the
possibility of coming back to it later. Remember that every problem is
worth the same amount. Do not sacrifice problems that you may be able
to do by getting hung up on a problem that is too hard for you.

9. Attempt the right number of questions


Many students make the mistake of thinking that they have to attempt
every single SAT math question when they are taking the test. There is no
such rule. In fact, most students will increase their SAT score by
reducing the number of questions they attempt.
There are two math sections on the SAT – one where a calculator is
allowed and one where a calculator is not allowed. The calculator section
has 30 multiple choice (mc) questions and 8 free response (grid in)
questions. The non-calculator section has 15 multiple choice (mc)
questions and5 free response (grid in) questions.
You should first make sure that you know what you got on your last SAT
practice test, actual SAT, or actual PSAT (whichever you took last).
What follows is a general goal you should go for when taking the exam.

MC Grid In MC Grid In
Score (Calculator (Calculator (Calculator (Calculator
Allowed) Allowed) Not Allowed) Not Allowed)

< 330 10/30 3/8 4/15 1/5


330 – 370 15/30 4/8 6/15 2/5
380 – 430 18/30 5/8 8/15 2/5
440 – 490 21/30 6/8 9/15 3/5
500 – 550 24/30 6/8 11/15 4/5
560 – 620 27/30 7/8 13/15 4/5
630 – 800 30/30 8/8 15/15 5/5
For example, a student with a current score of 450 should attempt the first
21 multiple choice questions and the first 6 grid ins from the section
where a calculator is allowed, and 9 multiple choice questions and 3 grid
in questions from the section where a calculator is not allowed.

12
[Link]

This is just a general guideline. Of course it can be fine-tuned. As a


simple example, if you are particularly strong at Algebra problems, but
very weak at Geometry and Trig problems, then you may want to try
every Algebra problem no matter where it appears, and you may want to
reduce the number of Geometry and Trig problems you attempt.
Remember that there is no guessing penalty on the SAT, so you should
not leave any questions blank. This does not mean you should attempt
every question. It means that if you are running out of time make sure
you fill in answers for all the questions you did not have time to attempt.

10. Use your calculator wisely.


 Use a TI-84 or comparable calculator if possible when practicing
and during the SAT.
 Make sure that your calculator has fresh batteries on test day.
 You may have to switch between DEGREE and RADIAN modes
during the test. If you are using a TI-84 (or equivalent) calculator
press the MODE button and scroll down to the third line when
necessary to switch between modes.
Below are the most important things you should practice on your
graphing calculator.
 Practice entering complicated computations in a single step.
 Know when to insert parentheses:
 Around numerators of fractions
 Around denominators of fractions
 Around exponents
 Whenever you actually see parentheses in the expression

Examples:
We will substitute a 5 in for x in each of the following examples.
Expression Calculator computation
7x  3
(7*5 + 3)/(2*5 – 11)
2 x  11
(3x  8) 2 x9 (3*5 – 8)^(2*5 – 9)
 Clear the screen before using it in a new problem. The big screen
allows you to check over your computations easily.
 Press the ANS button (2ND (-) ) to use your last answer in the
next computation.

13
[Link]

 Press 2ND ENTER to bring up your last computation for editing.


This is especially useful when you are plugging in answer
choices, or guessing and checking.
 You can press 2ND ENTER over and over again to cycle
backwards through all the computations you have ever done.
 Know where the √ , 𝜋, ^, and 𝑒 𝑥 buttons are so you can reach
them quickly.
 Change a decimal to a fraction by pressing MATH ENTER
ENTER.
 Press the MATH button - in the first menu that appears you can
take cube roots and nth roots for any n. Scroll right to NUM and
you have lcm( and gcd(. Scroll right to PRB and you have nPr,
nCr, and ! to compute permutations, combinations and factorials
very quickly.
 Know how to use the SIN, COS and TAN buttons as well as
SIN-1, COS-1 and TAN-1.
You may find the following graphing tools useful.
 Press the Y= button to enter a function, and then hit ZOOM 6 to
graph it in a standard window.
 Practice using the WINDOW button to adjust the viewing
window of your graph.
 Practice using the TRACE button to move along the graph and
look at some of the points plotted.
 Pressing 2ND TRACE (which is really CALC) will bring up a
menu of useful items. For example selecting ZERO will tell you
where the graph hits the x-axis, or equivalently where the
function is zero. Selecting MINIMUM or MAXIMUM can find
the vertex of a parabola. Selecting INTERSECT will find the
point of intersection of 2 graphs.

14
[Link]

11. Grid your answers correctly

The computer only grades what you have marked in


the bubbles. The space above the bubbles is just for
your convenience, and to help you do your bubbling
correctly.
Never mark more than one circle in a column or the
problem will automatically be marked wrong. You do
not need to use all four columns. If you don’t use a
column just leave it blank.
The symbols that you can grid in are the digits 0
through 9, a decimal point, and a division symbol for
fractions. Note that there is no negative symbol. So
answers to grid-ins cannot be negative. Also, there are
only four slots, so you can’t get an answer such as 52,326.
Sometimes there is more than one correct answer to a grid-in question.
Simply choose one of them to grid-in. Never try to fit more than one
answer into the grid.
If your answer is a whole number such as 2451 or a decimal that only
requires four or less slots such as 2.36, then simply enter the number
starting at any column. The two examples just written must be started in
the first column, but the number 16 can be entered starting in column 1, 2
or 3.
Note that there is no zero in column 1, so if your answer is 0 it must be
gridded into column 2, 3 or 4.
Fractions can be gridded in any form as long as there are enough slots.
The fraction 2/100 must be reduced to 1/50 simply because the first
representation won’t fit in the grid.
Fractions can also be converted to decimals before being gridded in. If a
decimal cannot fit in the grid, then you can simply truncate it to fit. But
you must use every slot in this case. For example, the decimal
.167777777… can be gridded as .167, but .16 or .17 would both be
marked wrong.
Instead of truncating decimals you can also round them. For example, the
decimal above could be gridded as .168. Truncating is preferred because
there is no thinking involved and you are less likely to make a careless
error.

15
[Link]

Here are three ways to grid in the number 8/9.

1
Never grid-in mixed numerals. If your answer is 24, and you grid in the
1
mixed numeral 24, then this will be read as 21/4 and will be marked
wrong. You must either grid in the decimal 2.25 or the improper fraction
9/4.
𝟏
Here are two ways to grid in the mixed numeral 1𝟐 correctly.

16
[Link]

PROBLEMS BY LEVEL AND TOPIC WITH


FULLY EXPLAINED SOLUTIONS
Note: An asterisk (*) before a question indicates that a calculator is
required. An asterisk (*) before a solution indicates that the quickest
solution is being given.

LEVEL 1: HEART OF ALGEBRA


1. Which of the following expressions is equivalent to
5𝑎 + 10𝑏 + 15𝑐 ?
(A) 5(𝑎 + 2𝑏 + 3𝑐)
(B) 5(𝑎 + 2𝑏 + 15𝑐)
(C) 5(𝑎 + 10𝑏 + 15𝑐)
(D) 5(𝑎 + 2𝑏) + 3𝑐
Solution by picking numbers: Let’s choose values for a, b, and c, say
a = 2, b = 3, c = 4. Then
5a + 10b + 15c = 5(2) + 10(3) + 15(4) = 10 + 30 + 60 = 100.
Put a nice big dark circle around 100 so you can find it easier later. We
now substitute a = 2, b = 3, c = 4 into each answer choice:
(A) 5(2 + 2 ∙ 3 + 3 ∙ 4) = 100
(B) 5(2 + 2 ∙ 3 + 15 ∙ 4) = 340
(C) 5(2 + 10 ∙ 3 + 15 ∙ 4) = 460
(D) 5(2 + 2 ∙ 3) + 3 ∙ 4 = 52
Since (B), (C), and (D) each came out incorrect, the answer is choice (A).
Important note: (A) is not the correct answer simply because it is equal
to 100. It is correct because all three of the other choices are not 100.
You absolutely must check all four choices!
Remark: All of the above computations can be done in a single step with
your calculator (if a calculator is allowed for this problem).
Notes about picking numbers: (1) Observe that we picked a different
number for each variable. We are less likely to get more than one answer
choice to come out to the correct answer this way.

17
[Link]

(2) We picked numbers that were simple, but not too simple. The number
2 is usually a good choice to start, if it is allowed. We then also picked 3
and 4 so that the numbers would be distinct (see note (1)).
(3) When using the strategy of picking numbers it is very important that
we check every answer choice. It is possible for more than one choice to
come out to the correct answer. We would then need to pick new
numbers to try to eliminate all but one choice.
* Algebraic solution: We simply factor out a 5 to get
5a + 10b + 15c = 5(a + 2b + 3c)
This is choice (A).
Remarks: (1) If you have trouble seeing why the right hand side is the
same as what we started with on the left, try working backwards and
multiplying instead of factoring. In other words we have
5(a + 2b + 3c) = 5a + 10b + 15c
Note how the distributive property is being used here. Each term in
parentheses is multiplied by the 5.
In general, the distributive property says that if 𝑥, 𝑦, and 𝑧 are real
numbers, then
𝑥(𝑦 + 𝑧) = 𝑥𝑦 + 𝑥𝑧.
This property easily extends to expressions with more than two terms.
For example,
𝑥(𝑦 + 𝑧 + 𝑤) = 𝑥𝑦 + 𝑥𝑧 + 𝑥𝑤.
(2) We can also solve this problem by starting with the answer choices
and multiplying (as we did in Remark (1)) until we get 5a + 10b + 15c.
2. Joseph joins a gym that charges $79.99 per month plus tax for a
premium membership. A tax of 6% is applied to the monthly
fee. Joseph is also charged a one-time initiation fee of $95 as
soon as he joins. There is no contract so that Joseph can cancel
at any time without having to pay a penalty. Which of the
following represents Joseph’s total charge, in dollars, if he keeps
his membership for 𝑡 months?
(A) 1.06(79.99 + 95)𝑡
(B) 1.06(79.99𝑡 + 95)
(C) 1.06(79.99𝑡) + 95
(D) (79.99 + .06𝑡) + 95

18
[Link]

Solution by picking a number: (We will be using a calculator for this


solution)
Let’s choose a value for 𝑡, say 𝑡 = 2, so that Joseph keeps his gym
membership for 2 months.
Now 6% of 79.99 is 4.80 (to the nearest cent). So each month of
membership, including tax, is 79.99 + 4.80 = 84.79 dollars. It follows
that 2 months of membership, with tax, is 2 ⋅ 84.79 = 169.58 dollars.
When we add the initiation fee we get 169.58 + 95 = 𝟐𝟔𝟒. 𝟓𝟖 dollars.
Put a nice big, dark circle around the number 264.58 so you can find it
easily later. We now substitute 𝑡 = 2 into each answer choice and use our
calculator:
(A) 1.06(79.99 + 95)*2 ≈ 370.98
(B) 1.06(79.99*2 + 95) ≈ 270.28
(C) 1.06(79.99*2) + 95 ≈ 264.58
(D) (79.99 + .06*2) + 95 = 175.11
Since choices (A), (B), and (D) came out incorrect, we can eliminate
them. Therefore the answer is choice (C).
Important note: (C) is not the correct answer simply because it came
out to 264.58. It is correct because all three of the other choices did not
come out correct.
* Algebraic solution: Since the monthly membership fee is 79.99
dollars, and the tax is 6%, the total monthly fee, with tax, is 1.06(79.99)
dollars per month. It follows that the total monthly fee for 𝑡 months is
1.06(79.99𝑡). Finally, we add in the one-time initiation fee to get
1.06(79.99𝑡) + 95, choice (C).
6
Notes: (1) 6% can be written either as the decimal .06 or the fraction .
100

To change a percent to a decimal, simply divide by 100, or equivalently,


move the decimal point two places to the left, adding in zeros if
necessary. Note that an integer has a “hidden” decimal point right after
the number. In other words, 6 can be written as 6., so when we move the
decimal point two places to the left we get .06 (we had to add in a zero as
a placeholder).
To change a percent to a fraction, simply place the number in front of the
percent symbol (%) over 100.

19
[Link]

(2) Since the tax is 6%, it follows that the tax for $79.99 is .06(79.99) or
6
100
(79.99) dollars.

It follows that the total monthly fee, including tax, is


79.99 + .06(79.99) dollars.
We can use the distributive property to simplify this expression as
follows:
79.99 + .06(79.99) = 1(79.99) + .06(79.99) = 1.06(79.99)
(3) See problem 1 for more information on the distributive property.
(4) In note (2) we saw that one way to get the total monthly fee, including
tax, is to add the amount of tax to the untaxed amount. A quicker way is
to simply multiply the monthly fee by 1.06. A justification for why this
works is given in the last line of note (2).
(5) If you need to pay a certain dollar amount more than once, simply
multiply by the number of times you need to pay.
For example, if you need to pay 100 dollars five times, then the final
result is that you pay 100 ⋅ 5 = 500 dollars. More generally, if you need
to pay 100 dollars 𝑡 times, then the final result is that you pay 100𝑡
dollars.
In this problem we want to pay the monthly fee 𝑡 times. Since the
monthly fee is 1.06(79.99), the final result is 1.06(79.99)𝑡, or
equivalently 1.06(79.99𝑡)
(6) Don’t forget to add on the one-time initiation fee to 1.06(79.99𝑡) to
get 1.06(79.99𝑡) + 95 dollars.
3. A high school has a $1000 budget to buy calculators. Each
scientific calculator will cost the school $12.97 and each
graphing calculator will cost the school $73.89. Which of the
following inequalities represents the possible number of
scientific calculators 𝑆 and graphing calculators 𝐺 that the
school can purchase while staying within their specified budget?
(A) 12.97𝑆 + 73.89𝐺 > 1000
(B) 12.97𝑆 + 73.89𝐺 ≤ 1000
12.97 73.89
(C) 𝑆
+ 𝐺 > 1000
12.97 73.89
(D) 𝑆
+ 𝐺 ≤ 1000

20
[Link]

* Algebraic solution: The total cost, in dollars, for 𝑆 scientific


calculators is 12.97𝑆, and the total cost, in dollars, for 𝐺 graphing
calculators is 73.89𝐺.
It follows that the total cost, in dollars, for 𝑆 scientific calculators and 𝐺
graphing calculators is 12.97𝑆 + 73.89𝐺.
To stay within the school’s budget, we need this total cost to be less than
or equal to 1000 dollars.
So the answer is 12.97𝑆 + 73.89𝐺 ≤ 1000, choice (B).
Notes: (1) When using the symbols “<” and “>”, the symbol always
points to the smaller number (and similarly for “≤” and “≥”).
(2) To stay within the specified budget means that the total must not
exceed $1000. Some equivalent ways to say this are as follows:
 the total must not be greater than $1000.
 the total must be less than or equal to $1000.
 the total 𝑇 must satisfy 𝑇 ≤ 1000.
(3) If the school were to spend exactly $1000, they would still be within
their budget. this is why the solution has “≤” instead of “<.”
27 13
4. If − < 2 − 5𝑥 < − , then give one possible value of
10 5
20𝑥 − 8.
* Solution by trying a simple operation: Observe that
20𝑥 − 8 = 4(5𝑥 − 2) = −4(2 − 5𝑥).
So we have
13 27
(−4) (− ) < −4(2 − 5𝑥) < (−4)(− )
5 10
or equivalently
52 54
5
< 20𝑥 − 8 < 5
So we can grid in 𝟓𝟑/𝟓.
Notes: (1) The simple operation we used here was multiplication by −4.
We simply multiplied each of the three parts of the given inequality by
−4, noting that the inequalities reverse because we are multiplying by a
negative number.

21
[Link]

27 13
(2) Take careful note to how − 10 and − 5
changed positions when we
multiplied by the negative number −4.
(3) If we are allowed to use a calculator for this problem we could
27 13
multiply each of − 10 and − 5 by −4 in our calculator to get
27 13
(−4) (− ) = 10.8 and (−4) (− ) = 10.4
10 5

So we can grid in 𝟏𝟎. 𝟓, 𝟏𝟎. 𝟔, or 𝟏𝟎. 𝟕.


(4) We actually do not need to worry too much about the inequalities
27 13
reversing in this problem. We can simply multiply each of − 10 and − 5
by −4, and then choose a number between the two numbers that we get.

5. The expression 3(5𝑥 + 8) − 4(3𝑥 − 2) is simplified to the form


𝑎𝑥 + 𝑏. What is the value of ab ?
* Algebraic solution:
3(5x + 8) – 4(3x – 2) = 15x + 24 – 12x + 8 = 3x + 32.
So 𝑎 = 3, 𝑏 = 32, and therefore 𝑎𝑏 = 3 ⋅ 32 = 𝟗𝟔.
Note: Make sure you are using the distributive property correctly here.
For example 3(5x + 8) = 15x + 24. A common mistake would be to write
3(5x + 8) = 15x + 8.
Also, −4(3𝑥 − 2) = −12𝑥 + 8. A common mistake would be to write
−4(3𝑥 − 2) = −12𝑥 − 2.
See problem 1 for more information on the distributive property.
6. If 𝑥 + 7𝑦 = 15 and 𝑥 + 3𝑦 = 7, what is the value of 𝑥 + 5𝑦?
* Solution by trying a simple operation: We add the two equations

𝑥 + 7𝑦 = 15
𝑥 + 3𝑦 = 7
2𝑥 + 10𝑦 = 22
22
Now observe that 2𝑥 + 10𝑦 = 2(𝑥 + 5𝑦). So 𝑥 + 5𝑦 = 2
= 𝟏𝟏.

Notes: (1) We can also finish the problem by dividing each term of
2𝑥 + 10𝑦 = 22 by 2.

22
[Link]

2𝑥 10𝑦 22 2𝑥 10𝑦 22
We have 2 = 𝑥, 2 = 5𝑦, and 2
= 11. So we get 2
+ 2
= 2
, or
equivalently 𝑥 + 5𝑦 = 11.
(2) Although I do not recommend this for this problem, we could solve
the system of equations for 𝑥 and 𝑦, and then substitute those values in
for 𝑥 and 𝑦 in the expression 𝑥 + 5𝑦.
See problem 73 for several different ways to do this.

LEVEL 1: GEOMETRY AND TRIG


7. Given right triangle ∆𝑃𝑄𝑅 below, what is the length of ̅̅̅̅
𝑃𝑄 ?

(A) √2
(B) √5
(C) 5
(D) 7
* Solution using Pythagorean triples: We use the Pythagorean triple
5, 12, 13 to see that PQ = 5, choice (C).
Note: The most common Pythagorean triples are 3, 4, 5 and 5, 12, 13.
Two others that may come up are 8, 15, 17 and 7, 24, 25.
Solution by the Pythagorean Theorem: By the Pythagorean Theorem,
we have 132 = (PQ)2 + 122. So 169 = (PQ)2 + 144. Subtracting 144 from
each side of this equation yields 25 = (PQ)2, or PQ = 5, choice (C).
Remarks: (1) The Pythagorean Theorem says that if a right triangle has
legs of length a and b, and a hypotenuse of length c, then c2 = a2 + b2.
(2) Be careful in this problem: the length of the hypotenuse is 13. So we
replace c by 13 in the Pythagorean Theorem

23
[Link]

(3) The equation x2 = 25 would normally have two solutions: x = 5 and


x = –5. But the length of a side of a triangle cannot be negative, so we
reject –5.
8. What is the radius of a circle whose circumference is 𝜋?
1
(A) 2
(B) 1
𝜋
(C) 2
(D) 𝜋
Solution by plugging in answer choices: The circumference of a circle
𝜋
is C = 2𝜋r. Let’s start with choice (C) as our first guess. If r = 2 , then
𝜋
C = 2𝜋( ) = π2. Since this is too big we can eliminate choices (C) and
2
(D).
Let’s try choice (B) next. If r = 1, then C = 2𝜋(1) = 2π, still too big.
1
The answer must therefore be choice (A). Let’s verify this. If r = 2, then
1
C = 2𝜋(2) = 𝜋. So the answer is indeed choice (A).

Note: When plugging in answer choices, it’s always a good idea to start
with choice (B) or (C) unless there is a specific reason not to. In this
problem, eliminating choice (C) allowed us to eliminate choice (D) as
well, possibly saving us from having to do one extra computation.
* Algebraic solution: We use the circumference formula C = 2𝜋r, and
substitute 𝜋 in for C.
C = 2𝜋r
𝜋 = 2𝜋r
𝜋
2𝜋
=r
1
2
=r
This is choice (A).

24
[Link]

5
9. In ∆𝑃𝑄𝑅 above, tan 𝑅 = 12. What is the length of side PR ?

(A) 11
(B) 13
(C) 15
(D) 16
OPP 5 OPP
* Since tan 𝑅 = ADJ
, we have 12 = ADJ
. Since the adjacent side is 12, the
opposite side must be 5. So we have the following picture.

We now find PR by using the Pythagorean Theorem, or better yet,


recognizing the Pythagorean triple 5, 12, 13.
So 𝑃𝑅 = 13, choice (B).
Remarks: (1) If you don’t remember the Pythagorean triple 5, 12, 13,
you can use the Pythagorean Theorem.
In this problem we have 𝑐 2 = 52 + 122 = 169. So 𝑐 = 13.
(2) See problem 7 for more information about Pythagorean triples and the
Pythagorean Theorem.
(3) The equation 𝑐 2 = 169 would normally have two solutions: 𝑐 = 13
and 𝑐 = −13. But the length of a side of a triangle cannot be negative, so
we reject –13.

25
[Link]

Here is a quick lesson in right triangle trigonometry for those of you


that have forgotten.
Let’s begin by focusing on angle A in the following picture:

Note that the hypotenuse is ALWAYS the side opposite the right angle.
The other two sides of the right triangle, called the legs, depend on which
angle is chosen. In this picture we chose to focus on angle A. Therefore
the opposite side is BC, and the adjacent side is AC.
Now you should simply memorize how to compute the six trig functions:
OPP HYP
sin A = HYP csc A = OPP
ADJ HYP
cos A = sec A =
HYP ADJ
OPP ADJ
tan A = ADJ
cot A = OPP

Here are a couple of tips to help you remember these:


(1) Many students find it helpful to use the word SOHCAHTOA. You
can think of the letters here as representing sin, opp, hyp, cos, adj, hyp,
tan, opp, adj.
(2) The three trig functions on the right are the reciprocals of the three
trig functions on the left. In other words, you get them by interchanging
the numerator and denominator. It’s pretty easy to remember that the
reciprocal of tangent is cotangent. For the other two, just remember that
the “s” goes with the “c” and the “c” goes with the “s.” In other words,
the reciprocal of sine is cosecant, and the reciprocal of cosine is secant.
To make sure you understand this, compute all six trig functions for each
of the angles (except the right angle) in the triangle given in this problem.
Please try this yourself before looking at the answers below.

26
[Link]

12 13 5 13
sin P = 13 csc P = 12 sin R = 13 csc R = 5

5 13 12 13
cos P = 13 sec P = 5
cos R = 13 sec R = 12

12 5 5 12
tan P = 5
cot P = 12 tan R = 12 cot R = 5

10. Let 𝑥 = cos 𝜃 and 𝑦 = sin 𝜃 for any real value 𝜃. Then x2 + y2 =
(A) −1
(B) 0
(C) 1
(D) It cannot be determined from the information given
* Solution using a Pythagorean identity:
𝑥 2 + 𝑦 2 = (cos 𝜃)2 + (sin 𝜃)2 = 1
This is choice (C).
Notes: (1) (cos 𝜃)2 is usually abbreviated as cos2 𝜃.
Similarly, (sin 𝜃)2 is usually abbreviated as sin2 𝜃.
In particular, (cos 𝜃)2 + (sin 𝜃)2 would be written as cos2 𝜃 + sin2 𝜃.
(2) One of the most important trigonometric identities is the Pythagorean
Identity which says
𝐜𝐨𝐬 𝟐 𝒙 + 𝐬𝐢𝐧𝟐 𝒙 = 𝟏.
2
11. A line with slope 3 is translated up 5 units and right 1 unit. What
is the slope of the new line?
* Any translation of a line is parallel to the original line and therefore has
the same slope. The new line therefore has a slope of 𝟐/𝟑.
Notes: (1) If we only moved some of the points on the line, then the slope
might change. But here we are moving all points on the line
simultaneously. Therefore the exact shape and orientation of the line are
preserved.
(2) We could also grid in one of the decimals .666 or .667.
(3) If the solution is not clear, it is recommended that you draw a picture.
2
Start by drawing a line with slope 3. One way to do this would be to plot
points at (0,0) and (3,2) and then draw a line through these two points.

27
[Link]

Now take those same two points and move them up 5 units and right 1
unit to the points (1,5) and (4,7). Draw a line through these two points.
Note that the two lines are parallel.
(4) Recall that the formula for the slope of a line is
rise 𝑦 −𝑦
Slope = 𝑚 = run
= 𝑥2 −𝑥1
2 1

Let’s verify that the slopes of the two lines mentioned in note (3) are the
same.
2−0 2
For the line passing through (0,0) and (3,2), the slope is 3−0 = 3, and for
7−5 2
the line passing through (1,5) and (4,7), the slope is = .
4−1 3

So we see that the two slopes are equal.


(5) Parallel lines always have the same slope.

12. In the figure above, adjacent sides meet at right angles and the
lengths given are in inches. What is the perimeter of the figure,
in inches?

* Solution by moving the sides of the figure around: Recall that to


compute the perimeter of the figure we need to add up the lengths of all 8
line segments in the figure. We “move” the two smaller vertical segments
to the right, and each of the smaller horizontal segments up or down as
shown below.

28
[Link]

Note that the “bold” length is equal to the “dashed” length. We get a
rectangle with length 30 and width 15. Thus, the perimeter is
(2)(30) + (2)(15) = 60 + 30 = 90.
Warning: Although lengths remain unchanged by moving line segments
around, areas will be changed. This method should not be used in
problems involving areas.

LEVEL 1: PASSPORT TO ADVANCED MATH


13. If 2𝑥 2 − 11 = 5 − 2𝑥 2 , what are all possible values of x ?
(A) 2 only
(B) −2 only
(C) 0 only
(D) 2 and −2 only
Solution by plugging in the answer choices: According to the answer
choices we need only check 0, 2, and −2.
𝑥 = 0: 2(0)2 − 11 = 5 − 2(0)2 −11 = 5 False
𝑥 = 2: 2(2)2 − 11 = 5 − 2(2)2 −3 = −3 True
𝑥 = −2: 2(−2)2 − 11 = 5 − 2(−2)2 −3 = −3 True
So the answer is choice (D).
Notes: (1) Since all powers of 𝑥 in the given equation are even, 2 and −2
must give the same answer. So we didn’t really need to check −2.
(2) Observe that when performing the computations above, the proper
order of operations was followed. Exponentiation was done first,
followed by multiplication, and then subtraction was done last.

29
[Link]

For example, we have 2(2)2 − 11 = 2 ⋅ 4 − 11 = 8 − 11 = −[Link]


5 − 2(2)2 = 5 − 2 ⋅ 4 = 5 − 8 = −3.
Order of Operations: Here is a quick review of order of operations.
PEMDAS
P Parentheses
E Exponentiation
M Multiplication
D Division
A Addition
S Subtraction
Note that multiplication and division have the same priority, and addition
and subtraction have the same priority.
* Algebraic solution: We add 2𝑥 2 to each side of the given equation to
get 4𝑥 2 − 11 = 5. We then add 11 to get 4𝑥 2 = 5 + 11 = 16. Dividing
16
each side of this last equation by 4 gives 𝑥 2 = = 4. We now use the
4
square root property to get 𝑥 = ±2. So the answer is choice (D).
Notes: (1) The equation 𝑥 2 = 4 has two solutions: 𝑥 = 2 and 𝑥 = −2. A
common mistake is to forget about the negative solution.

(2) The square root property says that if 𝑥 2 = 𝑐, then 𝑥 = ±√𝑐.


This is different from taking the positive square root of a number. For
example, √4 = 2, whereas the equation 𝑥 2 = 4 has two solutions 𝑥 = ±2.
(3) Another way to solve the equation 𝑥 2 = 4 is to subtract 4 from each
side of the equation, and then factor the difference of two squares as
follows:
𝑥2 − 4 = 0
(𝑥 − 2)(𝑥 + 2) = 0
We now set each factor equal to 0 to get 𝑥 − 2 = 0 or 𝑥 + 2 = 0.
So 𝑥 = 2 or 𝑥 = −2.
14. A function 𝑔(𝑥) is defined as 𝑔(𝑥) = −5𝑥 2 . What is 𝑔(−2)?
(A) −100
(B) −20
(C) 20
(D) 50

30
[Link]

2
* 𝑔(−2) = −5(– 2) = −5(4) = −20, choice (B).
Notes: (1) The variable x is a placeholder. We evaluate the function 𝑔 at
a specific value by substituting that value in for x. In this question we
replaced x by –2.
(2) The exponentiation was done first, followed by the multiplication. See
the end of the solution to problem 13 for more information on order of
operations.
(3) To square a number means to multiply it by itself. So
(–2)2 = (–2)( –2) = 4.
(4) We can do the whole computation in our calculator (if a calculator is
allowed for the problem) in one step. Simply type -5(-2)^2 ENTER. The
output will be −20.
Make sure to use the minus sign and not the subtraction symbol.
Otherwise the calculator will give an error.

𝑥2 − 𝑦2 = 9
𝑥2 𝑦2
16
+ 4
=1
𝑥 + 2𝑦 = 4

15. A system of three equations in two unknowns and their graphs


in the 𝑥𝑦-plane are shown above. How many solutions does the
system have?
(A) None
(B) Two
(C) Four
(D) Six
* Solution by looking at the graph: There is no point that is common to
all three graphs. So the system has no solutions, choice (A).

31
[Link]

Notes: (1) A solution to the system of equations is a point that satisfies


all three equations simultaneously. Graphically this means that the point
is on all three graphs. Although there are several points that are common
to two of the graphs, there are none that are common to all three.
(2) The graph of the equation 𝑥 2 − 𝑦 2 = 9 is the hyperbola in the figure
above with vertices (−3,0) and (3,0).
𝑥2 𝑦2
(3) The graph of the equation + = 1 is the ellipse in the figure
16 4
above with vertices (−4,0), (4, 0), (0,2), and (0, −2)
(4) The graph of the equation 𝑥 + 2𝑦 = 4 is the line in the figure above
with intercepts (4, 0) and (0,2).
(4,0) is the 𝒙-intercept of the line, and (0,2) is the 𝒚-intercept of the
line.
(5) Consider the following system of equations:
𝑥2 𝑦2
16
+ 4
=1
𝑥 + 2𝑦 = 4
This system has the two solutions (0,2) and (4,0). These are the two
points common to the graphs of these two equations (the ellipse and the
line), also known as points of intersection of the two graphs.
(6) Consider the following system of equations:
𝑥2 − 𝑦2 = 9
𝑥 + 2𝑦 = 4
This system also has two solutions. These are the two points common to
the hyperbola and the line. Finding these two solutions requires solving
the system algebraically, which we won’t do here. One of these solutions
can be seen on the graph. It looks to be approximately (3.1,0.5). The
second solution does not appear on the portion of the graph that is
displayed. If we continued to graph the line and hyperbola to the left we
would see them intersect one more time.
(6) Consider the following system of equations:
𝑥2 − 𝑦2 = 9
𝑥2 𝑦2
16
+ 4
=1

32
[Link]

This system has four solutions. These are the four points common to the
hyperbola and the ellipse. Finding these four solutions requires solving
the system algebraically, which we won’t do here. These solutions can be
seen clearly on the graph.
* Algebraic solution: Observe from the graph that the points (0,2) and
(4,0) are intersection points of the line and the ellipse. In other words
they are solutions to the following system:
𝑥2 𝑦2
16
+ 4
=1
𝑥 + 2𝑦 = 4
We can verify this by substituting each point into each equation.
𝑥2 𝑦2 02 22 4
(0,2): 16
+ 4
= 1 ⇔ 16 + 4
= 1 ⇔4 = 1 ⇔1 = 1
𝑥 + 2𝑦 = 4 ⇔ 0 + 2(2) = 4 ⇔ 4 = 4
𝑥2 𝑦2 42 02 16
(4,0): 16
+ 4
= 1 ⇔ 16 + 4
= 1 ⇔ 16 = 1 ⇔ 1 = 1
𝑥 + 2𝑦 = 4 ⇔ 4 + 2(0) = 4 ⇔ 4 = 4
When we plug each of these points into the equation for the hyperbola
however, we get the following:
(0,2): 𝑥 2 − 𝑦 2 = 9 ⇔ 02 − 22 = 9 ⇔ −4 = 9
(4,0): 𝑥 2 − 𝑦 2 = 9 ⇔ 42 − 02 = 9 ⇔ 16 = 9
Since we wound up with false equations, neither of these points are on
the hyperbola.
It follows that the system of equations has no solutions, choice (A).
Notes: (1) Although I do not recommend this for this problem, we can
solve the following system formally using the substitution method.
𝑥2 𝑦2
16
+ 4
=1
𝑥 + 2𝑦 = 4
Let’s begin by solving the second equation for 𝑥 by subtracting 2𝑦 from
each side of the equation to get 𝑥 = 4 − 2𝑦.
We now replace 𝑥 by 4 − 2𝑦 in the first equation and solve for 𝑦.
𝑥2 𝑦2
16
+ 4
= 1
(4−2𝑦)2 𝑦2
16
+ 4
=1

33
[Link]

We multiply each side of this last equation by 16 to get


(4 − 2𝑦)2 + 4𝑦 2 = 16
Now (4 − 2𝑦)2 = (4 − 2𝑦)(4 − 2𝑦) = 16 − 8𝑦 − 8𝑦 + 4𝑦 2 . So we
have
16 − 8𝑦 − 8𝑦 + 4𝑦 2 + 4𝑦 2 = 16
We cancel the 16 from each side and combine like terms on the left to get
−16𝑦 + 8𝑦 2 = 0
−16𝑦 8𝑦 2
We factor −8𝑦 and note that −8𝑦
= 2 and −8𝑦 = −𝑦 to get
−8𝑦(2 − 𝑦) = 0
We now set each factor equal to zero.
−8𝑦 = 0 or 2−𝑦 =0
So we get the two solutions 𝑦 = 0 and 𝑦 = 2.
We can now substitute these 𝑦-values into either equation. Let’s use the
equation of the line since it’s simpler:
𝑦 = 0: 𝑥 + 2𝑦 = 4 ⇔ 𝑥 + 2(0) = 4 ⇔ 𝑥 = 4
𝑦 = 2: 𝑥 + 2𝑦 = 4 ⇔ 𝑥 + 2(2) = 4 ⇔ 𝑥 + 4 = 4 ⇔ 𝑥 = 0
So we see that the two points of intersection of the ellipse and the line are
(4,0) and (0,2).
(2) If we wanted to find the intersection points of the line and the
hyperbola we would solve the following system as we did in note (1):
𝑥2 − 𝑦2 = 9
𝑥 + 2𝑦 = 4
In this case however the algebra will be much messier and the solutions
do not “look very nice.”
It will never be necessary to do such messy algebra on the SAT, so we
leave this an optional exercise for the interested reader.
Similarly for the intersection points of the hyperbola and the ellipse we
would solve the following system:
𝑥2 − 𝑦2 = 9
𝑥2 𝑦2
+ =1
16 4

34
[Link]

Again, the algebra here is messy, and we leave this as an optional


exercise.
16. Which of the following graphs could not be the graph of a
function?

* Only choice (D) fails the vertical line test. In other words, we can draw
a vertical line that hits the graph more than once:

So the answer is choice (D).


𝑓(𝑥) = 5𝑥 + 3
𝑔(𝑥) = 𝑥 2 − 5𝑥 + 2

17. The functions 𝑓 and 𝑔 are defined above. What is the value of
𝑓(10) − 𝑔(5)?
* We have
𝑓(10) = 5(10) + 3 = 50 + 3 = 53
and
𝑔(5) = 52 − 5(5) + 2 = 25 − 25 + 2 = 2.
Therefore 𝑓(10) − 𝑔(5) = 53 − 2 = 𝟓𝟏.

35
[Link]

𝑥 𝑝(𝑥) 𝑞(𝑥) 𝑟(𝑥)


1 5 6 11
2 –3 7 –10
3 –4 –7 3
4 –5 –7 –2
5 –6 0 5

18. The table above gives some values of the functions 𝑝, 𝑞, and 𝑟.
At which value of 𝑥 does 𝑞(𝑥) = 𝑝(𝑥) + 𝑟(𝑥)?
Solution by guessing: The answer is an integer between 1 and 5
inclusive (these are the 𝑥-values given). So let’s start with 𝑥 = 3 as our
first guess. From the table 𝑝(3) = −4, 𝑞(3) = −7, and 𝑟(3) = 3.
Therefore 𝑝(3) + 𝑟(3) = −4 + 3 = −1. This is not equal to 𝑞(3) = −7
so that 3 is not the answer.
Let’s try 𝑥 = 4 next. From the table 𝑝(4) = −5, 𝑞(4) = −7, and
𝑟(4) = −2. So 𝑝(4) + 𝑟(4) = −5 + (−2) = −7 = 𝑞(4).
Therefore the answer is 4.
* Quick solution: We can just glance at the rows quickly and observe
that in the row corresponding to 𝑥 = 4, we have −5 + (−2) = −7. Thus,
the answer is 4.

36
[Link]

LEVEL 1: PROBLEM SOLVING AND DATA


Questions 19 - 21 refer to the following information.
Ten 25 year old men were asked how many hours per week they
exercise and their resting heart rate was taken in beats per
minute (BPM). The results are shown as points in the scatterplot
below, and the line of best fit is drawn.

19. How many of the men have a resting heart rate that differs by
more than 5 BPM from the resting heart rate predicted by the
line of best fit?
(A) None
(B) Two
(C) Three
(D) Four
* The points that are more than 5 BPM away from the line of best fit
occur at 1, 4, and 8 hours. So there are Three of them, choice (C).
Notes: (1) One of the two men that exercise 1 hour per week has a resting
heart rate of approximately 68 BPM. The line of best fit predicts
approximately 77 BPM. So this difference is 77 – 68 = 9 BPM.
Similarly, at 4 we have a difference of approximately 75 – 67 = 8 BPM,
and at 8 we have a difference of approximately 60 – 54 = 6 BPM.

37
[Link]

(2) At 5, the point below the curve corresponds to a heart rate that differs
from that predicted by the line of best fit by approximately 64 – 59 = 5
BPM. Since this is not more than 5, we do not include this point in the
count.
20. Based on the line of best fit, what is the predicted resting heart
rate for someone that exercises three and a half hours per week?
(A) 66 BPM
(B) 68 BPM
(C) 70 BPM
(D) 72 BPM
* The point (3.5, 68) seems to be on the line of best fit. So the answer is
68 BPM, choice (B).
21. Which of the following is the best interpretation of the slope of
the line of best fit in the context of this problem?
(A) The predicted number of hours that a person must
exercise to maintain a resting heart rate of 50 BPM.
(B) The predicted resting heart rate of a person that does not
exercise.
(C) The predicted decrease in resting heart rate, in BPM, for
each one hour increase in weekly exercise.
(D) The predicted increase in the number of hours of exercise
needed to decrease the resting heart rate by one BPM.
change in predicted heart rate
* The slope of the line is the change in hours of exercise
. If we make the
denominator a 1 hour increase, then the fraction is the change in predicted
heart rate per 1 hour increase. Since the line is moving downward from
left to right, we can replace “change” in the numerator by “decrease.” So
the answer is choice (C).
*Note: Recall that the slope of a line is
rise change in vertical distance
Slope = 𝑚 = run
= change in horizontal distance

In this problem the change in vertical distance is the change in resting


heart rate, in BPM, and the change in horizontal distance is the change in
hours of exercise per week.

38
[Link]

22. The mean annual salary of an NBA player, 𝑆, can be estimated


using the equation 𝑆 = 161,400(1.169)𝑡 , where 𝑆 is measured
in thousands of dollars, and 𝑡 represents the number of years
since 1980 for 0 ≤ 𝑡 ≤ 20. Which of the following statements is
the best interpretation of 161,400 in the context of this
problem?
(A) The estimated mean annual salary, in dollars, of an NBA
player in 1980.
(B) The estimated mean annual salary, in dollars, of an NBA
player in 2000.
(C) The estimated yearly increase in the mean annual salary
of an NBA player.
(D) The estimated yearly decrease in the mean annual salary
of an NBA player.
* When 𝑡 = 0, we have
𝑆 = 161,400(1.169)0 = 161,400(1) = 161,400.
Since 𝑡 = 0 corresponds to the year 1980, it follows that 161,400 is the
estimated mean annual salary, in dollars, of an NBA player in 1980. This
is choice (A).
Notes: (1) The year 2000 corresponds with 𝑡 = 20. So the estimated
mean annual salary, in dollars, of an NBA player in 2000 would be
𝑆 = 161,400(1.169)20. This is a number much larger than 161,400 (it is
approximately 3,666,011).
(2) The function given in this problem is an exponential function. In
general, exponential functions have the form 𝑦 = 𝑎𝑏 𝑡 . Note that 𝑡 = 0
corresponds to 𝑦 = 𝑎. In other words, the initial amount is always 𝑎.
In this problem 𝑡 = 0 corresponds to the year 1980, and so the 161,400
gives the mean annual salary in 1980.
Unlike a linear function, an exponential function does not have a constant
slope. So in this problem the yearly increase or decrease in mean annual
salary cannot be described by a single number.
(3) Let’s compare this to the analogous linear function. Suppose for a
moment that the equation given instead was
𝑆 = 1.169𝑡 + 161,400

39
[Link]

In this case, the number 161,400 would still describe the estimated mean
annual salary, in dollars, of an NBA player in 1980.
The number 1.169 would describe the estimated yearly increase in the
mean annual salary of an NBA player.
23. A biologist was interested in the number of times a field cricket
chirps each minute on a sunny day. He randomly selected 100
field crickets from a garden, and found that the mean number of
chirps per minute was 112, and the margin of error for this
estimate was 6 chirps. The biologist would like to repeat the
procedure and attempt to reduce the margin of error. Which of
the following samples would most likely result in a smaller
margin of error for the estimated mean number of times a field
cricket chirps each minute on a sunny day?
(A) 50 randomly selected crickets from the same garden.
(B) 50 randomly selected field crickets from the same
garden.
(C) 200 randomly selected crickets from the same garden.
(D) 200 randomly selected field crickets from the same
garden.
* Increasing the sample size while keeping the population the same will
most likely decrease the margin of error. So the answer is choice (D).
Notes: (1) Decreasing the sample size will increase the margin of error.
This allows us to eliminate choices (A) and (B).
(2) The original sample consisted of only field crickets. If we were to
allow the second sample to include all crickets, then we have changed the
population. We cannot predict what impact this would have on the mean
and margin of error. This allows us to eliminate choice (C).
Technical note: In reality there is a correlation between the frequency of
cricket chirps and temperature. You can estimate the current temperature,
in degrees Fahrenheit, by counting the number of times a cricket chirps in
15 seconds and adding 37 to the result.

40
[Link]

24. A survey was conducted among a randomly chosen sample of


250 single men and 250 single women about whether they
owned any dogs or cats. The table below displays a summary of
the survey results.

Dogs Cats
Both Neither Total
Only Only
Men 92 14 18 126 250
Women 75 42 35 98 250
Total 167 56 53 224 500

What fraction of the people surveyed who said they own dogs
are women?
* There are 75 + 35 = 110 women who said they own dogs, and there
are a total of 167 + 53 = 220 people who said they own dogs. Therefore
110
the fraction of reported dog owners that are women is 220 = 𝟏/𝟐 or . 𝟓.

Notes: (1) There are two columns that represent people who said they
own dogs: the column labeled “Dogs Only,” and the column labeled
“Both.”
Remember that the word “Both” indicates both dog and cat ownership,
and in particular dog ownership.
(2) The numerator of the fraction is the number of women who said they
own dogs. There are 75 women who said they own dogs only, and 35
women who said they own both dogs and cats. Therefore there are a total
of 75 + 35 = 110 women who said they own dogs.
(3) The denominator of the fraction is the number of people who said
they own dogs. There are 167 people who said they own dogs only, and
53 people who said they own both dogs and cats. Therefore there are a
total of 167 + 53 = 220 people who said they own dogs.
(4) This question is very closely related to conditional probability. See
problem 115 for more details about this concept.

41
[Link]

LEVEL 2: HEART OF ALGEBRA


5𝑘+50
25. Which of the following expressions is equivalent to ?
5

(A) 𝑘 + 10
(B) 𝑘 + 50
(C) 7𝑘 + 10
(D) 11𝑘
Solution by picking a number: Let’s choose a value for 𝑘, say 𝑘 = 2.
We first substitute a 2 in for 𝑘 into the given expression. If we can use
our calculator we type in the following: (5*2 + 50) / 5 to get 𝑘 = 𝟏𝟐. If a
calculator is not allowed, it’s not too hard to do the previous computation
by hand. Put a nice big, dark circle around this number so that you can
find it easily later. We now substitute a 2 for 𝑘 into each answer choice.
(A) 12
(B) 52
(C) 24
(D) 22
We now compare each of these numbers to the number that we put a nice
big, dark circle around. Since (B), (C), and (D) are incorrect we can
eliminate them. Therefore the answer is choice (A).
Important note: (A) is not the correct answer simply because it is equal
to 12. It is correct because all three of the other choices are not 12. You
absolutely must check all four choices!
Algebraic solution: Most students have no trouble at all adding two
fractions with the same denominator. For example,
5𝑘 50 5𝑘 + 50
+ =
5 5 5
But these same students have trouble reversing this process.
5𝑘 + 50 5𝑘 50
= +
5 5 5
Note that these two equations are identical except that the left and right
hand sides have been switched. Note also that to break a fraction into two
(or more) pieces, the original denominator is repeated for each piece.
* An algebraic solution to the above problem consists of the following
quick computation
5𝑘 + 50 5𝑘 50
5
= 5
+ 5
= k + 10, choice (A).

42
[Link]

26. If 𝑖 = √−1, then (7 + 5𝑖)(−2 − 6𝑖) =


(A) 16
(B) −44
(C) 16 − 52𝑖
(D) −44 − 52𝑖
* (7 + 5𝑖)(−2 − 6𝑖) = (−14 + 30) + (−42 − 10)𝑖 = 16 − 52𝑖, choice
(C).
Notes: (1) Here we used the following formula for multiplying two
complex numbers:
(𝒂 + 𝒃𝒊)(𝒄 + 𝒅𝒊) = (𝒂𝒄 − 𝒃𝒅) + (𝒂𝒅 + 𝒃𝒄)𝒊
One option is to have this formula memorized, although this is not
necessary (see note(3) below).
2
(2) Since 𝑖 = √−1, it follows that 𝑖 2 = (√−1) = −1.
(3) We can multiply the two complex numbers by using the distributive
property and replacing 𝑖 2 by −1.
(7 + 5𝑖)(−2 − 6𝑖) = (7 + 5𝑖)(−2) + (7 + 5𝑖)(−6𝑖)
= −14 − 10𝑖 − 42𝑖 − 30𝑖 2 = −14 − 52𝑖 − 30(−1) = −14 + 30 − 52𝑖
= 16 − 52𝑖
(4) We can also use the shortcut of FOILing like many of us do for
multiplication of binomials.
(7 + 5𝑖)(−2 − 6𝑖) = (7)(−2) + 7(−6𝑖) + (5𝑖)(−2) + (5𝑖)(−6𝑖)
= −14 − 42𝑖 − 10𝑖 − 30𝑖 2 = −14 − 52𝑖 − 30(−1) = −14 + 30 − 52𝑖
= 16 − 52𝑖
3 12
27. If = , where 𝑧 ≠ 0, what is 𝑧 in terms of 𝑥 ?
𝑥 2 +2 𝑧
2
(A) 4𝑥 2 +
3
(B) 4𝑥 2 + 8
(C) 4𝑥 2 + 24
3
(D) √2 𝑥 − 2

* Algebraic solution: We begin by cross multiplying to get

43
[Link]

3𝑧 = 12(𝑥 2 + 2) = 12𝑥 2 + 24.


We now divide each side of this equation by 3 to get
12𝑥 2 +24 12𝑥 2 24
𝑧= 3
= 3
+ 3
= 4𝑥 2 + 8.
This is choice (B).
Notes: (1) Be careful to distribute the 12 properly. A common mistake is
to write 12(𝑥 2 + 2) = 12𝑥 2 + 2 instead of 12𝑥 2 + 24.
(2) Another common mistake is to divide a binomial incorrectly. For
12𝑥 2 +24
example, some students will write 3
= 4𝑥 2 + 24. This is not
𝑎+𝑏 𝑎
correct. In other words, in general ≠ + 𝑏.
𝑐 𝑐
𝑎+𝑏 𝑎 𝑏
The correct equation is 𝑐
= 𝑐 + 𝑐.

(3) Another way to perform the division is to factor first and then divide:
12𝑥 2 +24 12(𝑥 2 +2)
𝑥= 3
= 3
= 4(𝑥 2 + 2),

Note however that with the solution above this would create an extra step.
(4) As an alternative, we can solve 3𝑧 = 12(𝑥 2 + 2) for 𝑧 by first
dividing by 3 to get 𝑧 = 4(𝑥 2 + 2) = 4𝑥 2 + 8. Once again, be careful
about distributing correctly at the end.
Solution by picking a number: Let’s pick a value for 𝑥, say 𝑥 = 2. The
3 3 3 1
left hand side of the given equation then becomes 𝑥 2 +2 = 22 +2 = 6 = 2.
12 1
So we must have = , and it follows that 𝑧 = 𝟐𝟒.
𝑧 2

Put a nice big, dark circle around the number 24 so you can find it easily
later. We now substitute 𝑥 = 2 into each answer choice:
2 2
(A) 4(2)2 + 3 = 16 + 3 ≈ 16.67
(B) 4(2)2 + 8 = 16 + 8 = 24
(C) 4(2)2 + 24 = 16 + 24 = 40
3
(D) √2 (2) − 2 = √3 − 2 = √1 = 1

Since choices (A), (C), and (D) did not come out correct, the answer is
choice (B).

44
[Link]

Important note: (B) is not the correct answer simply because it came
out to 24. It is correct because all three of the other choices did not come
out correct.
28. Tickets for a concert cost $4.50 for children and $12.00 for
adults. 4460 concert tickets were sold for a total cost of $29,220.
Solving which of the following systems of equations yields the
number of children, 𝑐, and number of adults, 𝑎, that purchased
concert tickets?
(A) 𝑐 + 𝑎 = 4460
4.50𝑐 + 12𝑎 = 58,440
(B) 𝑐 + 𝑎 = 4460
4.50𝑐 + 12𝑎 = 29,220
(C) 𝑐 + 𝑎 = 4460
4.50𝑐 + 12𝑎 = 14,610
(D) 𝑐 + 𝑎 = 29,220
4.50𝑐 + 12𝑎 = 4460
* 𝑐 is the number of tickets sold to children, 𝑎 is the number of tickets
sold to adults, and 4460 is the total number of tickets sold. It follows that
𝑐 + 𝑎 = 4460.
Since each children’s ticket costs $4.50, it follows that 4.50𝑐 is the total
cost for children’s tickets.
Similarly, since each adult’s ticket costs $12, it follows that 12𝑎 is the
total cost for adult’s tickets.
So 4.50𝑐 + 12𝑎 is the total ticket cost, and so 4.50𝑐 + 12𝑎 = 29,220.
So the answer is choice (B).
7 − (4 − 𝑞) 3(5 − 𝑞)
=
8 12
29. In the equation above, what is the value of q ?
* Algebraic solution: We first apply the distributive property in each
7−4+𝑞 15−3𝑞
numerator to get = .
8 12
We then combine like terms in the numerator on the left hand side to get
3+𝑞 15−3𝑞
= .
8 12
Now cross multiply: 12(3 + 𝑞) = 8(15 − 3𝑞).
Distribute again on each side: 36 + 12𝑞 = 120 − 24𝑞.

45
[Link]

Add 24𝑞 to and subtract 36 from each side: 36𝑞 = 84.


84 𝟕
Divide each side by 36: 𝑞 = = or 2.33.
36 𝟑
Note: Make sure you are using the distributive property correctly here.
For example 3(5 − 𝑞) = 15 − 3𝑞. A common mistake would be to write
3(5 − 𝑞) = 15 − 𝑞.
An even more common mistake would be to write −(4 − 𝑞) = −4 − 𝑞.
If you frequently fall into this trap, it might help to first rewrite −(4 − 𝑞)
as −1(4 − 𝑞). So we have −(4 − 𝑞) = −1(4 − 𝑞) = −4 + 𝑞.
5𝑧 2𝑧
4 4
4𝑡 𝑤
8 8
+9 +9
52 34
30. In the correctly worked addition problems above, what is the
value of 3𝑧 + 4𝑡 − 𝑤?
Solution by trying a simple operation: Let’s rewrite the equations
horizontally since that is how most of us are used to seeing equations.
5z + 4 + 4t + 8 + 9 = 52
2z + 4 + w + 8 + 9 = 34
We now use a simple operation. The operation to use here is subtraction.
Let’s go ahead and subtract term by term.
5z + 4 + 4t + 8 + 9 = 52
2z + 4 + w + 8 + 9 = 34
3z + (4t – w) = 18
Remark: Whenever we are trying to find an expression that involves
addition, subtraction, or both, adding or subtracting the given equations
usually does the trick.
* Visualizing the answer: You can save a substantial amount of time by
performing the subtraction in your head (left equation minus right
equation).
Note that above the lines the subtraction yields 3z + 4t – w. This is
exactly what we’re looking for. Thus, we need only subtract below the
lines to get the answer: 52 – 34 = 18.

46
[Link]

Solution by picking numbers: If we choose any value for z, then t and w


will be determined. So, let’s set z equal to 0. Then
4 + 4t + 8 + 9 = 52
4t + 21 = 52
4t = 31
31
t = = 7.75
4
and
4 + w + 8 + 9 = 34
w + 21 = 34
w = 13
So 3z + 4t – w = 0 + 4(7.75) – 13 = 18.
Remarks: (1) Any choice for z will give us the same answer. We could
have chosen a value for t or w as well. But once we choose a value for
one of the variables the other two are determined.
(2) It was actually unnecessary to solve for t above. We could have
stopped at 4t = 31. We then have 3z + 4t – w = 0 + 31 – 13 = 18.

LEVEL 2: GEOMETRY AND TRIG


31. A rectangle has a perimeter of 16 meters and an area of 15
square meters. What is the longest of the side lengths, in meters,
of the rectangle?
(A) 3
(B) 5
(C) 10
(D) 15
* Solution by plugging in answer choices: Let’s start with choice (C)
and guess that the longest side of the rectangle is 10 meters long. But then
the length of the two longer sides of the rectangle adds up to 20 meters
which is greater than the perimeter. So we can eliminate (C) and (D).
Let’s try choice (B) next. So we are guessing that the longest side of the
rectangle is 5 meters long. Since the perimeter is 16, it follows that the
shortest side must have length 3 (see Remark (1) below for more
clarification). So the area is (5)(3) = 15. Since this is correct, the answer
is choice (B).

47
[Link]

Remarks: (1) If one side of the rectangle has a length of 5 meters, then
the opposite side also has a length of 5 meters. Since the perimeter is 16
meters, this leaves 16 – 5 – 5 = 6 meters for the other two sides. It follows
6
that a shorter side of the rectangle has length 2 = 3 meters.

(2) When guessing the longest side of the rectangle we can use the area
instead of the perimeter to find the shortest side. For example, if we guess
that the longest side is 5, then since the area is 15 it follows that the
shortest side is 3. We would then check to see if we get the right
perimeter. In this case we have P = 2(5) + 2(3) = 16 which is correct.
Algebraic solution: We are given that 2x + 2y = 16 and xy = 15. If we
divide each side of the first equation by 2, we get x + y = 8. Subtracting
each side of this equation by x, we get y = 8 – x.
We now replace y by 8 – x in the second equation to get x(8 – x) = 15.
Distributing the x on the left yields 8x – x2 = 15. Subtracting 8x and
adding x2 to each side of this equation gives us 0 = x2 – 8x + 15. The right
hand side can be factored to give 0 = (x – 5)(x – 3). So we have x – 5 = 0
or x – 3 = 0. So x = 5 or x = 3. Since the question asks for the longest of
the side lengths, the answer is x = 5, choice (B).
Notes: (1) Here is a picture for extra clarification.

(2) The perimeter of a rectangle is 𝑷 = 𝟐𝒍 + 𝟐𝒘.


(3) The area of a rectangle is 𝑨 = 𝒍𝒘.

48
[Link]

32. In the figure above, ∠𝑀𝑃𝑄 ≅ ∠𝑆𝑇𝑈. Each of the following


statements must be true EXCEPT
(A) ̅̅̅̅
𝑃𝑄 ∥ ̅𝑆𝑇
̅̅̅
(B) 𝑚∠𝑀𝑃𝑄 + 𝑚∠𝑅𝑇𝑆 = 180°
(C) Δ𝑃𝑄𝑅 ∼ Δ𝑇𝑆𝑅
(D) Δ𝑃𝑄𝑅 ≅ Δ𝑇𝑆𝑅
* Solution by drawing another representation of the figure: Since the
figure is not drawn to scale, let’s draw a second representation of the
figure,. different from the given one, that satisfies the given condition.

Note that we drew the figure so that ∠𝑀𝑃𝑄 ≅ ∠𝑆𝑇𝑈 is still true. Since
Δ𝑃𝑄𝑅 is clearly larger than Δ𝑇𝑆𝑅, we see that Δ𝑃𝑄𝑅 ≅ Δ𝑇𝑆𝑅 can be
false, choice (D).
Notes: (1) The symbol ∥ stands for “parallel,” so that ̅̅̅̅
𝑃𝑄 ∥ ̅𝑆𝑇
̅̅̅ is read
“segment 𝑃𝑄 is parallel to segment 𝑆𝑇.”
(2) 𝑚∠𝑀𝑃𝑄 is read “the measure of angle 𝑀𝑃𝑄.” So choice (B) can be
read “the sum of the measures of angle 𝑀𝑃𝑄 and angle 𝑅𝑇𝑆 is 180
degrees.”

49
[Link]

(3) The symbol ∼ stands for “similar,” so that Δ𝑃𝑄𝑅 ∼ Δ𝑇𝑆𝑅 is read
“triangle 𝑃𝑄𝑅 is similar to triangle 𝑇𝑆𝑅.”
Two triangles are similar if they have the same angle measures.
(4) In this problem, ∠𝑃𝑅𝑄 and ∠𝑇𝑅𝑆 are vertical angles. Since vertical
angles have the same measure, 𝑚∠𝑃𝑅𝑄 = 𝑚∠𝑇𝑅𝑆.
It is also true that 𝑚∠𝑃𝑄𝑅 = 𝑚∠𝑇𝑆𝑅 and 𝑚∠𝑄𝑃𝑅 = 𝑚∠𝑆𝑇𝑅. See the
next solution for details.
It follows that Δ𝑃𝑄𝑅 ∼ Δ𝑇𝑆𝑅.
(5) Two determine that two triangles are similar, it is sufficient to show
that two pairs of angles have the same measure. We get the third pair for
free because the angle measures in a triangle always sum to 180°.
(6) The symbol ≅ stands for “congruent,” so that ∠𝑀𝑃𝑄 ≅ ∠𝑆𝑇𝑈 is read
“angle 𝑀𝑃𝑄 is congruent to angle 𝑆𝑇𝑈,” and Δ𝑃𝑄𝑅 ≅ Δ𝑇𝑆𝑅 is read
“triangle 𝑃𝑄𝑅 is congruent to triangle 𝑇𝑆𝑅.”
Two line segments are congruent if they have the same length. Two
angles are congruent if they have the same measure. Two triangles are
congruent if all corresponding sides and interior angles are congruent.
In the last figure we drew, we have ∠𝑃𝑅𝑄 ≅ ∠𝑇𝑅𝑆 (because they are
vertical angles), but Δ𝑃𝑄𝑅 ≇ Δ𝑇𝑆𝑅.
* Solution by process of elimination: Let’s first redraw the picture with
just one transversal.

So we have the lines ̅̅̅̅


𝑃𝑄 and ̅𝑆𝑇 ̅̅̅ cut by the transversal ̅̅̅̅̅
𝑀𝑈 with
∠𝑀𝑃𝑄 ≅ ∠𝑆𝑇𝑈. By the converse to the alternate exterior angle theorem,
we have ̅̅̅̅
𝑃𝑄 ∥ ̅̅
𝑆𝑇̅̅. So we can eliminate choice (A).

It then follows that the parallel lines and transversal form 8 angles, any
two of which are congruent or supplementary. Since angles 𝑀𝑃𝑄 and

50
[Link]

𝑅𝑇𝑆 are clearly not necessarily congruent (in the above figure one is
obtuse, the other acute), they must be supplementary. It follows that
𝑚∠𝑀𝑃𝑄 + 𝑚∠𝑅𝑇𝑆 = 180°, and so we can eliminate choice (B).
Now since ̅̅̅̅
𝑃𝑄 ∥ ̅𝑆𝑇
̅̅̅, it follows that the alternate interior angles cut by the
transversal are congruent. That is ∠𝑄𝑃𝑅 ≅ ∠𝑆𝑇𝑅. In the original figure
we also have ∠𝑃𝑅𝑄 ≅ ∠𝑇𝑅𝑆 because vertical angles are congruent. It
follows that Δ𝑃𝑄𝑅 ∼ Δ𝑇𝑆𝑅, and so we can eliminate choice (C).
Since we have eliminated choices (A), (B), and (C), the answer is choice
(D).
Notes: (1) In the last figure above with lines ̅̅̅̅
𝑃𝑄 and ̅𝑆𝑇
̅̅̅ cut by the
transversal ̅̅̅̅̅
𝑀𝑈, we have the following definitions:
(a) ∠𝑀𝑃𝑄 and ∠𝑆𝑇𝑈 are called alternate exterior angles.
(b) ∠𝑄𝑃𝑅 and ∠𝑆𝑇𝑅 are called alternate interior angles.
(c) ∠𝑀𝑃𝑄 and ∠𝑅𝑇𝑉 are called corresponding angles.
Observe that there is 1 more pair of alternate exterior angles, 1 more pair
of alternate interior angles, and 3 more pairs of corresponding angles.
Can you find them?
(2) If the lines ̅̅̅̅
𝑃𝑄 and ̅𝑆𝑇
̅̅̅ happen to be parallel, then the alternate
exterior angles formed are congruent. This is known as the alternate
exterior angle theorem.
(3) There are also two similar theorems for alternate interior angles and
corresponding angles.
(4) The converses of each of these theorems are also true. For example,
the converse to the alternate exterior angle theorem says “If two lines are
cut by a transversal and the alternate exterior angles formed are
congruent, then the two lines are parallel.
Can you state the converses to the other two analogous theorems?
(5) To summarize the three main theorems, whenever parallel lines are
cut by a transversal, eight angles are formed.
We can split these eight angles into two groups of four. Any two angles
in each group are congruent, and if we take two angles from different
groups they are supplementary (their measures add to 180°).
For example, ∠𝑀𝑃𝑄 ≅ ∠𝑆𝑇𝑈, whereas 𝑚∠𝑀𝑃𝑄 + 𝑚∠𝑅𝑇𝑆 = 180°.

51
[Link]

(6) Many of the notes from the first solution to this problem are relevant
for this solution as well.
33. In ∆𝐶𝐴𝑇, ∠𝐴 is a right angle. Which of the following is equal to
tan T ?
𝐶𝐴
(A)
𝐶𝑇
𝐶𝐴
(B) 𝐴𝑇
𝐶𝑇
(C)
𝐶𝐴
𝐶𝑇
(D) 𝐴𝑇

* Solution by drawing a picture: Let’s draw a picture.

OPP 𝐶𝐴
Now just note that tan 𝑇 = ADJ
= 𝐴𝑇, choice (B).

OPP
Remark: If you do not see why we have tan 𝑇 = ADJ
, review the basic
trigonometry given after the solution to problem 9.

34. As shown above, a 10-foot ramp forms an angle of 23° with the
ground, which is horizontal. Which of the following is an
expression for the vertical rise, in feet, of the ramp?
(A) 10 cos 23°
(B) 10 sin 23°
(C) 10 tan 23°
(D) 10 cot 23°

52
[Link]

OPP OPP
* We have sin 23° = HYP = 10
. So OPP = 10 sin 23°, choice (B).
OPP
Remarks: (1) If you do not see why we have sin 23° = , review the
HYP
basic trigonometry given after the solution to problem 9.
OPP
(2) To get from sin 23° = 10 to OPP = 10 sin 23°, we simply multiply
each side of the first equation by 10.
For those of you that like to cross multiply, the original equation can first
sin 23° OPP
be rewritten as = .
1 10

35. * In the figure above 𝜃 = 65°. What is the approximate value of


x?
(A) 5.07
(B) 5.60
(C) 10.87
(D) 13.24
OPP 12
* sin 𝜃 = = . Multiplying each side of this equation by 𝑥 gives us
HYP 𝑥
12 12
𝑥 sin 𝜃 = 12. So 𝑥 = = ≈ 13.24, choice (D).
sin 𝜃 sin 65°
OPP
Remarks: (1) If you do not see why we have sin 𝜃 = , review the
HYP
basic trigonometry given after the solution to problem 9.
(2) If you prefer, you can think of the multiplication above as cross
sin 𝜃
multiplication by first rewriting sin 𝜃 as 1 .
sin 𝜃 12
So we have 1 = 𝑥 . Cross multiplying yields 𝑥 sin 𝜃 = 1(12). This
yields the same result as in the above solution.
36. * In ∆𝐴𝐵𝐶 with right angle 𝐶, 𝐵𝐶 = 5 and cos 𝐵 = 0.7. What is
the length of AB ?

53
[Link]

* Solution by drawing a picture: Let’s draw a picture.

ADJ 5
cos 𝐵 = HYP. So we have 0.7 = 𝑥. Multiplying each side of this equation
5
by 𝑥 gives us 0.7𝑥 = 5. So 𝑥 = 0.7 ≈ 𝟕. 𝟏𝟒.

LEVEL 2: PASSPORT TO ADVANCED MATH


37. Suppose that ℎ(𝑥) = 4𝑥 − 5 and ℎ(𝑏) = 17. What is the value
of b ?
(A) 4
(B) 5.5
(C) 10
(D) 17.5
Solution by plugging in answer choices: Let’s start with choice (C) and
guess that b = 10. Then ℎ(𝑏) = 4b – 5 = 4(10) – 5 = 40 – 5 = 35. This is
too big. So we can eliminate choices (C) and (D).
Let’s try choice (B) next. So we are guessing that b = 5.5. We then have
that ℎ(𝑏) = 4b – 5 = 4(5.5) – 5 = 22 – 5 = 17. This is correct. So the
answer is choice (B).
* Algebraic solution: ℎ(𝑏) = 17 is equivalent to 4𝑏 − 5 = 17. We add
5 to each side of this equation to get 4b = 22. We then divide each side of
this equation by 4 to get that b = 5.5, choice (B).
38. Let a function of 2 variables be defined by
ℎ(𝑥, 𝑦) = 𝑥 2 + 3𝑥𝑦 − (𝑦 − 𝑥), what is the value of ℎ(5,4) ?
(A) 76
(B) 84
(C) 85
(D) 86

54
[Link]

* h(5,4) = 52 + 3(5)(4) – (4 – 5) = 25 + 60 – (–1) = 85 + 1 = 86, choice


(D).
Notes: (1) This is very similar to problem 14. Everywhere we see an x we
replace it by 5 and everywhere we see a y we replace it by 4. Remember
to follow the correct order of operations (see problem 13 for details).
(2) We can do the whole computation in our calculator (if allowed) in one
step. Simply type 5^2 + 3*5*4 – (4 – 5) ENTER. The output will be 86.
39. The expression 𝑥 2 − 𝑥 − 12 can be written as the product of
two binomial factors with integer coefficients. One of the
binomials is (𝑥 + 3). Which of the following is the other
binomial?
(A) 𝑥2 − 4
(B) 𝑥2 + 4
(C) 𝑥−4
(D) 𝑥+4
Solution by picking a number: Let’s choose a value for x, say x = 2.
Then x2 – x – 12 = 22 – 2 – 12 = 4 – 14 = –10 and (x + 3) = 2 + 3 = 5. So
10
the answer should be − = –2. Put a nice big dark circle around –2 so
5
you can find it easier later. We now substitute 2 for x into each answer
choice:
(A) 22 – 4 = 4 – 4 = 0
(B) 22 + 4 = 4 + 4 = 8
(C) 2 − 4 = −2
(D) 2+4=6
Since (A), (B) and (D) each came out incorrect, the answer is choice (C).
Important note: (C) is not the correct answer simply because it is equal
to –2. It is correct because all three of the other choices are not –2. You
absolutely must check all four choices!
Notes: (1) By picking the number x = 2, we have changed the problem to
“the number –10 can be written as a product of 5 and what other
number?” This is why the answer (to this new problem) is –2.
(2) A binomial has two terms. For example, the two terms of (x + 3) are x
and 3. The two terms of (x – 4) are x and −4.

55
[Link]

* Algebraic solution: We are being asked to factor x2 – x – 12. But we


12
are also given that one of the factors is (x + 3). Since − 3 = – 4, the other
factor must be (x – 4), choice (C).
Note: On the SAT, an expression of the form x2 + bx + c will usually
factor as (x + m)(x + n) where m and n are integers and 𝑚 ⋅ 𝑛 = 𝑐.
12
In this problem, c = –12 and m = 3. So n = − 3
= –4.
𝑥 2 −4
40. If 𝑔(𝑥) = and 𝑎 = −5, what is the value of |𝑔(𝑎)| ?
𝑥+2

We have a2 – 4 = (–5)2 – 4 = 25 – 4 = 21. Also a + 2 = –5 + 2 = –3. We


then divide 21 / (–3) = –7. So 𝑔(𝑎) = 𝑔(−5) = −7. It follows that
|𝑔(𝑎)| = 𝟕.
Remarks: (1) If you are doing these computations on your calculator,
make sure that –5 is put in parentheses before squaring: –5 ^ 2 will give
an output of –25 which is not correct.
(2) 𝑔(𝑎) can be computed with a single calculator computation as
follows:
( (–5) ^ 2 – 4) / (–5 + 2)
Note that the whole numerator is inside parentheses and the whole
denominator is inside parentheses.
(3) We can actually compute |𝑔(𝑎)| in one step with our calculator as
follows:
Press MATH, scroll over to NUM, press ENTER (to select abs( ), now
enter the expression from note (2), and press ENTER. The display looks
as follows:
abs(( (–5) ^ 2 – 4) / (–5 + 2)
Note that we do not need the closing parenthesis, although you can put it
in if you wish.
(4) I don’t necessarily recommend using the absolute value function on
the calculator for this. It’s quicker to do the computation without this, and
then just ignore the minus sign if it appears.
41. Define ⟦𝑥⟧ to be the largest integer less than 𝑥. What is the
value of ⟦√75⟧ ?

56
[Link]

* Calculator solution: If we take the square root of 75 in our calculator


we get approximately 8.66. The largest integer less than this is 8.

Solution without a calculator: 64 < 75 < 81. So √64 < √75 < √81.
Since √64 = 8 and √81 = 9, we have 8 < √75 < 9. So the largest
integer less than √75 is 8.
𝑠2 −𝑟 2
42. The operation & is defined as 𝑟 & 𝑠 = 𝑟+𝑠 where 𝑟 and 𝑠 are
real numbers and 𝑟 ≠ −𝑠. What is the value of 3 & 4 ?
42 −32 16−9 7
*3&4= 3+4
= 7
= 7 = 𝟏.

LEVEL 2: PROBLEM SOLVING AND DATA


43. * A chemist has a supply of 5.2 liter bottles of a certain solvent
that must be shipped to a central warehouse. The warehouse can
accept the solvent at the rate of 3 hectoliters per minute for a
maximum of 8 hours per day. If 1 hectoliter equals 100 liters,
what is the maximum number of bottles that the warehouse
could receive from the chemist each day?
(A) 461
(B) 462
(C) 27,692
(D) 83,200
* The warehouse can receive 3 ⋅ 60 ⋅ 8 = 1440 hectoliters of the solvent
per day, or equivalently, 1440 ⋅ 100 = 144,000 liters of the solvent per
day. Therefore the number of bottles that can be accepted each day is
144,000
5.2
≈ 27,692.30769. The maximum number of bottles that the
warehouse can accept in one day is therefore 27,692, choice (C).
Notes: (1) Since there are 60 minutes in an hour, “3 hectoliters per
minute” is the same as 3 ⋅ 60 = 180 hectoliters per hour.
Similarly, since the warehouse can accept the solvent for a maximum of 8
hours per day, “180 hectoliters per hour” is equivalent to a maximum of
180 ⋅ 8 = 1440 hectoliters per day.
(2) In the above solution we combined the two conversions given in note
(1) into a single conversion: “3 hectoliters per minute” is equivalent to a
maximum of 3 ⋅ 60 ⋅ 8 = 1440 hectoliters per day.

57
[Link]

(3) Since 1 hectoliter equals 100 liters, we can convert hectoliters to liters
by multiplying by 100. So
1440 hectoliters is equal to 1440 ⋅ 100 = 144,000 liters.
(4) We can convert between hectoliters and liters more formally by
setting up a ratio. The two things being compared are “liters” and
“hectoliters.”
liters 100 𝑥
hectoliters 1 1440
Now draw in the division symbols and equal sign, cross multiply and
divide the corresponding ratio to find the unknown quantity x.

100 𝑥
=
1 1440
1𝑥 = 100 ⋅ 1440
𝑥 = 144,000
(5) Instead of converting 1440 hectoliters to 144,000 liters, and then
5.2
dividing by 5.2, we can instead convert 5.2 liters to = .052
100
1440
hectoliters, and then divide .052
≈ 27,692.30769, giving the same
answer of 27,692, choice (C).
44. A psychologist wanted to determine if there is an association
between diet and stress levels for the population of middle aged
women in New York. He surveyed a random sample of 1500
middle aged female New Yorkers and found substantial
evidence of a positive association between diet and stress levels.
Which of the following conclusions is well supported by the
data?
(A) A dietary change causes an increase in stress levels for
middle aged women from New York.
(B) An increase in stress levels causes middle aged women
from New York to change their diets.
(C) There is a positive association between diet and stress
levels for middle aged women in New York.
(D) There is a positive association between diet and stress
levels for middle aged women in the world.

58
[Link]

* A relationship in the given data should only be generalized to the


population that the sample was drawn from. In this case, that is middle
aged women in New York. So we can eliminate choice (D).
Furthermore, choices (A) and (B) involve a cause-effect relationship.
This type of relationship can be established only when the participants
surveyed are randomly assigned to groups that are treated differently. In
this case all the participants are surveyed the same way. So we can
eliminate choices (A) and (B).
The answer is therefore choice (C).

45. A small puddle is monitored by scientists for the number of


paramecia present. The scientists are interested in two distinct
species, let’s call them “species 𝐴” and “species 𝐵.” At time
𝑡 = 0, the scientists measure and estimate the amount of species
𝐴 and species 𝐵 present in the puddle. They then proceed to
measure and record the number of each species of paramecium
present every hour for 12 days. The data for each species were
then fit by a smooth curve, as shown in the graph above. Which
of the following is a correct statement about the data above?
(A) At time 𝑡 = 0, the number of species 𝐵 present is 150%
greater than the number of species 𝐴 present.
(B) At time 𝑡 = 0, the number of species 𝐴 present is 75%
less than the number of species 𝐵 present.
(C) For the first 3 days, the average growth rate of species 𝐵
is higher than the average growth rate of species 𝐴.
(D) The growth rate of both species 𝐴 and species 𝐵
decreases for the last 8 days.

59
[Link]

* The last 8 days correspond to times 𝑡 = 4 through 𝑡 = 12. During this


time, the growth rate of both species is decreasing. So the answer is
choice (D).
Notes: (1) 300 = 100 + 2 ⋅ 100, and therefore 300 is 200% greater than
100. This eliminates choice (A).
(2) We can also use the percent change formula
𝐶ℎ𝑎𝑛𝑔𝑒
𝑃𝑒𝑟𝑐𝑒𝑛𝑡 𝐶ℎ𝑎𝑛𝑔𝑒 = × 100
𝑂𝑟𝑖𝑔𝑖𝑛𝑎𝑙
Here the Original value is 100 and the Change is 300 – 100 = 200. It
200
follows that 𝑃𝑒𝑟𝑐𝑒𝑛𝑡 𝐶ℎ𝑎𝑛𝑔𝑒 = 100 × 100 = 200%.

(3) To eliminate choice (B) we can use the percent change formula again
with Original value 300 and Change 300 – 100 = 200:
200 200 2
𝑃𝑒𝑟𝑐𝑒𝑛𝑡 𝐶ℎ𝑎𝑛𝑔𝑒 = 300 × 100 = 3
= 66 3 %.

(4) We can compute the average growth rate over the interval from 𝑡 = 𝑎
to 𝑡 = 𝑏, by computing the slope of the line passing through the points
𝑓(𝑏)−𝑓(𝑎)
(𝑎, 𝑓(𝑎)) and (𝑏, 𝑓(𝑏)). That is, we would compute 𝑚 = 𝑏−𝑎
.

For example, over the first 3 days, the average growth rate of species 𝐴 is
400−100 300
approximately = = 100 paramecia per day. The two points
3−0 3
we used here were (0,100) and (3,400).
Similarly, over the first 3 days, the average growth rate of species 𝐵 is
350−300 50 2
approximately 3−0 = 3 = 16 3 paramecia per day. The two points
we used here were (0,300) and (3,350).
This eliminates choice (C).
(5) It should be noted that we do not actually need to compute the growth
rates to determine which growth rate is higher. We can simply look at the
“steepness” of the two curves. An easy way to do this is to draw a
“tangent line” to each curve at the point where we wish to examine the
growth rate. Here is an example of such an analysis at 𝑡 = 2:

60
[Link]

Notice that the tangent line for species 𝐴 rises faster than the tangent line
for species 𝐵. This shows that species 𝐴 is growing faster than species 𝐵
at time 𝑡 = 2.
(6) We can use a similar analysis as we did in note 5 to see that the
growth rate is decreasing for each species between times 𝑡 = 4 and
𝑡 = 12. Here as an example of such an analysis for Species 𝐵:

Notice that the tangent line at time 𝑡 = 4 is steeper then the tangent line
at time 𝑡 = 8. This suggests that the growth rate of species 𝐵 is
decreasing from 𝑡 = 4 to 𝑡 = 12.

61
[Link]

46. * A survey was conducted among a randomly chosen sample of


250 single men and 250 single women about whether they
owned any dogs or cats. The table below displays a summary of
the survey results.

Dogs Cats
Both Neither Total
Only Only
Men 92 27 5 126 250
Women 75 43 34 98 250
Total 167 70 39 224 500

According to the table, which of the following statements is


true?
(A) The number of men that reported they had any pets is
greater than the number of people that reported they had
dogs only.
(B) The number of people that reported they had any pets is
less than twice the number of women that reported they
had dogs only.
(C) The percentage of men that reported they had dogs only
is less than the percentage of people that reported they
had cats only.
(D) The percentage of women that reported they had any pets
is greater than the percentage of people that reported they
did not have any pets.
* Solution by plugging in answer choices: Let’s start with choice (C).
The percentage of men that reported they had dogs only is
92
⋅ 100 = 36.8%. The percentage of people that reported they had cats
250
70
only is 500 ⋅ 100 = 14%. Since 36.8 > 14, we can eliminate choice (C).

Let’s try (D) next. The percentage of women that reported they had pets
75+43+34
is ⋅ 100 = 60.8%. The percentage of people that reported they
250
224
did not have pets is 500 ⋅ 100 = 44.8%. Since 60.8 > 44.8, the answer is
choice (D).

62
[Link]

Notes: (1) Although there is no advantage to starting with choice (C) in


this problem, there is sometimes an advantage to doing so. Therefore if
there is no specific reason not to, always begin with choice (C) as your
first guess (starting with choice (B) would be okay too).
(2) To compute a percentage, use the simple formula
𝑃𝑎𝑟𝑡
𝑃𝑒𝑟𝑐𝑒𝑛𝑡𝑎𝑔𝑒 = × 100
𝑊ℎ𝑜𝑙𝑒
For example, from the table we see that 92 men reported that they had
dogs only, and a total of 250 men were surveyed. It follows that the
percentage of men that reported they had dogs only is
92
250
⋅ 100 = 36.8%.

(3) There are three columns that represent having a pet – the columns
labeled “Dogs Only,” “Cats Only,” and “Both.”
So when computing the percentage of women that reported they had
pets, the numerator of the fraction is 75 + 43 + 34.
(4) For completeness let’s check (A) and (B).
For choice (A), the number of men that reported they had any pets is
92 + 27 + 5 = 124, and the number of people that reported they had
dogs only is 167. Since 124 < 167, we can eliminate choice (A).
For choice (B), the number of people that reported they had pets is
167 + 70 + 39 = 276, and the number of women that reported they had
dogs only is 75. Since 276 > 2 ⋅ 75 = 150, we can eliminate choice (B).
(5) Another way to compute the number of men that reported they had
pets is to subtract the number of men that reported they did not have pets
from the total number of men: 250 − 126 = 124.
Similarly, the number of women that reported they had pets is
250 − 98 = 152, and the number of people that reported they had pets is
500 − 224 = 276.

63
[Link]

47. The average (arithmetic mean) of three numbers is 57. If one of


the numbers is 16, what is the sum of the other two?
* We change the average to a sum using the formula
Sum = Average ⋅ Number
We are averaging 3 numbers so that the Number is 3. The Average is
given to be 57. Therefore the Sum of the 3 numbers is 57 ⋅ 3 = 171. Since
one of the numbers is 16, the sum of the other two is 171 – 16 = 155.
Note: The above formula comes from eliminating the denominator in the
definition of average:
Sum
Average =
Number
A complete algebraic solution for the advanced student: This method
is not recommended for the SAT, but it is included for completeness. Let
𝑥+𝑦+16
𝑥, 𝑦, and 16 be the three numbers. Then 3
= 57. Multiplying each
side of this equation by 3 yields 𝑥 + 𝑦 + 16 = 171. Finally, we subtract
16 from both sides to get 𝑥 + 𝑦 = 𝟏𝟓𝟓.
48. To decrease the mean of 5 numbers by 3, by how much would
the sum of the 5 numbers have to decrease?
Solution by picking numbers: Let’s pick some numbers. The numbers
10, 10, 10, 10, and 10 have a mean of 10 and a sum of 50. The numbers
7, 7, 7, 7, and 7 have a mean of 7 and a sum of 35. So to decrease the
mean by 3 we had to decrease the sum by 15.
* Quick solution: One way to decrease the mean of a list of numbers by
3 is to decrease each number in the list by 3. Since there are 5 numbers,
the sum must be decreased by 3(5) = 15.

64
[Link]

LEVEL 3: HEART OF ALGEBRA


49. The daily cost for a publishing company to produce 𝑥 books is
𝐶(𝑥) = 4𝑥 + 800. The company sells each book for $36. Let
𝑃(𝑥) = 𝑅(𝑥) − 𝐶(𝑥) where 𝑅(𝑥) is the total income that the
company gets for selling 𝑥 books. The company takes a loss for
the day if 𝑃(𝑥) < 0. Which of the following inequalities gives
all possible integer values of 𝑥 that guarantee that the company
will not take a loss on a given day?
(A) 𝑥 > 24
(B) 𝑥 < 24
(C) 𝑥 > 144
(D) 𝑥 < 144
* Algebraic solution: First note that 𝑅(𝑥) = 36𝑥, so that
𝑃(𝑥) = 36𝑥 − (4𝑥 + 800) = 36𝑥 − 4𝑥 − 800 = 32𝑥 − 800.
The company will not take a loss if 𝑃(𝑥) ≥ 0. So we solve the inequality
32𝑥 − 800 ≥ 0 for 𝑥. Adding 800 to each side of this inequality yields
800
32𝑥 ≥ 800, so 𝑥 ≥ 32 = 25.

Since we are only looking at integer values for 𝑥, 𝑥 ≥ 25 is equivalent to


𝑥 > 24, choice (A).
Notes: (1) Income that a company receives is called revenue. A revenue
function 𝑅(𝑥) gives the amount of income the company receives for
selling 𝑥 items.
If the company receives a fixed amount of 𝑑 dollars per item, then the
revenue function is 𝑅(𝑥) = 𝑑𝑥.
In this problem, the company receives 36 dollars per item, and so
𝑅(𝑥) = 36𝑥.
If the company sells 1 book, then the revenue is 𝑅(1) = 36 ⋅ 1 = 36
dollars.
If the company sells 2 books, then the revenue is 𝑅(2) = 36 ⋅ 2 = 72
dollars.
And so on…

65
[Link]

(2) A company’s profit is revenue minus cost. A profit function 𝑃(𝑥)


gives the amount of profit (or loss) the company makes (or loses) if it
sells 𝑥 items.
𝑃(𝑥) = 𝑅(𝑥) − 𝐶(𝑥)
If revenue is greater than cost, then 𝑃(𝑥) > 0, and the company makes a
profit of 𝑃(𝑥) dollars.
If revenue is less than cost, then 𝑃(𝑥) < 0, and the company takes a loss
of |𝑃(𝑥)| dollars.
The break-even point occurs when 𝑃(𝑥) = 0.
In this problem the break-even point occurs when 25 books are sold.
(3) When simplifying 𝑃(𝑥), make sure you are using the distributive
property correctly. A common mistake would be to write
36𝑥 − (4𝑥 + 800) = 36𝑥 − 4𝑥 + 800.
This error comes from forgetting to distribute the minus sign.
If you frequently fall into this trap, it might help to first rewrite the
expression −(4𝑥 + 800) as −1(4𝑥 + 800). So we have
−(4𝑥 + 800) = −1(4𝑥 + 800) = −4𝑥 − 800.
(4) In general, 𝑥 ≥ 25 and 𝑥 > 24 are not equivalent expressions. For
example, if 𝑥 is allowed to range over all real numbers, then 𝑥 = 24.5
satisfies the second inequality, but not the first.
In this question however we are restricting our 𝑥-values to positive
integers. In this case, the two inequalities are equivalent.
(5) 𝑃(𝑥) ≥ 0 is equivalent to 𝑅(𝑥) ≥ 𝐶(𝑥). So we could also solve the
inequality 36𝑥 ≥ 4𝑥 + 800. This method of solution avoids having to
use the distributive property. We simply subtract 4𝑥 from each side of the
inequality to get 32𝑥 ≥ 800, and then divide by 32 to get 𝑥 ≥ 25.
Solution by picking a number: Using the answer choices as a guide,
let’s choose a value for 𝑥, say 𝑥 = 50.
Then 𝑅(𝑥) = 𝑅(50) = 36 ⋅ 50 = 1800, and
𝐶(𝑥) = 𝐶(50) = 4 ⋅ 50 + 800 = 200 + 800 = 1000.
So 𝑃(𝑥) = 𝑅(𝑥) − 𝐶(𝑥) = 1800 − 1000 = 800.

66
[Link]

So, for 𝑥 = 50, the company does not take a loss. Therefore 𝑥 = 50
should be a solution to the inequality. So we can eliminate choices (B)
and (C).
We still need to decide if the answer is choice (A) or (D). So let’s pick
another number, say 𝑥 = 20.
Then R(𝑥) = 𝑅(20) = 36 ⋅ 20 = 720, and
𝐶(𝑥) = 𝐶(20) = 4 ⋅ 20 + 800 = 80 + 800 = 880.
So 𝑃(𝑥) = 𝑅(𝑥) − 𝐶(𝑥) = 720 − 880 = −160.
So, for 𝑥 = 20, the company does take a loss. Therefore 𝑥 = 20 should
not be a solution to the inequality. So we can eliminate choice (D), and
therefore the answer is choice (A).
2
50. If 𝑦 = 𝑘 −3 , where 𝑘 > 𝑦 > 0, which of the following equations
gives 𝑘 in terms of 𝑦 ?
3
(A) 𝑘 = − √𝑦 2
(B) 𝑘 = −√𝑦 3
1
(C) 𝑘 = 3
√𝑦 2
1
(D) 𝑘 =
√𝑦 3
2
Algebraic solution: We raise each side of the equation 𝑦 = 𝑘 −3 to the
3
3 2 − −2 −3
3 − − 2
−2 power to get 𝑦 2 = (𝑘 ) 3 = 𝑘 ( 3 )( 2 ) = 𝑘 1 = 𝑘. It follows that
3
1 1
𝑘 = 𝑦 −2 = 3 = , choice (D).
𝑦2 √𝑦 3

Notes: (1) For the laws of exponents used here, see the table below.
3
2 − −2 −3
− 2
For example, we used the law (𝑥 𝑎 )𝑏 =𝑥 𝑎𝑏
to get (𝑘 ) 3 = 𝑘 ( 3 )( 2 ).

(2) To get rid of an exponent, we raise to the reciprocal exponent. The


𝑏
𝑎 𝑎 𝑎 𝑏
𝑎 𝑏
reciprocal of is
𝑏 𝑎
. Observe that (𝑥 ) = 𝑥 (𝑏)(𝑎) = 𝑥 1 = 𝑥.
𝑏

3
2 − −2 −3
3 2 − 2
The reciprocal of −2 is − 3. So we have (𝑘 ) 3 = 𝑘 ( 3 )( 2 ) = 𝑘 1 = 𝑘.

67
[Link]

(3) We can change a negative exponent to a positive exponent by taking a


3
1
reciprocal. In this problem, we have 𝑦 −2 = 3 .
𝑦2
3 3
A common mistake is to write 𝑦 −2 = −𝑦 2 . This is completely false. The
expression on the left hand side is positive, whereas the expression on the
right is negative.
𝑏
(4) In the expression 𝑎 𝑐 , 𝑎 is the base, 𝑏 is the power, and 𝑐 is the root.
𝑏
𝑐
So we can write 𝑎 𝑐 = √𝑎𝑏 .
3
For example, 𝑦 2 = √𝑦 3 .
3
1 1
It follows that 𝑦 −2 = 3 = .
𝑦2 √𝑦 3

Laws of Exponents: For those students that have forgotten, here is a


brief review of the laws of exponents needed here:
Law Example
x0 = 1 30 = 1
x1 = x 91 = 9
xaxb = xa+b x3x5 = x8
xa/xb = xa-b x11/x4 = x7
(xa)b = xab (x5)3 = x15
(xy)a = xaya (xy)4 = x4y4
(x/y)a = xa/ya (x/y)6 = x6/y6
x-1 = 1/x 3-1 = 1/3
x-a = 1/xa 9-2 = 1/81
3
x1/n = 𝒏√𝒙 x1/3 = √𝑥
𝒏 𝒏 𝒎 9
xm/n = √𝒙𝒎 =( √𝒙) x9/2 =√𝑥 9 =(√𝑥)

Solution by picking a number: Let’s choose a value for 𝑘, say 𝑘 = 8.


2
1 1 1 1 1
Then 𝑦 = 8−3 = 2 = 2 = = . So, when we substitute in for 𝑦,
3
( √8) 22 4 4
83
we should get 𝑘 = 𝟖. Put a nice big, dark circle around the number 8 so
you can find it easily later.
1
We now substitute 𝑦 = 4 into each answer choice:

68
[Link]

3 1 2
(A) 𝑘 = − √(4) < 0

1 3
(B) 𝑘 = −√(4) < 0
1 1 3 3
(C) 𝑘 = 3 2
= 3 1
= √16 < √27 = 3 < 8
√(1) √
4 16
1 1
(D) 𝑘 = 3
= 1
= √64 = 8
√(1) √
4 64

Since choices (A), (B), and (C) did not come out correct, the answer is
choice (D).
Notes: (1) Since we are plugging 𝑦 values into the answer choices, we
would normally pick a value for 𝑦. In this problem we chose a value for 𝑘
instead because it is much easier to choose 𝑘 and find 𝑦, than it is to do it
the other way around.
(2) Note that we picked the number 8 for 𝑘, as opposed to an integer like
2. This is because 8 is a perfect cube, and the denominator in the
exponent indicates that we will need to take a cube root.
(3) We can change a negative exponent to a positive exponent by taking a
2
1
reciprocal. In this problem, we have 8−3 = 2 .
83
2 2
A common mistake is to write 8−3 = −83 . This is completely false. The
number on the left hand side is positive, whereas the number on the right
is negative.
𝑏
(4) In the expression 𝑎 𝑐 , 𝑎 is the base, 𝑏 is the power, and 𝑐 is the root.
𝑏
𝑐
So we can write 𝑎 𝑐 = √𝑎𝑏 .
2
3
For example, 83 = √82 .
2
1 1
It follows that 8−3 = 2 = 3 .
83 √82

𝑐 𝑐 𝑏
In general, √𝑎𝑏 = ( √𝑎) . It is often easier to take the root first.
3 3 2
In this problem we have √82 = (√8) = 22 = 4.

69
[Link]

(5) There is no reason to continue evaluating answer choices once it is


clear that the choice will not yield the correct answer. For example, we
see right away that choices (A) and (B) will give negative answers. Since
we know our answer should be positive, we can eliminate them.
(6) If we are allowed to use a calculator for this problem, we could use it
to evaluate the given expression and compute the answer choices quickly.
2
For example, given that we chose 𝑘 = 8, we can compute 𝑦 = 𝑘 −3 , easily
by typing 8 ^ (-2/3) ENTER into our calculator. The display will show
.25.
We can then substitute .25 in for 𝑦 in each answer choice and eliminate
any answers that do not come out to 8.
(7) When using the strategy of picking numbers it is very important that
we check every answer choice. It is possible for more than one choice to
come out to the correct answer. We would then need to pick a new
number to try to eliminate all but one choice.
51. For all real numbers 𝑥 and 𝑦, |𝑥 − 𝑦| is equivalent to which of
the following?
(A) 𝑥+𝑦
(B) √𝑥 − 𝑦
(C) (𝑥 − 𝑦)2
(D) √(𝑥 − 𝑦)2
* Solution using the definition of absolute value: One definition of the
absolute value of x is |𝑥| = √𝑥 2 . So |𝑥 − 𝑦| = √(𝑥 − 𝑦)2 , choice (D).
Note: Here we have simply replaced x by x – y on both sides of the
equation |𝑥| = √𝑥 2 .
Solution by picking numbers: Let’s choose values for x and y, let’s say
x = 2 and y = 5. Then |x – y| = |2 – 5| = |–3| = 3.
Put a nice big dark circle around 3 so you can find it easily later. We now
substitute x = 2 and y = 5 into each answer choice:
(A) 7
(B) √−3
(C) (−3)2 = 9
(D) √(−3)2 = √9 = 3

70
[Link]

Since A, B and C each came out incorrect, we can eliminate them. So the
answer is choice D.
1 1
52. If 5 𝑥 + 7 𝑦 = 3, what is the value of 7𝑥 + 5𝑦?

* Solution by trying a simple operation: We multiply each side of the


equation by 35 to get 7𝑥 + 5𝑦 = 𝟏𝟎𝟓.
Remarks: (1) When we multiply the left hand side by 35, we have to
multiply each term by 35.
1 1
35 ( 𝑥) = 7𝑥 35 ( 𝑦) = 5𝑦
5 7

(2) 35 is a natural “first guess” because it is the least common multiple


of 5 and 7 (it is actually also the product of 5 and 7, but I mention the
least common multiple here because it is usually the better guess when
the two are not equal).
(3) Since 5 and 7 are prime, and therefore have no prime factors in
common, the least common multiple of 5 and 7 is the same as the product
of 5 and 7.
Solution using a “standard substitution trick”: We attempt to change
the left hand side of the given equation by putting in what we need. Next
to the 𝑥 we need a 7 and next to the 𝑦 we need a 5.
1 1 1 1
⋅ (7𝑥) + ⋅ (5𝑦) = 3
5 7 7 5
1 1
(7𝑥) + (5𝑦) = 3
35 35
1
(7𝑥 + 5𝑦) = 3
35
Note that the expression we are looking for is now in parentheses. So we
simply multiply each side of the equation by 35 to get
7𝑥 + 5𝑦 = 3 ⋅ 35 = 𝟏𝟎𝟓
Remarks: (1) Note that we undo multiplying 𝑥 by 7 by also multiplying
1 1
by 7. Similarly we undo multiplying 𝑦 by 5 by also multiplying by 5.
1 1
(2) We group the 7 with the 5 and leave 7𝑥 by itself. We can do this
because multiplication of real numbers is associative. Similarly for 𝑦.

71
[Link]

53. Gary takes a New York City cab 6 miles to work and must pay
$17.50 for the cab ride. After work, Gary takes another New
York City cab 10 miles to visit his family and must pay $27.50.
During both of these rides, Gary was charged a “drop fee” (an
initial charge when the cab’s meter is activated) of 𝑑 dollars,
1
plus an additional 𝑚 dollars for each of a mile travelled. What
5
is the value of md ?
* We can model the information here with a linear equation 𝐶 = 𝑚𝑥 + 𝑑,
1
where 𝑚 is the cost of the cab ride per mile, 𝑑 is the drop fee, and 𝐶 is
5
𝑥
the total cost for a cab that travels miles.
5

We are essentially given two points on the line, but we should convert
1
miles to 5 miles as we write down the points. So 6 miles is equivalent to
1
an 𝑥-value of 6 ⋅ 5 = 30 units of 5 miles, and 10 miles is equivalent to an
1
𝑥-value of 10 ⋅ 5 = 50 units of 5 miles. It follows that the two points
given are (30,17.50) and (50,27.50).
We can now find the slope of the line passing through these two points:
27.50−17.50 10
𝑚 = 50−30 = 20 = .50.

Using the point (30,17.50) and the slope 𝑚 = .5 we can now write an
equation of the line in point-slope form: 𝐶 − 17.50 = .5(𝑥 − 30).
Distributing the .5 on the right hand side of the equation gives us
𝐶 − 17.50 = .5𝑥 − 15, and then adding 17.50 to each side of this last
equation gives us 𝐶 = .5𝑥 + 2.50.
It follows that 𝑚 = .5 and 𝑑 = 2.50. So 𝑚𝑑 = 𝟏. 𝟐𝟓 or 𝟓/𝟒.
Notes: (1) The slope of a line passing through the points (𝑥1 , 𝑦1 ) and
(𝑥2 , 𝑦2 ) is
𝑟𝑖𝑠𝑒 𝑦 −𝑦
Slope = 𝑚 = = 𝑥2−𝑥1
𝑟𝑢𝑛 2 1

In this problem the two points are (30,17.50) and (50,27.50).


(2) Using the points (6,17.50) and (10,27.50) would have led to a wrong
answer (unless a conversion was done at the end) because 𝑥 is being
1
measured in 5 miles and not miles.

72
[Link]

(3) The slope-intercept form of an equation of a line is 𝒚 = 𝒎𝒙 + 𝒃


where 𝑚 is the slope of the line and 𝑏 is the 𝑦-coordinate of the
𝑦-intercept, i.e. the point (0, 𝑏) is on the line. Note that this point lies on
the 𝑦-axis.
In this problem we used the letter 𝐶 (for cost) instead of 𝑦, and 𝑑 (for
drop) instead of 𝑏.
It turned out that 𝑚 = .5 and 𝑑 = 2.50
(4) The point-slope form of an equation of a line is
𝒚 − 𝒚𝟎 = 𝒎(𝒙 − 𝒙𝟎 )
where 𝒎 is the slope of the line and (𝒙𝟎, 𝒚𝟎 ) is any point on the line.

In this problem 𝑚 = .5 and (𝑥0, 𝑦0 ) = (30,17.50).


(5) We could have used the point (50,27.50) instead when writing an
equation of the line in point-slope form. In this case we would get
𝐶 − 27.50 = .5(𝑥 − 50). I leave it to the reader to show that when you
solve this equation for 𝐶 you get the same slope-intercept form as before.
(6) As an alternative to using point-slope form, after finding 𝑚, we could
plug one of the points into the slope-intercept form of the equation and
solve for 𝑑 as follows:
𝐶 = .5𝑥 + 𝑑
17.50 = .5(30) + 𝑑
17.50 = 15 + 𝑑
𝑑 = 17.50 − 15 = 2.5
We now have 𝑚 = .5 and 𝑑 = 2.5, so that 𝑚𝑑 = 1.25 or 5/4.
(7) If we were to take our points as (6,17.50) and (10,27.50), we would
27.50−17.50 10
get a slope of 𝑚 = = = 2.50. This is the cost per mile. We
10−6 4
1 2.50
can change this to the cost per 5
mile by dividing by 5 to get 5
= .5 as
before.
(8) Technical note: In reality a new York city cab also charges 𝑚 dollars
for each 60 seconds of “wait time” when the cab is stuck in traffic. The
question given here avoided this extra complication.
54. If 2𝑥 + 3𝑦 = 5, 2𝑦 + 𝑧 = 3, and 𝑥 + 5𝑦 + 𝑧 = 3, then 𝑥 =

73
[Link]

* Solution by performing simple operations: We add the first two


equations and subtract the second equation to get 𝑥 = 5 + 3 – 3 = 5.
Computations in detail: We add the first two equations:
2𝑥 + 3𝑦 =5
2𝑦 + 𝑧 = 3
2𝑥 + 5𝑦 + 𝑧 = 8
We then subtract the third equation from this result:
2𝑥 + 5𝑦 + 𝑧 = 8
𝑥 + 5𝑦 + 𝑧 = 3
𝑥 =5
Solution using Gauss-Jordan reduction: We will need to use our
graphing calculator for this solution.
Let’s write the system of equations vertically:
2𝑥 + 3𝑦 =5
2𝑦 + 𝑧 = 3
𝑥 + 5𝑦 + 𝑧 = 3
The augmented matrix for this system is
2 3 0 5
[0 2 1 3]
1 5 1 3
Push the MATRIX button, scroll over to EDIT and then select [A] (or
press 1). We will be inputting a 3 × 4 matrix, so press 3 ENTER 4
ENTER. Then enter the numbers 2, 3, 0 and 5 for the first row, 0, 2, 1 and
3 for the second row, and 1, 5, 1 and 3 for the third row.
Now push the QUIT button (2ND MODE) to get a blank screen. Press
MATRIX again. This time scroll over to MATH and select rref( (or press
B). Then press MATRIX again and select [A] (or press 1) and press
ENTER.
The display will show the following.
[ [1 0 0 5 ]
[0 1 0 − 1.67 ]
[0 0 1 6.33]]
The first line is interpreted as x = 5.

74
[Link]

Notes: (1) We began by creating the augmented matrix for the system
of equations. This is simply an array of numbers which contains the
coefficients of the variables together with the right hand sides of the
equations.
(2) We then put the matrix into reduced row echelon form (rref). In this
form we can read off the solution to the original system of equations.
Warning: Be careful to use the rref( button (2 r’s), and not the ref( button
(which has only one r).

LEVEL 3: GEOMETRY AND TRIG


55. The area of a rectangular garden is 448 square feet. A statue
with a rectangular base that is twice as long as it is wide is
placed in the center of the garden so that the garden extends 6
feet beyond the base of the statue on the top and bottom and 4
feet beyond the base of the statue on the left and right sides.
How many feet wide is the base of the statue?
(A) 6
(B) 8
(C) 14
(D) 20
Let’s draw a picture.

In this picture the larger rectangle represents the garden and the smaller
rectangle inside represents the statue.

75
[Link]

If we let x be the width of the statue, then the length is 2x (since the statue
is twice as long as it is wide). Since the garden extends 6 feet beyond the
statue on top and bottom, the length of the garden is 2x + 6 + 6 = 2x + 12
feet. Since the garden extends 4 feet beyond the statue on the left and the
right, the width of the garden is x + 4 + 4 = x + 8 feet.
It follows that the area of the garden is (x + 8)(2x + 12). We need to find
the value of x that makes this expression 448. Here are two ways we can
do this:
* Method 1 – Plugging in answer choices: Let’s start with choice (C)
and guess that x = 14. It follows that
(x + 8)(2x + 12) = (14 + 8)(2 ∙ 14 + 12) = 22 ∙ 40 = 880.
This is too big so we can eliminate choices (C) and (D).
Let’s try choice (B) next and guess that x = 8. It follows that
(x + 8)(2x + 12) = (8 + 8)(2 ∙ 8 + 12) = 16 ∙ 28 = 448.
This is correct. So the answer is choice (B).
Method 2 – Solving a quadratic equation: We need to solve the
equation (x + 8)(2x + 12) = 448. Multiplying out the left hand side gives
2x2 + 12x + 16x + 96 = 448. Dividing by 2 gives x2 + 6x + 8x + 48 = 224.
We now combine like terms on the left to get x2 + 14x + 48 = 224.
Subtract 224 from each side and we have x2 + 14x – 176 = 0. The left
hand side can be factored to get (x – 8)(x + 22) = 0. So we have x – 8 = 0
or x + 22 = 0. Therefore x = 8 or x = –22. We reject the negative solution
to get x = 8, choice (B).
Notes: (1) As you can see from method 2, a complete algebraic solution
is quite tedious here. Plugging in is a better choice for most students.
(2) We can also plug in answer choices right at the beginning of the
problem before doing any algebra at all.
(3) The equation x2 + 14x – 176 = 0 can be solved several different ways.
We did it by factoring above, but completing the square and using the
quadratic formula are two other alternatives.

76
[Link]

56. The figure above shows a right triangle whose hypotenuse is 4


feet long. How many feet long is the shorter leg of this triangle?
(A) 2
(B) 8
(C) 2√3
2√3
(D) 3

* Solution using a 30, 60, 90 right triangle: The following figure is


given at the beginning of each math section on the SAT.

Comparing the figure given in the problem to this figure, we see that
2𝑥 = 4 and so 𝑥 = 2, choice (A).
Note: For the SAT, the following two special triangles are given at the
beginning of each math section:

Some students get a bit confused because there are variables in these
pictures. But the pictures become simplified if we substitute a 1 in for the
variables. Then the sides of the 30, 60, 90 triangle are 1, 2 and √3 and the
sides of the 45, 45, 90 triangle are 1, 1 and √2. The variable just tells us
that if we multiply one of these sides by a number, then we have to
multiply the other two sides by the same number. For example, instead of
1, 1 and √2, we can have 3, 3 and 3√2 (here s = 3), or √2, √2, and 2
(here s = √2).

77
[Link]

* Trigonometric solution: The shorter leg of the triangle is adjacent to


ADJ ADJ
the 60° angle. So we will use cosine. We have cos 60° = HYP = 4 . So
1
ADJ = 4cos 60° = 4(2) = 2, choice (A).
ADJ
Remarks: (1) If you do not see why we have cos 60° = HYP
, review the
basic trigonometry given after the solution to problem 9.
ADJ
(2) To get from cos 60° = 4 to ADJ = 4cos 60°, we simply multiply
each side of the first equation by 4.
For those of you that like to cross multiply, the original equation can first
cos 60° ADJ
be rewritten as 1 = 4 .

(3) There are several ways to compute cos 60°. The easiest is to simply
put it into your calculator (if you are allowed to use your calculator for
the question). The output will be .5.
(4) If you are using your calculator, make sure it is in degree mode.
Otherwise you will get an incorrect answer.
If you are using a TI-84 (or equivalent) calculator press MODE and on
the third line make sure that DEGREE is highlighted. If it is not, scroll
down and select it. If possible do not alter this setting until you are
finished taking your test.
(5) We can also compute cos 60° using the figure from the first solution
ADJ 𝑥 1
(given on the SAT). For this problem, we have cos 60° = HYP = 2𝑥 = 2.
57. In ∆𝐷𝑂𝐺, the measure of ∠𝐷 is 60° and the measure of ∠𝑂 is
30°. If ̅̅̅̅
𝐷𝑂 is 8 units long, what is the area, in square units, of
∆𝐷𝑂𝐺 ?
(A) 4
(B) 8
(C) 8√2
(D) 8√3
* Solution using a 30, 60, 90 right triangle: Let’s draw two pictures.

78
[Link]

The picture on the left is what is given in the problem. Comparing this to
the picture on the right we see that x = 4 and √3𝑥 = 4√3. So the area of
1
the triangle is 2(4)(4√3) = 8√3, choice (D).

Note: See problem 56 for more information on 30, 60, 90 triangles.


𝐷𝐺
Trigonometric solution: We have sin 30° = 8
. So DG = 8sin 30°.
𝑂𝐺
Similarly, cos 30° = 8
. So OG = 8cos 30°. So the area of the triangle is
1 1 1 √3 1
(OG)(DG) = (8cos 30°)(8sin 30°) = (8 ⋅ ) (8 ⋅ ) = 8√3, choice
2 2 2 2 2
(D).
Notes: (1) We can evaluate cos 30° and sin 30° by using the 30, 60, 90
triangle shown in the figure above on the right.
ADJ √3𝑥 √3 OPP 𝑥 1
cos 30° = HYP = 2𝑥
= 2
and sin 30° = HYP = 2𝑥 = 2

(2) If we are allowed to use a calculator, then we can compute


1 1
2
(OG)(DG) = 2(8cos 30°)(8sin 30°) ≈ 13.8564.

We can then plug the answer choices into the calculator to see that

8√3 ≈ 13.8564
So once again we get that the answer is choice (D).
Remark: Make sure that your calculator is in degree mode. Otherwise
you will get the wrong answer (see problem 56).
58. In the (𝑥, 𝑦) coordinate plane, what is the radius of the circle
having the points (2,−4) and (−4,4) as endpoints of a diameter?
* We can find the length of the diameter of the circle by using the
distance formula. We have

79
[Link]

2
𝑑 = √(−4 − 2)2 + (4 − (−4)) = √(−6)2 + 82 = √36 + 64 = √100 = 10

10
It follows that the radius is 2
= 5.

Notes: (1) The distance between the two points (ℎ, 𝑘) and (𝑎, 𝑏) is

𝑑 = √(𝑎 − ℎ)2 + (𝑏 − 𝑘)2 or equivalently 𝑑2 = (𝑎 − ℎ)2 + (𝑏 − 𝑘)2


This formula is called the distance formula.
1 1
(2) The radius of a circle is 2 the diameter, or r = 2d.

(3) Here is a picture of the circle, the two points, and the diameter.

𝑥
59. * If 0 < 𝑥 < 90° and sin 𝑥 = 0.525, what is the value of cos(3)

* Calculator solution:
𝑥 31.67
𝑥 = sin−1 0.525 ≈ 31.67. So cos(3) ≈ cos ( 3
) ≈ .983.

60. The dimensions of a triangular block are shown above. What is


the value of cos 𝑥 ?
* Since the triangle is isosceles, the median and altitude from the vertex
are the same.

80
[Link]

ADJ 9 𝟑
It follows that cos 𝑥 = = = or 1.5.
HYP 6 𝟐

Notes: (1) A triangle is isosceles if it has two sides of equal length.


Equivalently, an isosceles triangle has two angles of equal measure.
(2) An altitude of a triangle is perpendicular to the base. A median of a
triangle splits the base into two equal parts. An angle bisector of a
triangle splits an angle into two equal parts. In an isosceles triangle, the
altitude, median, and angle bisector are all equal (when you choose the
base that is not one of the equal sides).

LEVEL 3: PASSPORT TO ADVANCED MATH


61. If 𝑏 = 5𝑎3 − 2𝑎 + 7 and 𝑐 = 2𝑎2 + 𝑎 + 3, what is 3𝑐 − 𝑏 in
terms of 𝑎 ?
(A) −5𝑎3 + 6𝑎2 + 𝑎 + 16
(B) −5𝑎3 + 6𝑎2 + 3𝑎 − 4
(C) −5𝑎3 + 6𝑎2 + 5𝑎 + 2
(D) 𝑎2 + 5𝑎 + 2
* Algebraic solution: 3𝑐 − 𝑏 = 3(2𝑎2 + 𝑎 + 3) − (5𝑎3 − 2𝑎 + 7)
= 6𝑎2 + 3𝑎 + 9 − 5𝑎3 + 2𝑎 − 7 = −5𝑎3 + 6𝑎2 + 5𝑎 + 2.
This is choice (C).
Notes: (1) The distributive property needs to be used twice here. Make
sure you are using it correctly.
The first time it needs to be used is when multiplying 𝑐 by 3:
3𝑐 = 3(2𝑎2 + 𝑎 + 3) = 6𝑎2 + 3𝑎 + 9.
A common mistake is to write 3(2𝑎2 + 𝑎 + 3) = 6𝑎2 + 𝑎 + 3.

81
[Link]

(2) An even more common error is to forget to put the expression for 𝑏 in
parentheses. This flawed computation would look like this:
3𝑐 − 𝑏 = 3(2𝑎2 + 𝑎 + 3) − 5𝑎3 − 2𝑎 + 7
= 6𝑎2 + 3𝑎 + 9 − 5𝑎3 − 2𝑎 + 7 = −5𝑎3 + 6𝑎2 + 𝑎 + 16.
This error leads to choice (A), the wrong answer.
(3) Even when putting in the parentheses, many students mess this up.
Look at the following use of the distributive property carefully:
−(5𝑎3 − 2𝑎 + 7) = −5𝑎3 + 2𝑎 − 7
(4) Negating an expression is equivalent to multiplying that expression by
−1. This may help some students avoid the previous error.
More precisely, −(5𝑎3 − 2𝑎 + 7) is the same as −1(5𝑎3 − 2𝑎 + 7), so
that −(5𝑎3 − 2𝑎 + 7) = −1(5𝑎3 − 2𝑎 + 7) = −5𝑎3 + 2𝑎 − 7.
Solution by picking a number: Let’s choose a value for 𝑎, say 𝑎 = 2. It
follows that 𝑏 = 5𝑎3 − 2𝑎 + 7 = 5(2)3 − 2(2) + 7 = 40 − 4 + 7 = 43
and 𝑐 = 2𝑎2 + 𝑎 + 3 = 2(2)2 + 2 + 3 = 8 + 5 = 13.
So 3𝑐 − 𝑏 = 3 ⋅ 13 − 43 = 39 − 43 = −𝟒.
Put a nice big, dark circle around the number −4 so you can find it easily
later. We now substitute 𝑎 = 2 into each answer choice:
(A) −5𝑎3 + 6𝑎2 + 𝑎 + 16 = −5(2)3 + 6(2)2 + 2 + 16 = 2
(B) −5𝑎3 + 6𝑎2 + 3𝑎 − 4 = −5(2)3 + 6(2)2 + 3 ⋅ 2 − 4 = −14
(C) −5𝑎3 + 6𝑎2 + 5𝑎 + 2 = −5(2)3 + 6(2)2 + 5 ⋅ 2 + 2 = −4
(D) 𝑎2 + 5𝑎 + 2 = 22 + 5 ⋅ 2 + 2 = 16

Since choices (A), (B), and (D) did not come out correct, the answer is
choice (C).
Important note: (C) is not the correct answer simply because it came
out to −4. It is correct because all three of the other choices did not come
out correct.

82
[Link]

62. Jessica’s car gets 18 miles per gallon when the car travels at an
average speed of 35 miles per hour. Jessica begins a trip with 14
gallons of gas, and she travels at 35 miles per hour for the first 4
hours of her trip. Which of the following functions 𝑔 is the most
accurate model for the number of gallons of gas remaining in the
tank 𝑡 hours after the trip begins for 0 ≤ 𝑡 ≤ 4 ?
14−35𝑡
(A) 𝑔(𝑡) = 18
14−18𝑡
(B) 𝑔(𝑡) =
35
35𝑡
(C) 𝑔(𝑡) = 14 − 18
18
(D) 𝑔(𝑡) = 14 − 35𝑡

Solution by picking a number and estimation: Let’s choose a value for


𝑡, say 𝑡 = 2. In this case Jessica travels for 2 hours. Since she is
travelling 35 miles per hour, she has travelled 𝑑 = 𝑟𝑡 = 35 ⋅ 2 = 70
70 35
miles. Since she gets 18 miles per gallon, she has used 18 = 9 ≈ 4
gallons. So there is about 14 − 4 = 𝟏𝟎 gallons remaining in the tank.
Put a nice big, dark circle around the number 10 so you can find it easily
later.
We now substitute 𝑡 = 2 into each answer choice:
14−35⋅2 −56
(A) = <0
18 18
14−18⋅2 −22
(B) 35
= 35
<0
35⋅2 70
(C) 14 − 18
= 14 − 18 ≈ 14 − 4 = 10
18 18
(D) 14 − 35⋅2 = 14 − 70 > 13

We see that the only possible answer is choice (C).


Notes: (1) Note that we picked a number that was simple but not too
simple. The number 2 is usually a good choice if it is allowed.
(2) We used the formula “distance = rate × time” or 𝑑 = 𝑟𝑡.
In this problem the rate is 𝑟 = 35 miles/hour and we chose 𝑡 = 2 hours.

83
[Link]

36 35
(3) Since 9
= 4, we have that 9
is almost 4.

(4) Notice how we just estimated here. This worked out well because
none of the answer choices wound up being close to each other with our
chosen value for 𝑡. If two or more choices were close to 10, we would
have needed to be more careful.
(5) If we are allowed to use a calculator for this problem, we would get a
70
better decimal approximation for by dividing 70 by 18 in our
18
70
calculator to get 14 − ≈ 𝟏𝟎. 𝟏𝟏𝟏
18

We could then also use our calculator for each answer choice. For
35⋅2
example, 14 − 18 ≈ 𝟏𝟎. 𝟏𝟏𝟏.

(6) When using the strategy of picking numbers it is very important that
we check every answer choice. It is possible for more than one choice to
come out to the correct answer. We would then need to pick a new
number to try to eliminate all but one choice.
* Algebraic solution: When Jessica has travelled 𝑡 hours, her total
distance travelled is 𝑑 = 𝑟𝑡 = 35𝑡. Since she gets 18 miles per gallon,
𝑑 35𝑡 35𝑡
she has used 18 = 18 gallons, and therefore 14 − 18 gallons of gas are
remaining, choice (C).
Notes: (1) Notice how my algebraic solution mimics my solution of
picking numbers.
(2) The algebraic solution is confusing without the specific numbers to
guide you. Although picking numbers may take a little longer, in this case
it is worth choosing that method over the algebraic method to avoid
making a “careless” error.
63. What polynomial must be added to 𝑥 2 + 3𝑥 − 5 so that the sum
is 5𝑥 2 − 8 ?
(A) 4𝑥 2 − 5𝑥 + 6
(B) 4𝑥 2 − 3𝑥 − 3
(C) 5𝑥 2 − 3𝑥 − 3
(D) 5𝑥 2 + 3𝑥 + 6
Solution by picking a number: Let’s choose a value for x, say x = 2.

84
[Link]

Then we have x2 + 3x – 5 = (2)2 + 3(2) – 5 = 4 + 6 – 5 = 5 and we also


have 5x2 – 8 = 5(2)2 – 8 = 5(4) – 8 = 20 – 8 = 12. So we must add 7 to get
from 5 to 12 (indeed, 12 – 5 = 7).
Put a nice big dark circle around 7 so you can find it easier later. We now
substitute 2 for x into each answer choice:
(A) 4(2)2 – 5(2) + 6 = 12
(B) 4(2)2 – 3(2) – 3 = 7
(C) 5(2)2 – 3(2) – 3 = 11
(D) 5(2)2 + 3(2) + 6 = 32
Since (A), (C) and (D) each came out incorrect, the answer is choice (B).
Important note: (B) is not the correct answer simply because it is equal
to 7. It is correct because all three of the other choices are not 7. You
absolutely must check all four choices!
* Algebraic solution: We need to subtract (5x2 – 8) – (x2 + 3x – 5). We
first eliminate the parentheses by distributing the minus sign:
5x2 – 8 – x2 – 3x + 5
Finally, we combine like terms to get 4x2 – 3x – 3, choice (B).
Remark: Pay careful attention to the minus and plus signs in the solution
above. In particular, make sure you are distributing correctly.
64. If –7 and 5 are both zeros of the polynomial 𝑞(𝑥), then a factor
of 𝑞(𝑥) is
(A) 𝑥 2 − 35
(B) 𝑥 2 + 35
(C) 𝑥 2 + 2𝑥 + 35
(D) 𝑥 2 + 2𝑥 − 35
* Algebraic solution: (𝑥 + 7) and (𝑥 − 5) are both factors of 𝑞(𝑥).
Therefore so is (𝑥 + 7)(𝑥 − 5) = 𝑥 2 + 2𝑥 − 35, choice (D).
Note: There are several ways to multiply two binomials. One way
familiar to many students is by FOILing. If you are comfortable with the
method of FOILing you can use it here, but an even better way is to use
the same algorithm that you already know for multiplication of whole
numbers.

85
[Link]

x+7
x–5
–5x – 35
x2 + 7x + 0
x2 + 2x – 35
What we did here is mimic the procedure for ordinary multiplication. We
begin by multiplying –5 by 7 to get –35. We then multiply –5 by x to get
–5x. This is where the first row under the first line comes from.
Next we put 0 in as a placeholder on the next line. We then multiply x by
7 to get 7x. And then we multiply x by x to get x2. This is where the
second row under the first line comes from.
Now we add the two rows to get x2 + 2x – 35.
Solution by plugging in answer choices: We are looking for the
expression that gives 0 when we substitute in –7 and 5 for x.
Starting with choice (C) we have 52 + 2(5) + 35 = 70. So we eliminate
choice (C).
For choice (D) we have 52 + 2(5) – 35 = 0 and (–7)2 + 2(–7) – 35 = 0. So
the answer is (D).
Notes: (1) c is a zero of a function 𝑓(𝑥) if 𝑓(𝑐) = 0. For example, 5 is a
zero of 𝑥 2 + 2𝑥 − 35 because 52 + 2(5) – 35 = 0.
(2) A polynomial has the form 𝑎𝑛 𝑥 𝑛 + 𝑎𝑛−1 𝑥 𝑛−1 + ⋯ + 𝑎1 𝑥 + 𝑎0
where 𝑎0 , 𝑎1 ,…,𝑎𝑛 are real numbers. For example, 𝑥 2 + 2𝑥 − 35 is a
polynomial.
(3) 𝑝(𝑐) = 0 if and only if 𝑥 − 𝑐 is a factor of the polynomial 𝑝(𝑥).
65. What is the sum of the two solutions of the equation
𝑥 2 − 7𝑥 + 3 = 0 ?
* Quick solution: The sum of the solutions is the negative of the
coefficient of the x term. So the answer is 7.
Notes: (1) If 𝑟 and 𝑠 are the solutions of the quadratic equation
𝑥 2 + 𝑏𝑥 + 𝑐 = 0, then 𝑏 = −(𝑟 + 𝑠) and 𝑐 = 𝑟𝑠. So in this problem the
sum of the two solutions is 7 and the product of the two solutions is 3.
(2) Yes, you can also solve the equation x2 – 7x + 3 = 0 by completing the
square or using the quadratic formula, but this is very time consuming. It
is much better to use the quick solution given above.

86
[Link]

66. A function ℎ is defined as follows:


for 𝑥 > 0, ℎ(𝑥) = 𝑥 7 + 2𝑥 5 − 12𝑥 3 + 15𝑥 − 2
for 𝑥 ≤ 0, ℎ(𝑥) = 𝑥 6 − 3𝑥 4 + 2𝑥 2 − 7𝑥 − 5
What is the value of ℎ(−1) ?
* Since −1 ≤ 0, we use the second equation. It follows that
h(–1) = (–1)6 – 3(–1)4 + 2(–1)2 – 7(–1) – 5 = 1 – 3 + 2 + 7 – 5 = 2.

LEVEL 3: PROBLEM SOLVING AND DATA


67. The mean length of a pop song released in the 1980’s was 4
minutes and 8 seconds. The mean length of a pop song released
in the 1990’s was 4 minutes and 14 seconds. Which of the
following must be true about the mean length of a pop song
released between 1980 and 1999?
(A) The mean length must be equal to 4 minutes and 11
seconds.
(B) The mean length must be less than 4 minutes and 11
seconds.
(C) The mean length must be greater than 4 minutes and 11
seconds.
(D) The mean length must be between 4 minutes and 8
seconds and 4 minutes and 14 seconds.
Solution by picking numbers: Let’s suppose that there are 2 pop songs
from the 80’s each with length 4 minutes and 8 seconds, and 4 pops songs
from the 90’s each with length 4 minutes and 14 seconds. Note that the
given conditions are satisfied, and the combined mean is 4 minutes and
12 seconds. This eliminates choices (A) and (B).
Now let’s reverse the situation, and suppose that there are 4 pop songs
from the 80’s each with length 4 minutes and 8 seconds, and 2 pops songs
from the 90’s each with length 4 minutes and 14 seconds. Note once
again that the given conditions are satisfied, and the combined mean is 4
minutes and 10 seconds. This eliminates choice (C).
Since we have eliminated choices (A), (B), and (C), the answer is choice
(D).
Notes: (1) The combined mean length would only be equal to 4 minutes
and 11 seconds if the same exact number of pops songs were released in
both the 80’s and 90’s.

87
[Link]

(2) It is not true, of course, that there were only 2 pop songs released in
the 80’s and 4 pop songs released in the 90’s. Nonetheless, we can use
these simple numbers to eliminate answer choices.
(3) To actually compute the means above we use the formula
Sum
Mean =
Number
where “Sum” is the sum of all the data, and “Number” is the amount of
data. In each of the examples above there are 6 pieces of data so that the
Number is 6.
(3) When computing the various means in the solution above, we need
only worry about the seconds since the minutes are the same. For
example, in the first paragraph, we can pretend that our data is 8, 8, 14,
8+8+14+14+14+14 72
14, 14, 14. The mean of this data is 6
= 6 = 12. It
follows that the combined mean is 4 minutes and 12 seconds.
* Direct solution: Let’s let 𝑎 be the mean length of a pop song released
in the 1980’s, and 𝑏 be the mean length of a pop song released in the
1990’s. Since 𝑎 < 𝑏, it follows that the combined mean 𝑚 must satisfy
𝑎 < 𝑚 < 𝑏. That is, the combined mean must be between 4 minutes and
8 seconds and 4 minutes and 14 seconds.
68. Twenty one people were playing a game. 1 person scored 50
points, 2 people scored 60 points, 3 people scored 70 points, 4
people scored 80 points, 5 people scored 90 points, and 6 people
scored 100 points. Which of the following correctly shows the
order of the median, mode and average (arithmetic mean) of the
21 scores?
(A) average < median < mode
(B) average < mode < median
(C) median < mode < average
(D) median < average < mode
* The median of 21 numbers is the 11th number when the numbers are
listed in increasing order.
50, 60, 60, 70, 70, 70, 80, 80, 80, 80, 90.
So we see that the median is 90.
The mode is the number that appears most frequently. This is clearly 100.

88
[Link]

Finally, we compute the average.


1 ⋅ 50 + 2 ⋅ 60 + 3 ⋅ 70 + 4 ⋅ 80 + 5 ⋅ 90 + 6 ⋅ 100
≈ 83.33
21
Thus, we see that average < median < mode. This is choice (A).
69. The set 𝑄 consists of 15 numbers whose arithmetic mean is
zero? Which of the following must also be zero?
I. The median of the numbers in 𝑄.
II. The mode of the numbers in 𝑄.
III. The sum of the numbers in 𝑄.
(A) I only
(B) II only
(C) III only
(D) I and III only

* Solution by using a specific list: Consider the following list:


−14, 1, 1, 1, 1, 1, 1, 1, 1, 1, 1, 1, 1, 1, 1
This list has an arithmetic mean of 0, but a median and mode of 1. So I
and II do not need to be true. We can therefore eliminate choices (A), (B)
and (D). So the answer is choice (C).
Note: To see that III must be true recall the formula
Sum = Average · Number
Since the average (arithmetic mean) is zero, so is the sum.

89
[Link]

70. The scatterplot above shows the numbers of incidences of


melanoma, per 100,000 people from 1940 to 1970. Based on the
line of best fit to the data, as shown in the figure, which of the
following values is closest to the average yearly increase in the
number of incidences of melanoma?
(A) 33,00
(B) 13,000
(C) 0.33
(D) 0.13
* The average yearly increase in the number of incidences of melanoma
is just the slope of the line of best fit. So we get approximately
500,000−100,000 400,000
1970−1940
= 30 ≈ 13,333.33. So the best estimate is 13,000,
choice (B).
Notes: (1) Remember that the slope of the line passing through the points
𝑦 −𝑦
(𝑥1 , 𝑦1 ) and (𝑥2 , 𝑦2 ) is 𝑚 = 2 1 .
2 𝑥 −𝑥
1

(2) In this problem we need to be a bit careful about the 𝑦-values. As an


example, the number 1 on the 𝑦 axis actually represents 100,000 people,
and not 1 person. So when we compute the slope we are using the points
(1940, 100,000) and (1970, 500,000).
If we were to use the points (1940,1) and (1970,5) instead we would get
5−1 4
a slope of 1970−1940 = 30 ≈ .13, and we would choose (D), an answer
choice which is not correct!
(3) As an alternative, we can use the points (1940,1) and (1970,5) to get
a slope of approximately .13, and then multiply by 100,000 to get
approximately 13,000, choice (B).

90
[Link]

71. * John, a United States resident, is on vacation in Spain and uses


his credit card to purchase a souvenir for 184 euros. The bank
that issues the credit card converts the purchase price at the
foreign exchange rate for that day, and an additional fee of 6%
of the converted cost is applied before the bank posts the charge.
If the bank posts a charge of $212 to John’s account, what
exchange rate, in Euros per one U.S. dollar, did the bank use?
* If we let 𝐶 be the original cost of the item in dollars, then we have
212
𝐶 + .06𝐶 = 212, or equivalently 1.06𝐶 = 212. So 𝐶 = 1.06 = 200.

So we know that 184 euros corresponds to 200 dollars. We want to know


how many euros correspond to 1 dollar. So we set up a ratio.
The two things being compared are “euros” and “dollars.”
euros 184 𝑥
dollars 200 1

Now draw in the division symbols and equal sign, cross multiply and
divide the corresponding ratio to find the unknown quantity x.

184 𝑥
=
200 1
200𝑥 = 184 ⋅ 1
184
𝑥= = .92
200
So we grid in .92.
72. Let 𝑎, 𝑏 and 𝑐 be numbers with 𝑎 < 𝑏 < 𝑐 such that the average
of 𝑎 and 𝑏 is 2, the average of 𝑏 and 𝑐 is 4, and the average of 𝑎
and 𝑐 is 3. What is the average of 𝑎, 𝑏 and 𝑐 ?
* We change the averages to sums using the formula
Sum = Average ∙ Number
So a + b = 4
b+c=8
a+c=6
Adding these equations gives us 2a + 2b + 2c = 18 so that a + b + c = 9.
9
Finally, we divide by 3 to get that the average of a, b and c is 3 = 3.

91
[Link]

LEVEL 4: HEART OF ALGEBRA


2𝑥 + 𝑦 = 7 − 2𝑦
5𝑦 − 𝑥 = 5 − 4𝑥
73. If (𝑥, 𝑦) is a solution to the above system of equations, what is
𝑦+1
the value of 𝑥 ?

(A) −11
1
(B) − 2
(C) 2
(D) 20
* Solution using the elimination method: We begin by making sure that
the two equations are “lined up” properly. We do this by adding 2y to
each side of the first equation, and adding 4x to each side of the second
equation.
2𝑥 + 3𝑦 = 7
3𝑥 + 5𝑦 = 5
We will now multiply each side of the first equation by 5, and each side
of the second equation by −3.
5(2𝑥 + 3𝑦) = (7)(5)
−3(3𝑥 + 5𝑦) = (5)(−3)
Do not forget to distribute correctly on the left. Add the two equations.
10𝑥 + 15𝑦 = 35
−9𝑥 − 15𝑦 = −15
𝑥 = 20
Using the first equation in the solution to find 𝑦, we have
2 ⋅ 20 + 3𝑦 = 7
40 + 3𝑦 = 7
3𝑦 = 7 − 40 = −33
−33
𝑦= = −11
3
𝑦+1 −11+1 10 1
So 𝑥
= 20
= − 20 = − 2, choice (B).

92
[Link]

Remarks: (1) We chose to use 5 and −3 because multiplying by these


numbers makes the y column “match up” so that when we add the two
equations in the next step the y term vanishes. We could have also used
−5 and 3.
(2) If we wanted to find y first instead of x we would multiply the two
equations by 3 and −2 (or −3 and 2). In general, if you are only looking
for one variable, try to eliminate the one you are not looking for. In this
case we need to find both so it doesn’t matter which we find first.
(3) We chose to multiply by a negative number so that we could add the
equations instead of subtracting them. We could have also multiplied the
first equation by 5, the second by 3, and subtracted the two equations, but
a computational error is more likely to occur this way.
Solution using the substitution method: We solve the second equation
for y and substitute into the first equation.
5−3𝑥 5 3𝑥 3𝑥
5y = 5 – 3x implies 𝑦 = 5
=5− 5
=1− 5
. So now using the first
equation we have
3𝑥 9𝑥 9𝑥
2x = 7 – 3y = 7 – 3(1 – 5
) =7–3+ 5
=4+ 5
.

Multiply each side of this equation by 5 to get rid of the denominator on


the right. So we have 10x = 20 + 9x, and therefore x = 20.
Now proceed as in the previous solution to find 𝑦 = −11, and so
𝑦+1 −11+1 10 1
𝑥
= 20
= − 20 = − 2, choice (B).

Remark: If we wanted to find y first instead of x we would solve the first


equation for x and substitute into the second equation.
Solution using Gauss-Jordan reduction: As in the first solution
(elimination method), we first make sure the two equations are “lined up”
properly.
2𝑥 + 3𝑦 = 7
3𝑥 + 5𝑦 = 5

93
[Link]

Begin by pushing the MATRIX button (which is 2ND x-1). Scroll over to
EDIT and then select [A] (or press 1). We will be inputting a 2 × 3
matrix, so press 2 ENTER 3 ENTER. We then begin entering the
numbers 2, 3, and 7 for the first row, and 3, 5, and 5 for the second row.
To do this we can simply type 2 ENTER 3 ENTER 7 ENTER 3 ENTER
5 ENTER 5 ENTER.
Note: What we have just done was create the augmented matrix for the
system of equations. This is simply an array of numbers which contains
the coefficients of the variables together with the right hand sides of the
equations.
Now push the QUIT button (2ND MODE) to get a blank screen. Press
MATRIX again. This time scroll over to MATH and select rref( (or press
B). Then press MATRIX again and select [A] (or press 1) and press
ENTER.
Note: What we have just done is put the matrix into reduced row
echelon form. In this form we can read off the solution to the original
system of equations.
Warning: Be careful to use the rref( button (2 r’s), and not the ref( button
(which has only one r).
The display will show the following.
[ [1 0 20]
[0 1 − 11]]
The first line is interpreted as x = 20 and the second line as y = −11.
So we have
𝑦+1 −11+1 10 1
= =− = − , choice (B).
𝑥 20 20 2

Graphical solution: We begin by solving each equation for y.


3𝑦 = 7 − 2𝑥 5𝑦 = 5 − 3𝑥
7 2𝑥 3𝑥
𝑦 =3− 3 𝑦 =1− 5

In your graphing calculator press the Y= button, and enter the following.
Y1 = 7/3 – 2X/3
Y2 = 1 – 3X/5

94
[Link]

Now press ZOOM 6 to graph these two lines in a standard window. It


looks like the point of intersection of the two lines is off to the right. So
we will need to extend the viewing window. Press the WINDOW button,
and change Xmax to 50 and Ymin to -20. Then press 2nd TRACE (which
is CALC) 5 (or select INTERSECT). Then press ENTER 3 times. You
will see that the point of intersection of the two lines is (20, −11).
𝑦+1 −11+1 10 1
So 𝑥
= 20
= − 20 = − 2, choice (B).

Remark: The choices made for Xmax and Ymin were just to try to
ensure that the point of intersection would appear in the viewing window.
Many other windows would work just as well.
74. If 𝑦 = 3𝑥 , which of the following expressions is equivalent
to 9𝑥 − 3𝑥+2 for all positive integer values of 𝑥 ?
(A) 3𝑦 − 3
(B) 𝑦 2 − 𝑦
(C) 𝑦 2 − 3𝑦
(D) 𝑦 2 − 9𝑦
Solution by picking a number: Let’s choose a value for 𝑥, say 𝑥 = 2.
Then
𝑦 = 32 = 9, and 9𝑥 − 3𝑥+2 = 92 − 34 = 𝟎.
Put a nice big dark circle around the number 0. We now substitute
𝑦 = 9 into each answer choice.

(A) 3 ⋅ 9 – 3 = 27 – 3 = 24
(B) 92 – 9 = 81 – 9 = 72
(C) 92 – 3 ⋅ 9 = 81 – 27 = 54
(D) 92 – 9 ⋅ 9 = 81 – 81 = 0

Since (A), (B) and (C) are incorrect we can eliminate them. Therefore the
answer is choice (D).

* Algebraic solution:
9𝑥 − 3𝑥+2 = (32 )𝑥 − 3𝑥 32 = (3𝑥 )2 − 9(3𝑥 ) = 𝑦 2 − 9𝑦.
This is choice (D).

95
[Link]

Note: For a review of the basic laws of exponents we used here see
problem 50.

75. In the real numbers, what is the solution of the equation


4𝑥+2 = 82𝑥−1 ?
7
(A) − 4
1
(B) − 4
5
(C) 4
7
(D) 4

* Algebraic solution: The numbers 4 and 8 have a common base of 2. In


fact, 4 = 22 and 8 = 23. So we have 4𝑥+2 = (22 )𝑥+2 = 22𝑥+4 and we have
82𝑥−1 = (23 )2𝑥−1 = 26𝑥−3. Thus, 22𝑥+4 = 26𝑥−3 . So 2x + 4 = 6x – 3. We
subtract 2x from each side of this equation to get 4 = 4x – 3. We now add
3 to each side of this last equation to get 7 = 4x. Finally we divide each
7
side of this equation by 4 to get 4 = x, choice (D).

Notes: (1) For a review of the laws of exponents used here see the end of
the solution to problem 50.
(2) This problem can also be solved by plugging in (start with choice C).
We leave it to the reader to solve the problem this way. Make sure to use
your calculator if you are allowed to.
(7+5𝑖)
76. If 𝑖 = √−1, and (−2−6𝑖)
= 𝑎 + 𝑏𝑖, where 𝑎 and 𝑏 are real
numbers, then what is the value of |𝑎 + 𝑏| ?
(7+5𝑖) (7+5𝑖) (−2+6𝑖) (−14−30)+(42−10)𝑖 −44+32𝑖 44 32
* (−2−6𝑖) = (−2−6𝑖) ⋅ (−2+6𝑖) = 4+36
= 40
= − 40 + 40 𝑖
11 4
= − 10 + 5 𝑖
11 4
So 𝑎 = − 10 and 𝑏 = 5.
11 8 3
Therefore |𝑎 + 𝑏| = |− 10 + 10| = |− 10| = 𝟑/𝟏𝟎 or . 𝟑.

Notes: (1) For a review of multiplying complex numbers see problem 26.
(2) The conjugate of 𝑎 + 𝑏𝑖 is 𝑎 − 𝑏𝑖. The product of conjugates is
always a real number. In fact,

96
[Link]

(𝑎 + 𝑏𝑖)(𝑎 − 𝑏𝑖) = 𝑎2 + 𝑏 2
For example, (−2 − 6𝑖)(−2 + 6𝑖) = (−2)2 + 62 = 4 + 36 = 40.
In practice, if you forget this rule, you can simply do the multiplication
formally.
(3) One way to divide two complex numbers is to multiply both the
numerator and denominator by the conjugate of the denominator. This is
what was done in the first equality in the solution above.
Since the product of conjugates is always a real number, this method
always produces a real number in the denominator. This allows us to
write the quotient in the standard form 𝑎 + 𝑏𝑖.
77. If 𝑥𝑦 = 6, 𝑦𝑧 = 10, 𝑥𝑧 = 15, and 𝑥 > 0, then 𝑥𝑦𝑧 =
Solution by trying a simple operation: The operation to use here is
multiplication.
𝑥𝑦 = 6
𝑦𝑧 = 10
𝑥𝑧 = 15
(𝑥𝑦)(𝑦𝑧)(𝑥𝑧) = (6)(10)(15)
𝑥 2 𝑦 2 𝑧 2 = 900
Notice that we multiply all three left hand sides together, and all three
right hand sides together. Now just take the square root of each side of
the equation to get 𝑥𝑦𝑧 = 30. Thus, the answer is 30.
* Quick computation: With a little practice, we can get the solution to
this type of problem very quickly. Here, we multiply the three numbers
together to get (6)(10)(15) = 900. We then take the square root of 900 to
get 30.
2 5 𝑥
78. If 5 – = 2 – , then | | =
𝑥 𝑥 3
5
* Algebraic solution: We add and subtract 5 from each side of the
𝑥
3
equation to get 𝑥
= −3. We then take the reciprocal of each side to get
𝑥 1 𝑥
3
= − 3. So |3| = 𝟏/𝟑 or .333.

97
[Link]

LEVEL 4: GEOMETRY AND TRIG


1
79. If 𝑐 is a positive constant different from 2, which of the
following could not describe the graph of 𝑥 + 2𝑦 = 𝑐(2𝑥 − 𝑦)
in the 𝑥𝑦-plane?
(A) A straight line passing through the origin that moves
upwards from left to right
(B) A straight line passing through the origin that moves
downwards from left to right
(C) The 𝑦-axis
(D) The 𝑥-axis
1
* Quick algebraic solution: Since 𝑐 CANNOT be 2, the given equation
1
CANNOT be 𝑥 + 2𝑦 = (2𝑥 − 𝑦).
2
Multiplying each side of this equation by 2 yields 2𝑥 + 4𝑦 = 2𝑥 − 𝑦. We
can cancel 2𝑥 from each side to get 4𝑦 = −𝑦, and then adding 𝑦 to each
side of this last equation gives 5𝑦 = 0. Finally, dividing by 5 gives
0
𝑦 = 5 = 0.

The equation 𝑦 = 0 is precisely the 𝑥-axis, choice (D).


* Alternate solution: We solve the equation for 𝑦. Let’s first distribute
the 𝑐 on the right hand side to get 𝑥 + 2𝑦 = 2𝑐𝑥 − 𝑐𝑦. We now add 𝑐𝑦 to
and subtract 𝑥 from each side of the equation to get 2𝑦 + 𝑐𝑦 = 2𝑐𝑥 − 𝑥.
Now factor 𝑦 on the left and 𝑥 on the right to get (2 + 𝑐)𝑦 = (2𝑐 − 1)𝑥.
2𝑐−1
Finally divide by 2 + 𝑐 (assuming 𝑐 ≠ −2) to get 𝑦 = 2+𝑐 𝑥.
1
Note that this is a line passing through the origin. Since 𝑐 ≠ 2, we have
1
2𝑐 − 1 ≠ 2 (2) − 1 = 1 − 1 = 0. So the slope of the line cannot be 0,
and therefore the line cannot be horizontal. Therefore the graph cannot be
the 𝑥-axis, choice (D).
2𝑐−1
Notes: (1) Note that the expression 2+𝑐 is undefined when 𝑐 = −2. Let’s
substitute 𝑐 = −2 into the original equation:
𝑥 + 2𝑦 = (−2)(2𝑥 − 𝑦) = −4𝑥 + 2𝑦
Subtracting 2𝑦 from each side of this equation gives 𝑥 = −4𝑥.
We now add 4𝑥 to each side of this last equation to get 5𝑥 = 0.

98
[Link]

Finally divide each side of this last equation by 5 to get 𝑥 = 0. This is the
𝑦-axis.
This shows that the graph of the given equation CAN be the 𝑦-axis, and
so we can eliminate choice (C).
1 1
(2) If 𝑐 > 2, then 2𝑐 − 1 > 2 (2) − 1 = 1 − 1 = 0, and 2 + 𝑐 > 0, and
2𝑐−1
so 2+𝑐 > 0. In this case the slope of the line is positive, and so the graph
will be a straight line passing through the origin that moves upwards from
left to right. This eliminates choice (A).
1 1
(3) If −2 < 𝑐 < 2, then 2𝑐 − 1 < 2 (2) − 1 = 1 − 1 = 0, and
2𝑐−1
2 + 𝑐 > 2 + (−2) = 0, and so < 0. In this case the slope of the line
2+𝑐
is negative, and so the graph will be a straight line passing through the
origin that moves downwards from left to right. This eliminates (B).
(4) I leave to the reader to determine what the graph of the given equation
looks like when 𝑐 < −2.

80. In the circle above with diameter 𝑑, chords 𝑃𝑄 ̅̅̅̅ and 𝑇𝑈̅̅̅̅ are
3
̅̅̅̅ ̅̅̅̅ ̅̅̅̅
parallel to diameter 𝑅𝑆. If 𝑃𝑄 and 𝑇𝑈 are each 4 of the length of
̅̅̅̅, what is the distance between chords 𝑃𝑄
𝑅𝑆 ̅̅̅̅ and 𝑇𝑈
̅̅̅̅ in terms of
d?
𝑑 √7
(A) 8
𝑑 √7
(B) 4
𝜋𝑑
(C) 4
3𝜋𝑑
(D) 4

99
[Link]

* Let’s add some information to the picture.

We draw segments ̅̅̅̅


𝑂𝑃 and ̅̅̅̅
𝑂𝐸 to form triangle 𝑂𝐸𝑃. Since the circle has
𝑑
diameter 𝑑, the radius of the circle is . Note that ̅̅̅̅
𝑂𝑃 is a radius of the
2
𝑑 3
circle and therefore has length 2 . Now we are given 𝑃𝑄 = 4 𝑑 so that
1 1 3 3𝑑
𝑃𝐸 = 𝑃𝑄 = ( 𝑑) = .
2 2 4 8
We now use the Pythagorean Theorem to find 𝑥:
𝑑 2 3𝑑 2
( ) = 𝑥2 + ( )
2 8
𝑑2 9𝑑 2
4
= 𝑥2 + 64
𝑑2 9𝑑 2 16𝑑 2 9𝑑 2 16𝑑 2 −9𝑑 2 7𝑑 2
𝑥2 = 4
− 64
= 16⋅4
− 64
= 64
= 64
√7𝑑 𝑑 √7
So 𝑂𝐸 = 𝑥 = 8
= 8
.

𝑑 √7 𝑑√7
It follows that the distance between ̅̅̅̅
𝑃𝑄 and ̅̅̅̅
𝑇𝑈 is 2𝑂𝐸 = 2 ⋅ 8 = 4 ,
choice (B).
Notes: (1) In problems involving circles, it is often helpful to draw in
your own radius. To find a suitable radius, look along the circumference
of the circle for “key points.” In the given figure, drawing a radius from
the center of the circle to any of points 𝑃, 𝑄, 𝑇, or 𝑈 would work.
(2) The diameter of a circle is twice the radius, or 𝑑 = 2𝑟. Equivalently,
𝑑
the radius of a circle is half the diameter, or 𝑟 = 2 .

(3) The Pythagorean Theorem says that if a right triangle has legs of
length 𝑎 and 𝑏, and a hypotenuse of length 𝑐, then 𝑐 2 = 𝑎2 + 𝑏 2 .

100
[Link]

Note that the hypotenuse of a right triangle is always opposite the right
angle, and the length of the hypotenuse is always by itself in the formula
for the Pythagorean Theorem.
3𝑑 𝑑
In this problem 𝑎 = 𝑥, 𝑏 = 8
, and 𝑐 = 2.
(4) The distance between two parallel lines (or line segments) is the
length of a line segment between the two lines that is perpendicular to
both lines.
In this case, the distance between ̅̅̅̅
𝑃𝑄 and ̅̅̅̅
𝑇𝑈 is 2𝑂𝐸.

81. In the figure above, sin 𝑘 = ?

(A) 3√2
√2
(B) 2
√3
(C)
2

(D) 1
* Let’s add a little information to the picture.

By the Pythagorean Theorem, c2 = 32 + 32 = 9 + 9 = 18. So c = √18.

101
[Link]

We have √18 = √9 ⋅ 2 = √9√2 = 3√2. So 𝑐 = 3√2.


OPP 3 1 1 √2 √2
Now, sin 𝑘 = HYP = 3 = = ⋅ = , choice (B).
√2 √2 √2 √2 2

Remarks: (1) Since 18 = 9 ⋅ 2, √18 can be simplified as


3 3 1
√18 = √9 ∙ 2 = √9√2 = 3√2. So =3 = .
√18 √2 √2
1
Furthermore, we can rationalize the denominator in the expression to
√2
1 √2 √2
get ∙ = .
√2 √2 2

(2) Instead of using the Pythagorean Theorem, we can observe that the
triangle we formed is an isosceles right triangle which is the same as a 45,
45, 90 right triangle. So the hypotenuse of the triangle has length 3√2
(see the end of the solution to problem 56 for details).
(3) If we are allowed to use a calculator for this problem, then there is no
need to simplify the square root or rationalize the denominator of the
fraction. Once we get 𝑐 = √18, we can use our calculator to get
OPP 3
sin 𝑘 = HYP = ≈ .707.
√18

We now use our calculator to approximate each answer choice, and we


√2
see that 2
≈ .707. So the answer is choice (B).
𝜋
82. Which of the following is equal to cos ( 7 ) ?
𝜋
(A) −cos (− 7 )
𝜋
(B) − sin ( 7 )
5𝜋
(C) sin ( 14 )
5𝜋
(D) − cos ( )
14

* Solution using an identity: We use the following cofunction identity:


𝝅
𝐜𝐨𝐬 𝑨 = 𝐬𝐢𝐧 ( − 𝑨)
𝟐
𝜋 𝜋 𝜋 5𝜋
So we have cos ( 7 ) = sin ( 2 − 7 ) = sin ( 14 ), choice (C).

102
[Link]

Notes: (1) A function 𝑓 with the property that 𝑓(−𝑥) = 𝑓(𝑥) for all 𝑥 in
the domain of 𝑓 is called an even function.
cos 𝑥 is an even function. It follows that cos(−𝐴) = cos 𝐴. In particular,
𝜋 𝜋 𝜋 𝜋 𝜋
cos (− 7 ) = cos ( 7 ), and so − cos (− 7 ) = − cos ( 7 ) ≠ cos ( 7 ). This
eliminates choice (A).
(2) cos 𝐴 and sin 𝐴 are not negatives of each other in general. If
cos 𝐴 sin 𝐴
cos 𝐴 = − sin 𝐴, then sin 𝐴 = −1. Taking reciprocals, cos 𝐴 = −1, so that
3𝜋 7𝜋 11𝜋
tan 𝐴 = −1. This happens only when 𝐴 = ± ,± ,± ,…
4 4 4

This eliminates choice (B).


(3) Since we know that choice (C) is the answer, we can use the same
𝜋
reasoning as in note (2) to show that cos ( ) cannot be equal to
7
5𝜋
−cos ( 14 ).

(4) If a calculator were allowed for this problem, we could simply


approximate the given expression and all the answer choices in our
calculator to get the answer.
Cofunction Identities:
𝝅 𝝅
𝐬𝐢𝐧 ( − 𝑨) = 𝐜𝐨𝐬 𝑨 𝐜𝐬𝐜 ( − 𝑨) = 𝐬𝐞𝐜 𝑨
𝟐 𝟐
𝝅
𝐜𝐨𝐭 ( − 𝑨) = 𝐭𝐚𝐧 𝑨
𝟐

103
[Link]

83. * The head of a copper “hexagon head screw bolt” (one cross
section of which is shown above) has the shape of a cylinder
with a hole shaped like a regular hexagon. The cylindrical head
is 2 cm thick with a base diameter of 3 cm. The hexagonal hole
is only half the thickness of the entire head, and each side of a
hexagonal cross section has a length of 1 cm. Given that the
density of copper is 8.96 grams per cubic cm, and density is
mass divided by volume, find the mass of the head to the nearest
gram.
* We first compute the volume of the head. There are two parts to the
volume.
2
The bottom half of the head is a cylinder with height = 1 cm and base
2
3 3 2 9𝜋
radius 2. It follows that the volume is 𝑉 = 𝜋𝑟 2 ℎ = 𝜋 (2) (1) = 4 cm3.

The top half of the head consists of the same cylinder as the bottom half,
but this time we have to subtract off the volume of a hexagonal prism.
The regular hexagonal face can be divided into 6 equilateral triangles,
𝑠2 √3 12 √3 √3
each with area 𝐴 = 4
= 4
= 4 . So the volume of the hexagonal
6√3 3√3
prism is 𝑉 = 𝐵ℎ = ( ) (1) = cm3 and the volume of the top half of
4 2
9𝜋 3√3
the head is 4
− 2
cm3

It follows that the total volume of the head is


9𝜋 9𝜋 3√3 18𝜋 3√3 9𝜋−3√3
4
+(4 − 2
) = 4
− 2
= 2
cm3.

𝑀 𝑀 9𝜋−3√3
Finally, 𝐷 = 𝑉
⇒ 8.96 = 9𝜋−3√3
⇒ 𝑀 = 8.96 ⋅ 2
≈ 103.39 grams.
2

To the nearest gram, the answer is 𝟏𝟎𝟑.


1 1
Notes: (1) The radius of a circle is 2 the diameter, or r = 2d.

104
[Link]

In this problem the base diameter of the cylinder is 3 cm. It follows that
3
the base radius of the cylinder is 2 cm or 1.5 cm.

(2) The volume of a cylinder is 𝑽 = 𝝅𝒓𝟐 𝒉 where 𝑟 is the base radius of


the cylinder and ℎ is the height of the cylinder.
For example, the bottom half of the screw is a cylinder with base radius
3 3 2 9𝜋
2
cm and height 1 cm. So the volume is 𝑉 = 𝜋 (2) (1) = 4
cm3.

(3) A regular polygon is a polygon with all sides equal in length, and all
angles equal in measure.
The total number of degrees in the interior of an 𝑛-sided polygon is
(𝒏 − 𝟐) ∙ 𝟏𝟖𝟎
For example, a six-sided polygon (or hexagon) has
(6 – 2)·180 = 4·180 = 720 degrees
in its interior. Therefore each angle of a regular hexagon has
720
6
= 120 degrees.

(4) For those of us that do not like to memorize formulas, there is a quick
visual way to determine the total number of degrees in the interior of an
𝑛-sided polygon. Simply split the polygon up into triangles and
quadrilaterals by drawing nonintersecting line segments between vertices.
Then add 180 degrees for each triangle and 360 degrees for each
quadrilateral. For example, here is one way to do it for a hexagon.

105
[Link]

Since the hexagon has been split up into 2 triangles and 1 quadrilateral,
the hexagon has 2(180) + 360 = 720 degrees. This is the same number we
got from the formula.
To avoid potential mistakes, let me give a picture that would be incorrect.

The above figure cannot be used to compute the number of interior


̅̅̅̅ is “crossing through”
angles in the hexagon because segment 𝐴𝐷
̅̅̅̅̅
segment 𝐵𝐹.
(5) Now let’s draw a segment from the center of the hexagon to each
vertex of the hexagon.

106
[Link]

We see that the central angles formed must add up to 360 degrees.
Therefore each central angle is 60 degrees as shown in the figure above.

In general, the number of degrees in a central angle of an 𝑛-sided polygon


𝟑𝟔𝟎
is 𝒏 .

(6) It is worth looking at a regular hexagon in a bit more detail.


Each of the segments drawn in the last figure in note (5) is a radius of the
circumscribed circle of this hexagon, and therefore they are all congruent.
This means that each triangle is isosceles, and so the measure of each of
180−60
the other two angles of any of these triangles is 2 = 60. Therefore
each of these triangles is equilateral. This fact is worth committing to
memory.

√𝟑 𝟐
(7) The area of an equilateral triangle with side length 𝑠 is 𝑨 = 𝒔 (see
𝟒
note (8) below).
It follows that the area of an equilateral triangle with side length 1 is
√3 √3
(1)2 = .
4 4

(8) Most students do not know the formula for the area of an equilateral
triangle, so here is a quick derivation.
Let’s start by drawing a picture of an equilateral triangle with side length
𝑠, and draw an altitude from a vertex to the opposite base. Note that an
altitude of an equilateral triangle is the same as the median and angle
bisector (this is in fact true for any isosceles triangle).

107
[Link]

𝑠
So we get two 30, 60, 90 right triangles with a leg of length 2
and
hypotenuse of length 𝑠.
We can find ℎ by recalling that the side opposite the 60 degree angle has
length √3 times the length of the side opposite the 30 degree angle. So
√3𝑠
ℎ= .
2

Alternatively, we can use the Pythagorean Theorem to find ℎ:


𝑠 2 𝑠2 3𝑠2 √3𝑠
ℎ2 = 𝑠 2 − (2) = 𝑠 2 − 4
= 4
. So ℎ = 2
.

It follows that the area of the triangle is


1 𝑠 𝑠 √3𝑠 1 √3𝑠 √3 2
𝐴 = ( + )( ) = 𝑠( ) = 𝑠 .
2 2 2 2 2 2 4

(9) The volume of a prism is 𝑽 = 𝑩𝒉 where 𝐵 is the area of the base of


the prism and ℎ is the height of the prism.
6√3 3√3
In this problem we have a hexagonal prism with 𝐵 = = and
4 2
3√3 3 √3
ℎ = 1. It follows that the volume of this prism is 𝑉 = ( ) (1) = .
2 2

84. * If cos 𝑥 = 0.32, then what is the value of sin 𝑥 tan 𝑥 ?


* 𝑥 = cos−1 0.32, and so
sin 𝑥 tan 𝑥 = sin (cos−1 0.32) tan (cos−1 0.32) ≈ 2.81.

108
[Link]

LEVEL 4: PASSPORT TO ADVANCED MATH


1 3 1
+ =
𝑥 𝑥 2
85. Dennis is helping Billy assemble his new computer desk. Billy
can put the desk together three times as fast as Dennis, and
together Billy and Dennis can finish assembling the desk in 2
hours. The equation above represents this situation. Which of
3
the following describes what the expression represents in this
𝑥
equation?
(A) The fraction of the job that would be completed by Billy
in 1 hour.
(B) The fraction of the job that would be completed by
Dennis in 1 hour.
(C) The time, in hours, that it takes Billy to complete one
third of the job.
(D) The time, in hours that it takes Billy to assemble the desk
by himself.
Solution by solving the equation: We can solve the given equation by
1 3 1
first multiplying each side by 2𝑥 to get 2𝑥 ⋅ 𝑥 + 2𝑥 ⋅ 𝑥 = 2𝑥 ⋅ 2, or
3 𝟑
equivalently 2 + 6 = 𝑥. So 𝑥 = 8. It follows that 𝑥 = 𝟖.
3
It seems reasonable at this point that would represent the fraction of the
8
job that Billy would complete in 1 hour.
3
Let’s verify this. We assume that Billy can complete 8 of the job in 1
hour. Since Billy can put the desk together three times as fast as Dennis,
1
it follows that Dennis can complete 8 of the job in 1 hour. So together
3 1 4 1
they can complete + = = the job in 1 hour, and therefore they can
8 8 8 2
complete the whole job in 2 hours.
Since everything works out, the answer is choice (A).
1
Notes: (1) 𝑥 represents the fraction of the job that would be completed by
1
Dennis in 1 hour. So Dennis can complete 8 of the job in 1 hour.

109
[Link]

1
(2) 2 represents the fraction of the job that both Billy and Dennis,
working together, can complete in 1 hour.
(3) The time it would take Dennis to complete the job himself is 8 hours.
8
(4) The time it would take Billy to complete the job himself is hours (or
3
2
2 3 hours).
1
* Direct solution: The right hand side of the equation, , is the fraction
2
of the job that both Billy and Dennis, working together, can complete in 1
hour.
1
On the left hand side of the equation we are expressing 2 as a sum of two
1
terms. Each term represents the portion of 2 that each of Billy and Dennis
3
contribute. Since Billy works three times as fast as Dennis, is the
𝑥
fraction of the job that Billy contributes, choice (A).
86. For all real numbers 𝑥, let the function 𝑔 be defined by
𝑔(𝑥) = 𝑝(𝑥 − ℎ)2 + 𝑘, where 𝑝, ℎ, and 𝑘 are constants with
𝑝, 𝑘 > 0. Which of the following CANNOT be true?
(A) 𝑔(7) = −ℎ
(B) 𝑔(7) = 2
(C) 𝑔(0) = −2
(D) 𝑔(0) = 2
Solution by plugging in answer choices: Let’s start with choice (C) and
suppose that 𝑔(0) = −2. We have −2 = 𝑝(0 − ℎ)2 + 𝑘 = 𝑝ℎ2 + 𝑘.
Since 𝑝 and 𝑘 are greater than 0, 𝑝ℎ2 + 𝑘 > 0. Therefore 𝑝ℎ2 + 𝑘
CANNOT be −2, and the answer is choice (C).
Eliminating the other answer choices: This isn’t necessary to solve the
problem, but for completeness let’s show that each of the other answer
choices CAN be true.
(A) If 𝑔(7) = −ℎ, then −ℎ = 𝑝(7 − ℎ)2 + 𝑘. Let ℎ = −1. Then
1 1 1
1 = 64𝑝 + 𝑘, and so 𝑘 = 1 − 64𝑝. Now let 𝑝 = . Then 𝑘 = 1 − = .
128 2 2
2
(B) If 𝑔(7) = 2, then 2 = 𝑝(7 − ℎ) + 𝑘. Let ℎ = 0. So 2 = 49𝑝 + 𝑘,
1
and therefore 𝑘 = 2 − 49𝑝. Now let 𝑝 = 49. Then 𝑘 = 2 − 1 = 1.

110
[Link]

(D) If 𝑔(0) = 2, then 2 = 𝑝(0 − ℎ)2 + 𝑘. Let ℎ = 0 and 𝑝 = 1. Then


𝑘 = 2.
87. On January 1, 2015, a family living on an island releases their
two pet rabbits into the wild. Due to the short gestation period of
rabbits, and the fact that the rabbits have no natural predators on
this island, the rabbit population doubles each month. If 𝑃
represents the rabbit population 𝑡 years after January 1, 2015,
then which of the following equations best models the rabbit
population on this island over time?
𝑡+12
(A) 𝑃 = 2 12
(B) 𝑃 = 2𝑡+1
(C) 𝑃 = 212𝑡
(D) 𝑃 = 212𝑡+1
1
Solution by picking a number: Let’s choose a value for 𝑡, say 𝑡 = 2.
1
Note that 2 year is equal to 6 months.
Since there are initially 2 rabbits, after 1 month there are 2 ⋅ 2 = 4
rabbits, after 2 months there are 2 ⋅ 4 = 8 rabbits, after 3 months there are
2 ⋅ 8 = 16 rabbits, after 4 months there are 2 ⋅ 16 = 32 rabbits, after 5
months there are 2 ⋅ 32 = 64 rabbits, and after 6 months there are
2 ⋅ 64 = 𝟏𝟐𝟖 rabbits.
Put a nice big, dark circle around the number 128 so you can find it easily
later.
1
We now substitute 𝑡 = into each answer choice:
2
1 1 24
+12 + 25 25 1 25
2 2 2
(A) 2 12 =2 12 = 2 2 ÷12 = 2 2 ⋅12 = 224
1 1 2 3 1
(B) 22+1 = 22+2 = 22 = 82 = √8 = √4√2 = 2√2
1
(C) 212⋅2 = 26 = 64
1
(D) 212⋅2+1 = 26+1 = 27 = 128
Since choices (A), (B), and (C) did not come out correct, the answer is
choice (D).
1
Notes: (1) Note that we picked the number 2 as opposed to an integer like
2. This is because an integer like 2 will force us to do 24 computations or
recognize a pattern. The fraction minimizes the amount of computation

111
[Link]

and the amount of critical reasoning needed. An even smaller fraction


1 1
like 3 or 4 would have saved even more time.

(2) Instead of multiplying by 2 repeatedly we could save time by finding


a pattern as follows:
After 0 months there are 2 = 21 rabbits.
After 1 month there are 2 ⋅ 2 = 22 rabbits.
After 2 months there are 2 ⋅ 22 = 21+2 = 23 rabbits.
This should be enough to see the pattern. Can you see that After 𝑛
months there are 2𝑛+1 rabbits? In particular, after 6 months there are
27 = 128 rabbits.
(3) There is no reason to continue evaluating answer choices once it is
clear that the choice will not yield the correct answer. For example, for
1
+12 12.5
2
choice (A) it is not hard to see that 2 12 = 2 12 could not possibly be
equal to 128.
(5) If we are allowed to use a calculator for this problem, we could use it
1
to compute the answer choices quickly. For example, with 𝑡 = , choice
2
(A) would give 2^( (1 / 2 + 12) / 12) ≅ 2.0586. Clearly this is incorrect
and so we can eliminate it.
(6) When using the strategy of picking numbers it is very important that
you check every answer choice. It is possible for more than one choice to
come out to the correct answer. You would then need to pick a new
number to try to eliminate all but one choice.
* Solution using the exponential growth model formula: A quantity
that continually doubles over a fixed time period can be modeled by the
𝑡
exponential function 𝑃 = 𝑐(2)𝑑 where 𝑐 is the quantity at time 𝑡 = 0, and
𝑑 is the doubling time in years. In this case, there are initially 2 rabbits,
1
so that 𝑐 = 2, and the doubling time is every month, or every 12 year.
1
It follows that 𝑃 = 2(2)𝑡÷12 = 2(2)12𝑡 = 21 212𝑡 = 21+12𝑡 = 212𝑡+1 ,
choice (D).
Note: See problem 50 for a review of the laws of exponents used here.

112
[Link]

Solution using a general exponential function: A general exponential


function has the form 𝑃(𝑡) = 𝑎 ⋅ 𝑏 𝑐𝑡 , where 𝑎 = 𝑃(0) is the initial
amount and 𝑏 is the exponential rate.
In this problem, 𝑎 = 𝑃(0) = 2, and since the population is doubling, the
exponential rate is also 𝑏 = 2. So we have 𝑃(𝑡) = 2 ⋅ 2𝑐𝑡 .
We are also given that the population doubles each month, so that
𝑐 𝑐 𝑐
1
𝑃 (12) = 4. So we have 4 = 2 ⋅ 212 = 21+12 . So 22 = 21+12 , and
𝑐
therefore 1 + 12 = 2. Subtracting 1 from each side of this equation yields
𝑐
12
= 2 − 1 = 1, and so 𝑐 = 1 ⋅ 12 = 12.

So 𝑃(𝑡) = 2 ⋅ 212𝑡 = 21 ⋅ 212𝑡 = 21+12𝑡 = 212𝑡+1 , choice (D).


𝑔(𝑥) = 𝑥 4 − 𝑘𝑥 3 + 13𝑥 2 − 12𝑥 + 4
88. The function 𝑔 is defined above, and 𝑘 is a constant. In the
𝑥𝑦-plane, the graph of 𝑔 intersects the 𝑦-axis at (0,4) and
intersects the 𝑥-axis at (1,0) and (2,0). What is the value of 𝑘?
* Solution by plugging in the given points: We use the point (1,0) and
substitute in 1 for 𝑥 and 0 for 𝑔(𝑥) to get
0 = 14 − 𝑘 ⋅ 13 + 13 ⋅ 12 − 12 ⋅ 1 + 4 = 1 − 𝑘 + 13 − 12 + 4 = 6 − 𝑘.
So 𝑘 = 𝟔.
Notes: (1) If a point is an intersection point of the graph of 𝑔 and
something else, then in particular, that point lies on the graph of 𝑔.
This means that we can substitute any of the given points into the
equation for 𝑔 to get a true statement.
(2) For example, let’s look at the point (0,4). If we substitute in 𝑥 = 0
and 𝑦 = 𝑔(𝑥) = 4 in the given equation, we get
4 = 04 − 𝑘(0)3 + 13(0)2 − 12(0) + 4
or equivalently, 4 = 4, a true statement.
Unfortunately using this particular point isn’t particularly helpful since
the resulting equation does not involve 𝑘.
(3) It is much more useful to use the point (1,0) as we did in the above
solution.

113
[Link]

(4) We could also use the point (2,0), although this will lead to a messier
computation:
0 = 24 − 𝑘(2)3 + 13(2)2 − 12 ⋅ 2 + 4
So
0 = 16 − 8𝑘 + 13 ⋅ 4 − 24 + 4 = 16 − 8𝑘 + 52 − 24 + 4 = 48 − 8𝑘
Adding 8𝑘 to each side of this last equation gives 8𝑘 = 48. So we have
48
𝑘 = 8 = 𝟔, as in the original solution.

3𝑥 2 + 2𝑦 2 = 550
2𝑥 + 12𝑦 = 0
89. If (𝑥, 𝑦) is a solution to the system of equations above, what is
the value of 𝑦 2 ?
* Solution by substitution: We solve the second equation for 𝑥 by first
subtracting 12𝑦 from each side to get 2𝑥 = −12𝑦. We then divide each
side of this last equation by 2 to get 𝑥 = −6𝑦.
Now we replace 𝑥 by −6𝑦 in the left hand side of the first equation to get
3𝑥 2 + 2𝑦 2 = 3(−6𝑦)2 + 2𝑦 2 = 3(36𝑦 2 ) + 2𝑦 2
= 108𝑦 2 + 2𝑦 2 = 110𝑦 2 .
550
So we have 110𝑦 2 = 550, and so 𝑦 2 = 110 = 𝟓.

90. The graph of 𝑔(𝑥) = (3𝑥 − 9)(𝑥 − 1) is a parabola in the xy-


plane. If the vertex of this parabola has coordinates (ℎ, 𝑘), what
is the value of ℎ − 𝑘 ?
* Solution by using the 𝒙-intercepts of the graph: 𝑔(𝑥) = 0 when
9
3𝑥 − 9 = 0 and 𝑥 − 1 = 0, or equivalently 𝑥 = 3 = 3 and 𝑥 = 1.

The 𝑥-coordinate of the vertex of the parabola is midway between these


3+1 4
two 𝑥 values, and so ℎ = = = 2. It follows that
2 2

𝑘 = 𝑔(2) = (3 ⋅ 2 − 9)(2 − 1) = (−3)(1) = −3.


So ℎ − 𝑘 = 2 − (−3) = 2 + 3 = 𝟓.
Notes: (1) 𝑥 = 1 and 𝑥 = 3 are called zeros, roots, or solutions of the
function 𝑔. These are the 𝑥-values at which 𝑔(𝑥) = 0.

114
[Link]

For example, 𝑔(1) = (3 ⋅ 1 − 9)(1 − 1) = (−6)(0) = 0.


(2) If 𝑥 = 𝑐 is a zero of a function 𝑔, then the point (𝑐, 0) is an
𝑥-intercept of the graph of 𝑔.
So in this example, the 𝑥-intercepts of the graph of 𝑔 are (1,0) and (3,0).
(3) The 𝑥-coordinate of the vertex (or turning point) of a parabola is
always midway between the 𝑥 values of the two 𝑥-intercepts of the
parabola (if they exist).
(4) Once we find the 𝑥-coordinate ℎ of the vertex, we can find the
𝑦-coordinate 𝑘 of the vertex by substituting ℎ into the function 𝑔. That is,
𝑘 = 𝑔(ℎ).
Solution by putting the quadratic function into standard form: We
have 𝑔(𝑥) = (3𝑥 − 9)(𝑥 − 1) = 3𝑥 2 − 12𝑥 + 9
= 3(𝑥 2 − 4𝑥 + 3) = 3(𝑥 − 2)2 − 3 (details below).
From the final form of 𝑔 we see that the vertex of the parabola is (2, −3).
So ℎ = 2, 𝑘 = −3, and therefore ℎ − 𝑘 = 2 − (−3) = 2 + 3 = 𝟓.
Notes: (1) The general form for the equation of a parabola is
𝒚 = 𝒂𝒙𝟐 + 𝒃𝒙 + 𝒄.
We can put the function 𝑦 = 𝑔(𝑥) into general form by simply
multiplying (3𝑥 − 9) and (𝑥 − 1) together to get
(3𝑥 − 9)(𝑥 − 1) = 3𝑥 2 − 3𝑥 − 9𝑥 + 9 = 3𝑥 2 − 12𝑥 + 9
This form however is not that useful for identifying specific information
about the parabola such as the vertex.
(3) The standard form for the equation of a parabola is
𝒚 = 𝒂(𝒙 − 𝒉)𝟐 + 𝒌, or equivalently, 𝒚 − 𝒌 = 𝒂(𝒙 − 𝒉)𝟐
In either of these forms, we can identify the vertex of the parabola as
(ℎ, 𝑘).
(4) To change an equation of a parabola from general form to standard
form, we use a procedure called completing the square.
To complete the square on the expression 𝑥 2 + 𝑏𝑥, we take half of the
number 𝑏, and square the result to get 𝑏 2 .

115
[Link]

For example, to complete the square on 𝑥 2 − 4𝑥, we half −4 to get −2,


and then square −2 to get (−2)2 = 4.
We then add this to the original expression to get 𝑥 2 − 4𝑥 + 4. This new
expression is a perfect square. In fact, it factors as follows:
𝑥 2 − 4𝑥 + 4 = (𝑥 − 2)(𝑥 − 2) = (𝑥 − 2)2
Note that the number −2 is the same as the number we got from taking
half of −4. This is not a coincidence. It always happens.
(5) We must have an expression of the form 𝑥 2 + 𝑏𝑥 before completing
the square. In other words, there cannot be a number in front of 𝑥 2 (in
technical terms, the coefficient of 𝑥 2 must be 1).
In the given problem there is a coefficient of 3 in front of 𝑥 2 . We deal
with this by factoring 3 from the expression:
𝑔(𝑥) = 3𝑥 2 − 12𝑥 + 9 = 3(𝑥 2 − 4𝑥 + 3)
(6) Completing the square does not produce an expression that is
equivalent to the original expression. For example, the expression
(𝑥 − 2)2 = 𝑥 2 + 4𝑥 + 4 is 4 more than the original expression 𝑥 2 + 4𝑥.
We can fix this problem in two different ways:
Method 1: Add and subtract what we need inside the parentheses
Using this method, we write 𝑔(𝑥) = 3(𝑥 2 − 4𝑥 + 4 − 4 + 3).
Notice how we added and subtracted 4 inside the parentheses.
We can now simplify this expression to
𝑔(𝑥) = 3(𝑥 2 − 4𝑥 + 4 − 1) = 3(𝑥 2 − 4𝑥 + 4) − 3 = 3(𝑥 − 2)2 − 3
Note how we distributed the 3 to (𝑥 2 − 4𝑥 + 4) and (−1), a slightly
unconventional use of the distributive property.
Method 2: Add what we need inside the parentheses and adjust the other
constant accordingly.
Using this method, we write 𝑔(𝑥) = 3(𝑥 2 − 4𝑥 + 4 − 1).
Since we added 4, we adjusted the 3 by subtracting 4, ie. 3 − 4 = −1.
Now proceed as in Method 1 to put the equation in standard form.

116
[Link]

(7) If you don’t like dealing with the 3 on the right hand side of the
equation, it can temporarily be moved to the left before completing the
square as follows:
𝑔(𝑥)
We rewrite 𝑔(𝑥) = 3(𝑥 2 − 4𝑥 + 3) as 3
= 𝑥 2 − 4𝑥 + 3.

Now complete the square on 𝑥 2 − 4𝑥 to get 𝑥 2 − 4𝑥 + 4. So we need to


add 4 to the right hand side of the equation. We can undo this by either (i)
subtracting 4 from the same side (as in Method 1 above), (ii) adjusting
the 3 to −1 (as in Method 2 above), or (iii) adding 4 to the left hand side
to balance the equation.
𝑔(𝑥)
Let’s use (iii) this time and write = 𝑥 2 − 4𝑥 + 3 as
3

𝑔(𝑥)
+ 4 = 𝑥 2 − 4𝑥 + 4 + 3
3
𝑔(𝑥)
After completing the square we have 3
+ 4 = (𝑥 − 2)2 + 3.
𝑔(𝑥)
Subtracting 4 gives 3
= (𝑥 − 2)2 − 1.

Finally, multiplying by 3 yields 𝑔(𝑥) = 3(𝑥 − 2)2 − 3.


(8) Once the equation is in the standard form
𝑔(𝑥) = 3(𝑥 − 2)2 − 3 or 𝑔(𝑥) + 3 = 3(𝑥 − 2)2
we can easily pick out the vertex by matching the equation up with the
standard form
𝑦 = 𝑎(𝑥 − ℎ)2 + 𝑘 or 𝑦 − 𝑘 = 𝑎(𝑥 − ℎ)2
Observe that ℎ = 2 and 𝑘 = −3.
(9) It is very common for students to make sign errors here. Note that the
expression (𝑥 − 2)2 indicates that ℎ = 2, whereas the expression
𝑔(𝑥) + 3 indicates that 𝑘 = −3.
To see this, note that 𝑔(𝑥) + 3 = 𝑔(𝑥) − (−3).

117
[Link]

LEVEL 4: PROBLEM SOLVING AND DATA


𝑆 = 25.33𝐻 + 353.16
91. The linear regression model above is based on an analysis of the
relationship between SAT math scores (𝑆) and the number of
hours spent studying for SAT math (𝐻). Based on this model,
which of the following statements must be true?
I. The slope indicates that as 𝐻 increases by 1, 𝑆 decreases by
25.33.
II. For a student that studies 15 hours for SAT math, the
predicted SAT math score is greater than 700.
III. There is a negative correlation between 𝐻 and 𝑆.
(A) I only
(B) II only
(C) III only
(D) I and II only
25.33
* The slope of the line is 25.33 = . This indicates that as 𝐻 increases
1
by 1, 𝑆 increases by 25.33. Also since the slope is positive, there is a
positive correlation between 𝐻 and 𝑆. So I and III are false, and the
answer must be choice (B).
Notes: (1) We did not have to check II because once we determined that I
and III were false, there was only one answer choice left that excluded
both of them.
(2) For completeness let’s check that II is true. To see this we just need to
perform the following computation: 25.33(15) + 353.16 = 733.11 > 700.

118
[Link]

92. 2500 single men and 2500 single women were asked about
whether they owned any dogs or cats. The table below displays
a summary of the results.

Dogs Cats
Both Neither Total
Only Only
Men 920 270 50 1260 2500
Women 750 430 340 980 2500
Total 1670 700 390 2240 5000

Of the people who said they had neither dogs nor cats, 200 were
selected at random, and they were asked if they had any pets at
all. 43 people said they did have pets, and the remaining 157
said that they did not. Based on both the initial data given in the
table, together with the new data stated in this paragraph, which
of the following is most likely to be accurate?
(A) Approximately 482 of the original people surveyed
would say that they have no pets.
(B) Approximately 1758 of the original people surveyed
would say that they have no pets.
(C) Approximately 1963 of the original people surveyed
would say that they have no pets.
(D) Approximately 3925 of the original people surveyed
would say that they have no pets.
157
* 200 ⋅ 2240 = 1758.4 ≈ 1758. So the answer is choice (B).

Notes: (1) We are being asked to extrapolate information from a random


sample. In this case we want to estimate the number of people who have
no pets at all. We can do this by multiplying the fraction of people in the
random sample who have no pets by the total number of people with no
cat or dogs.
157
(2) The fraction of people from the random sample with no pets is .
200

(3) The total number of people with no cats or dogs is 2240, as can be
seen from the table.

119
[Link]

93. On a certain exam, the median grade for a group of 25 students


is 67. If the highest grade on the exam is 90, which of the
following could be the number of students that scored 67 on the
exam?
I. 5
II. 20
III. 24
(A) I only
(B) III only
(C) I and II only
(D) I, II, and III
* The highest exam grade must be 90. If the other 24 exam grades are 67,
then the median will be 67. So III is possible.
If 5 of the exam grades are 90 and the remaining 20 are 67, then the
median will be 67. So II is possible.
Finally, if 10 of the exam grades are 90, 10 are 60 and 5 are 67, then the
median will be 67. So I is possible.
Therefore the answer is choice (D).
94. If the average (arithmetic mean) of 𝑘 and 𝑘 + 7 is 𝑏 and if the
average of 𝑘 and 𝑘 − 11 is 𝑐, what is the sum of 𝑏 and 𝑐?
(A) 2𝑘 − 2
(B) 2𝑘 − 1
(C) 2𝑘
1
(D) 2𝑘 +
2

* Solution by changing averages to sums: Note that the sum of k and


k + 7 is k + (k + 7) = 2k + 7, so that 2k + 7 = 2b. Similarly, the sum of k
and k – 11 is k + (k – 11) = 2k – 11 so that 2k – 11 = 2c. So,
2b + 2c = 4k – 4
2(b + c) = 4k – 4
4𝑘 – 4 4𝑘 4
𝑏+𝑐= 2
= 2
– 2 = 2k – 2

Thus, the answer is choice (A).


Solution by picking numbers: Let us choose a value for k, say k = 5. It
follows that k + 7 = 5 + 7 = 12 and k – 11 = 5 – 11 = −6. So,

120
[Link]

5 + 12 17
b= 2
= 2 = 8.5
5−6 1
c= 2
= − 2 = −0.5
and the sum of b and c is 𝑏 + 𝑐 = 8.5 − 0.5 = 8. Put a nice big, dark circle
around this number so that you can find it easily later. We now substitute
k = 5 into each answer choice.
(A) 8
(B) 9
(C) 10
(D) 10.5
Compare each of these numbers to the number that we put a nice big,
dark circle around. Since (B), (C) and (D) are incorrect we can eliminate
them. Therefore the answer is choice (A).
Important note: (A) is not the correct answer simply because it is equal
to 8. It is correct because all three of the other choices are not 8. You
absolutely must check all four choices!
95. A farmer purchased several animals from a neighboring farmer:
6 animals costing $100 each, 10 animals costing $200 each, and
𝑘 animals costing $400 each, where 𝑘 is a positive odd integer.
If the median price for all the animals was $200, what is the
greatest possible value of 𝑘?
* Solution by listing: Let’s list the prices in increasing order, including
repetitions.
100, 100, 100, 100, 100, 100, 200, 200, 200, 200, 200, 200, 200, 200,
200, 200, 400,…
In order for k to be as large as possible we need the 200 in bold to be the
median. Since there are 15 numbers before the bold 200, we need 15
numbers after the bold 200 as well. So k = 15.
96. The average (arithmetic mean) salary of employees at an
advertising firm with 𝑃 employees in thousands of dollars is 53,
and the average salary of employees at an advertising firm with
𝑄 employees in thousands of dollars is 95. When the salaries of
both firms are combined, the average salary in thousands of
𝑃
dollars is 83. What is the value of 𝑄?

121
[Link]

* Solution by changing averages to sums: The Sum of the salaries of


employees at firm P (in thousands) is 53P.
The Sum of the salaries of employees at firm Q (in thousands) is 95Q.
Adding these we get the Sum of the salaries of all employees (in
thousands): 53P + 95Q.
We can also get this sum directly from the problem.
83(P + Q) = 83P + 83Q.
So we have that 53P + 95Q = 83P + 83Q.
We get P to one side of the equation by subtracting 53P from each side,
and we get Q to the other side by subtracting 83Q from each side.
12Q = 30P
𝑃
We can get 𝑄
to one side by performing cross division. We do this just
like cross multiplication, but we divide instead. Dividing each side of the
equation by 30Q will do the trick (this way we get rid of Q on the left and
30 on the right).
𝑃 12 𝟐
𝑄
= 30 = 𝟓

So we can grid in 2/5 or .4.

LEVEL 5: HEART OF ALGEBRA


3 3𝑦
97. If 𝑥 12 = 𝑧 and 𝑥 11 = 𝑧
which of the following is an expression
for 𝑥 in terms of 𝑦?

(A) 3𝑦
(B) 2𝑦
(C) 𝑦
1
(D)
𝑦

* Solution by trying a simple operation: The operation to use here is


division. We divide the left hand sides of each equation, and the right
hand sides of each equation. First the left. Recall that when we divide
expressions with the same base we need to subtract the exponents.

122
[Link]

𝑥 12
Therefore we have 𝑥 11
= 𝑥 1 = 𝑥. Now for the right. Recall that dividing
3 3𝑦 3 𝑧 1
is the same as multiplying by the reciprocal. So, 𝑧
÷ 𝑧
= 𝑧 ⋅ 3𝑦 = 𝑦.
1
Therefore, 𝑥 = and the answer is choice (D).
𝑦

Alternate Solution: Multiply each side of each equation by 𝑧 to get


𝑧𝑥 12 = 3
𝑧𝑥 11 = 3𝑦
Multiplying each side of the second equation by 𝑥 yields
𝑧𝑥 12 = 3𝑥𝑦
So we have 3𝑥𝑦 = 3 from which it follows that 𝑥𝑦 = 1, or equivalently
1
𝑥 = 𝑦. Thus, the answer is choice (D).

3𝑥 − 7𝑦 = 12
𝑘𝑥 + 21𝑦 = −35
98. For which of the following values of 𝑘 will the system of
equations above have no solution?
(A) 9
(B) 3
(C) −3
(D) −9
The system of equations
𝑎𝑥 + 𝑏𝑦 = 𝑐
𝑑𝑥 + 𝑒𝑦 = 𝑓
𝑎 𝑏 𝑐 3 −7
has no solution if 𝑑
= 𝑒 ≠ 𝑓 . So we solve the equation 𝑘
= 21
. Cross
63
multiplying yields 63 = −7𝑘 so that 𝑘 = −7
= −9, choice (D).
𝑏 𝑐 −7 12
Note: In this problem 𝑒 ≠ 𝑓 . Indeed, 21 ≠ −35 . This guarantees that the
system of equations has no solution instead of infinitely many solutions.
* Quick solution: We multiply −7 by −3 to get 21. So we have
𝑘 = (3)(−3) = −9, choice (D).
The general form of an equation of a line is 𝒂𝒙 + 𝒃𝒚 = 𝒄 where 𝑎, 𝑏
𝑎
and 𝑐 are real numbers. If 𝑏 ≠ 0, then the slope of this line is 𝑚 = − 𝑏 . If
𝑏 = 0, then the line is vertical and has no slope.

123
[Link]

Let us consider 2 such equations.


𝒂𝒙 + 𝒃𝒚 = 𝒄
𝒅𝒙 + 𝒆𝒚 = 𝒇
(1) If there is a number 𝑟 such that 𝑟𝑎 = 𝑑, 𝑟𝑏 = 𝑒, and 𝑟𝑐 = 𝑓, then the
two equations represent the same line. Equivalently, the two equations
𝑎 𝑏 𝑐
represent the same line if 𝑑 = 𝑒 = 𝑓. In this case the system of equations
has infinitely many solutions.
(2) If there is a number 𝑟 such that 𝑟𝑎 = 𝑑, 𝑟𝑏 = 𝑒, but 𝑟𝑐 ≠ 𝑓, then the
two equations represent parallel but distinct lines. Equivalently, the two
𝑎 𝑏 𝑐
equations represent parallel but distinct lines if 𝑑 = 𝑒 ≠ 𝑓. In this case the
system of equations has no solution.
𝑎 𝑏
(3) Otherwise the two lines intersect in a single point. In this case ≠ ,
𝑑 𝑒
and the system of equations has a unique solution.
These three cases are illustrated in the figure below.

Additional examples: The following two equations represent the same


line.
𝟐𝒙 + 𝟖𝒚 = 𝟔
𝟑𝒙 + 𝟏𝟐𝒚 = 𝟗
2 8 6 3
To see this note that 3
= 12 = 9 (or equivalently, let 𝑟 = 2 and note that
3 3 3
(2) (2) = 3, (2) (8) = 12, and (2) (6) = 9).

The following two equations represent parallel but distinct lines.


𝟐𝒙 + 𝟖𝒚 = 𝟔
𝟑𝒙 + 𝟏𝟐𝒚 = 𝟏𝟎

124
[Link]

2 8 6
This time 3 = 12 ≠ 10 .

The following two equations represent a pair of intersecting lines.


𝟐𝒙 + 𝟖𝒚 = 𝟔
𝟑𝒙 + 𝟏𝟎𝒚 = 𝟗
2 8
This time 3 ≠ 10 .

99. If 𝑥 + 𝑦 = 2𝑘 − 1, and 𝑥 2 + 𝑦 2 = 9 − 4𝑘 + 2𝑘 2 , what is 𝑥𝑦 in


terms of 𝑘?
(A) 𝑘−2
(B) (𝑘 − 2)2
(C) (𝑘 + 2)2
(D) 𝑘2 − 4

Solution by picking numbers: Let k = 0. Then we have x + y = −1, and


x2 + y2 = 9.
(x + y)2 = (x + y)(x + y) = x2 + 2xy + y2 = x2 + y2 + 2xy.
(−1)2 = 9 + 2xy
1 = 9 + 2xy
−8 = 2xy
−4 = xy
Put a nice big dark circle around the number −𝟒. Now substitute k = 0
into each answer choice.
(A) −2
(B) 4
(C) 4
(D) −4

Since (A), (B) and (C) came out incorrect we can eliminate them, and the
answer is choice (D).

* Algebraic solution:
(x + y)2 = x2 + y2 + 2xy.
(2k – 1)2 = 9 – 4k + 2k2 + 2xy
4k2 – 4k + 1 = 9 – 4k + 2k2 + 2xy
2k2 – 8 = 2xy
2(k2 – 4) = 2xy
k2 – 4 = xy
So xy = k – 4, choice (D).
2

125
[Link]

100. On the number line, the distance between the point whose
coordinate is 𝑠 and the point whose coordinate is 𝑡 is greater
than 500. Which of the following must be true?
I. |𝑠| ∙ |𝑡| > 500
II. |𝑠 − 𝑡| > 500
III. 𝑡 − 𝑠 > 500
(A) I only
(B) II only
(C) I and II only
(D) I, II, and III
* The first sentence is precisely the statement of II. Letting s = 1000 and
t = 0 gives a counterexample for both I and III. The answer is choice (B).
𝑘 = 𝑎 − 𝑏 + 12
𝑘 = 𝑏 − 𝑐 − 17
𝑘 = 𝑐 − 𝑎 + 11
101. In the system of equations above, what is the value of k ?
* Solution by trying a simple operation: Notice that when we add the
three given equations, all the variables on the right hand side add to zero.
6
So we have 3𝑘 = 12 − 17 + 11 = 6. Therefore 𝑘 = 3 = 𝟐.

102. An elephant traveled 7 miles at an average rate of 4 miles per


hour and then traveled the next 7 miles at an average rate of 1
mile per hour. What was the average speed, in miles per hour, of
the elephant for the 14 miles?
* Solution using Xiggi’s formula:
2(4)(1)
Average Speed = 4+1
= 8/5 or 1.6

Note: The following simple formula can be used to find an average speed
when two individual speeds for the same distance are known.
2(Speed 1)(Speed 2)
Average Speed 
Speed 1  Speed 2

I call the above formula Xiggi’s formula (it is more commonly known as
the Harmonic Mean formula).

126
[Link]

Solution using a “distance = rate · time chart”: Let’s put the given
information into the following chart:
Distance Rate Time
st 7
1 part of trip 7 4 4
7
2nd part of trip 7 1 1
=7
̅̅̅̅̅̅̅̅̅̅̅̅̅̅̅̅̅̅̅̅̅̅̅̅̅̅̅̅̅̅̅̅̅̅̅̅̅̅̅̅̅̅̅̅̅̅̅̅̅̅̅̅̅̅̅̅̅̅̅̅̅̅̅̅̅̅̅̅̅̅̅
total 14 8.75

Note that we computed the times by using “distance = rate · time” in the
distance
form “time = rate
.” Finally, we use the formula in the form

distance 14
rate = = = 1.6.
time 8.75

Note: To get the total distance we add the two distances, and to get the
total time we add the two times. Be careful – this doesn’t work for rates!

LEVEL 5: GEOMETRY AND TRIG


103. If 0 ≤ 𝑥 ≤ 360°, tan 𝑥 < 0 and cos 𝑥 tan 𝑥 > 0, then which of
the following is a possible value for 𝑥 ?
(A) 30°
(B) 150°
(C) 210°
(D) 330°

* tan 𝑥 < 0 in Quadrants II and IV. Since cos x tan 𝑥 > 0 we must have
cos 𝑥 < 0 . This is true in Quadrants II and III. So x must be in Quadrant
II and therefore 90° < 𝑥 < 180°. So the answer is choice (B).
Note: Many students find it helpful to remember the following diagram.

127
[Link]

This diagram tells us which trig functions are positive in which


quadrants. The A stands for “all” so that all trig functions are positive in
the first quadrant. Similarly S stands for “sine,” T stands for “tangent,”
and C stands for “cosine.”
So, for example, if angle x is in the second quadrant, then sin 𝑥 > 0,
tan 𝑥 < 0 and cos 𝑥 < 0.
104. For 0 < 𝑥 < 90°,
tan 𝑥 − tan(−𝑥) + sin 𝑥 − sin(−𝑥) + cos 𝑥 − cos(−𝑥) =
(A) 0
(B) 3
(C) 2 tan 𝑥
(D) 2 tan 𝑥 + 2 sin 𝑥
* cos 𝑥 is an even function, so that cos(−𝑥) = cos 𝑥. Also, sin 𝑥 and
tan 𝑥 are odd functions, so sin(−𝑥) = − sin 𝑥 and tan(−𝑥) = −tan 𝑥. So
we get
tan 𝑥 − tan(−𝑥) + sin 𝑥 − sin(−𝑥) + cos 𝑥 − cos(−𝑥)
= tan 𝑥 + tan 𝑥 + sin 𝑥 + sin 𝑥 + cos 𝑥 − cos 𝑥
= 2 tan 𝑥 + 2 sin 𝑥
This is choice (D).
Negative Identities: These identities are just restating what was already
described above.
𝐜𝐨𝐬(−𝑨) = 𝐜𝐨𝐬 𝑨 𝐬𝐢𝐧(−𝑨) = −𝐬𝐢𝐧 𝑨
𝐭𝐚𝐧(−𝑨) = −𝐭𝐚𝐧 𝑨
105. It is given that cos 𝑥 = 𝑘, where 𝑘 is the radian measure of an
3𝜋
angle and 𝜋 < 𝑥 < 2 . If cos 𝑧 = −𝑘, which of the following
could not be the value of 𝑧 ?
(A) 𝑥−𝜋
(B) 𝜋−𝑥
(C) 2𝜋 − 𝑥
(D) 3𝜋 − 𝑥
* Solution using coterminal angles and a negative identity:
cos(2𝜋 − 𝑥) = cos(𝑥 − 2𝜋) = cos 𝑥 = 𝑘 ≠ −𝑘.

128
[Link]

So 2𝜋 − 𝑥 could not be the value of 𝑧, choice (C).


Notes: (1) For the first equality we used the negative identity
cos(−𝐴) = cos 𝐴
(see problem 104), together with the fact that 𝑥 − 2𝜋 = −(2𝜋 − 𝑥).
(2) In general we have 𝑎 − 𝑏 = −(𝑏 − 𝑎). To see this simply distribute:
−(𝑏 − 𝑎) = −𝑏 + 𝑎 = 𝑎 − 𝑏.
(3) Using notes (1) and (2) together, we have
cos(2𝜋 − 𝑥) = cos(−(𝑥 − 2𝜋)) = cos(𝑥 − 2𝜋).
(4) For the second equality we used the fact that 𝑥 and 𝑥 − 2𝜋 are
coterminal angles.
If 𝑎 and 𝑏 are coterminal angles, then cos 𝑎 = cos 𝑏.
(5) Given an angle 𝑥, we get a coterminal angle by adding or subtracting
any integer multiple of 2𝜋. So the following are all coterminal with 𝑥:
… 𝑥 − 4𝜋, 𝑥 − 2𝜋, 𝑥, 𝑥 + 2𝜋, 𝑥 + 4𝜋, …
Solution using the cosine difference identity:
𝐜𝐨𝐬(𝑨 − 𝑩) = 𝐜𝐨𝐬 𝑨 𝐜𝐨𝐬 𝑩 + 𝐬𝐢𝐧 𝑨 𝐬𝐢𝐧 𝑩
Let’s start with choice (C), and apply the difference identity:
cos(2𝜋 − 𝑥) = cos 2𝜋 cos 𝑥 + sin 2𝜋 sin 𝑥
= 1 ⋅ cos 𝑥 + 0 ⋅ sin 𝑥 = cos 𝑥 = 𝑘 ≠ −𝑘.
So 2𝜋 − 𝑥 could not be the value of 𝑧, choice (C).
Notes: (1) Choice (C) is always a good choice to start with when
plugging in.
(2) Let’s apply the difference formula to the other answer choices as well:
cos(𝑥 − 𝜋) = cos 𝑥 cos 𝜋 + sin 𝑥 sin 𝜋
= cos 𝑥 (−1) + sin 𝑥 (0) = − cos 𝑥 = −𝑘.
This shows that 𝑧 can be 𝑥 − 𝜋, and so we can eliminate choice (A).

129
[Link]

For 𝜋 − 𝑥, we can use the difference identity again, or we can use the
negative identity as we did in the last solution to get
cos(𝜋 − 𝑥) = cos(𝑥 − 𝜋) = −𝑘
This shows that 𝑧 can also be 𝜋 − 𝑥, and so we can eliminate choice (B).
cos(3𝜋 − 𝑥) = cos 3𝜋 cos 𝑥 + sin 3𝜋 sin 𝑥
= (−1) cos 𝑥 + 0 ⋅ sin 𝑥 = − cos 𝑥 = −𝑘.
This shows that 𝑧 can also be 3𝜋 − 𝑥, and we can eliminate choice (D).
(3) 3𝜋 = 𝜋 + 2𝜋, and so 𝜋 and 3𝜋 are coterminal angles. It follows that
cos 3𝜋 = cos 𝜋 = −1 and sin 3𝜋 = sin 𝜋 = 0.
3𝜋
Solution using the unit circle: Since 𝜋 < 𝑥 < 2 , when 𝑥 is placed in
standard position, its terminal side falls in the third quadrant, and cos 𝑥
will be the 𝑥-coordinate of the point where the terminal side intersects the
unit circle.

Since 𝑥 is a third quadrant angle, it’s reference angle is the first quadrant
angle 𝑥 − 𝜋, and so from the figure above we see that cos(𝑥 − 𝜋) = −𝑘.
The corresponding second quadrant angle is then 𝜋 − (𝑥 − 𝜋) = 2𝜋 − 𝑥,
and so we see from the figure that cos(2𝜋 − 𝑥) = 𝑘 ≠ −𝑘, and so 𝑧
cannot equal 2𝜋 − 𝑘, choice (C).
Notes: (1) If 𝜃 is a first quadrant angle, then the corresponding angle in
the second quadrant is 𝜋 − 𝜃.
The corresponding angle in the third quadrant is 𝜋 + 𝜃.
And the corresponding angle in the fourth quadrant is 2𝜋 − 𝜃.

130
[Link]

So in this problem, the first quadrant angle is 𝑥 − 𝜋, the corresponding


second quadrant angle is 𝜋 − (𝑥 − 𝜋) = 𝜋 − 𝑥 + 𝜋 = 2𝜋 − 𝑥, the
corresponding third quadrant angle is (𝑥 − 𝜋) + 𝜋 = 𝑥, and the fourth
quadrant angle is 2𝜋 − (𝑥 − 𝜋) = 2𝜋 − 𝑥 + 𝜋 = 3𝜋 − 𝑥.
(2) From the last note, and the picture above we have cos(𝑥 − 𝜋) = −𝑘,
cos(2𝜋 − 𝑥) = 𝑘, cos 𝑥 = 𝑘, and cos(3𝜋 − 𝑥) = −𝑘.
𝑥 2 + 𝑦 2 + 4𝑥 − 10𝑦 − 20 = 0
106. The graph of the equation above is a circle with center (ℎ, 𝑘)
and radius 𝑟. Compute 𝑟 ⋅ (𝑘 − ℎ).
* Solution by completing the square: Let’s first rewrite the given
equation as follows:
(𝑥 2 + 4𝑥) + (𝑦 2 − 10𝑦) = 20
Note that we regrouped the terms so that all terms involving 𝑥 are
together, all terms involving 𝑦 are together, and the constant has been
moved to the right hand side.
We now take half of 4 which is 2, and square this number to get 4.
We also take half of −10 which is −5, and square this number to get 25.
We add each of these numbers to both sides of the equation, grouping
accordingly.
(𝑥 2 + 4𝑥 + 4) + (𝑦 2 − 10𝑦 + 25) = 20 + 4 + 25
We now factor each of the expressions in parentheses on the left hand
side, and simplify on the right hand side.
(𝑥 + 2)2 + (𝑦 − 5)2 = 49
We see that ℎ = −2, 𝑘 = 5, and 𝑟 = 7.
So 𝑟 ⋅ (𝑘 − ℎ) = 7(5 − (−2)) = 7 ⋅ 7 = 𝟒𝟗.
Notes: (1) The given equation has a graph which is a circle, a point, or
empty. On the SAT it will most likely be a circle.
(2) The general form for the equation of a circle is
𝑥 2 + 𝐵𝑥 + 𝑦 2 + 𝐶𝑦 + 𝐷 = 0.
This form is not that useful for identifying specific information about the
circle such as its center and radius.

131
[Link]

(3) The standard form for the equation of a circle is


(𝑥 − ℎ)2 + (𝑦 − 𝑘)2 = 𝑟 2
In this form, we can identify the center as (ℎ, 𝑘) and the radius as 𝑟.
(4) To change an equation of a circle from general form to standard form,
we use a procedure called completing the square.
To complete the square on the expression 𝑥 2 + 𝑏𝑥, we take half of the
number 𝑏, and square the result to get 𝑏 2 .
For example, to complete the square on 𝑥 2 + 4𝑥, we half 4 to get 2, and
then square 2 to get 22 = 4.
We then add this to the original expression to get 𝑥 2 + 4𝑥 + 4. This new
expression is a perfect square. In fact, it factors as follows:
𝑥 2 + 4𝑥 + 4 = (𝑥 + 2)(𝑥 + 2) = (𝑥 + 2)2
Note that the number 2 is the same as the number we got from taking half
of 4. This is not a coincidence. It always happens.
When we complete the square on 𝑦 2 − 10𝑦, we half −10 to get −5, and
then square −5 to get (−5)2 = (−5)(−5) = 25.
We then add this to the original expression to get 𝑦 2 − 10𝑦 + 25. This
new expression is also a perfect square. In fact, it factors as follows:
𝑦 2 − 10𝑦 + 25 = (𝑦 − 5)(𝑦 − 5) = (𝑦 − 5)2
Notice again the number −5 appearing in this final expression.
(5) Completing the square does not produce an expression that is
equivalent to the original expression. For example, the expression
(𝑥 + 2)2 = 𝑥 2 + 4𝑥 + 4 is 4 more than the original expression 𝑥 2 + 4𝑥.
We fix this problem by adding the same quantity (in this case 4) to the
other side of the equation.
(6) Once the equation is in the standard form
(𝑥 + 2)2 + (𝑦 − 5)2 = 49
we can easily pick out the center and radius by matching the equation up
with the standard form
(𝑥 − ℎ)2 + (𝑦 − 𝑘)2 = 𝑟 2 .

132
[Link]

Observe that ℎ = −2, 𝑘 = 5, and 𝑟 = 7.


(7) It is very common for students to make sign errors here. Note that the
expression (𝑦 − 5)2 indicates that 𝑘 = 5, whereas the expression
(𝑥 + 2)2 indicates that ℎ = −2.
2
To see this, note that (𝑥 + 2)2 = (𝑥 − (−2)) .
(8) Another common mistake is to say that 𝑟 = 49. This is not true.
𝑟 2 = 49, so that 𝑟 = 7.

107. * In the figure above, 𝐴𝐵 is the arc of a circle with center 𝑂. If


the length of arc 𝐴𝐵 is 4𝜋, what is the area of region 𝑂𝐴𝐵 to the
nearest tenth?
50 4𝜋
* We first find the circumference of the circle using the ratio = .
360 𝐶
1440𝜋 144𝜋
Cross multiplying gives 50C = 1440π, so C = = .
50 5
144𝜋 72
Since C = 2πr, we have 2πr = 5
, so r = 5
.
5184𝜋
The area of the circle is A = πr2 = 25
.
50 𝑎
Now we find the area of the sector using the ratio 360 = (5184𝜋)/25.

Cross multiplying gives us 360a = 10,368π.


10,368𝜋
So a = 360
≈ 90.478. To the nearest tenth this is 90.5.

Notes: Consider the circle in the following figure:

133
[Link]

(1) Notice that ̅̅̅̅


𝑂𝐴 and ̅̅̅̅
𝑂𝐵 are both radii of the circle. So 𝑂𝐴 = 𝑂𝐵 = 𝑟.
If we know the radius 𝑟, then we can find the diameter 𝑑 of the circle, the
circumference 𝐶 of the circle, and the area 𝐴 of the circle. Indeed,
𝑑 = 2𝑟, 𝐶 = 2𝜋𝑟, and 𝐴 = 𝜋𝑟 2 .
In fact, if we know any one of the four quantities we can find the other
three. For example, if we know that the area of a circle is 𝐴 = 9𝜋, then it
follows that 𝑟 = 3, 𝑑 = 6, and 𝐶 = 6𝜋.
(2) Suppose that in addition to the radius 𝑟, we know the angle 𝑥. We can
then use the following ratio to find the length 𝑠 of arc 𝐴𝐵.
𝑥 𝑠
=
360 𝐶
For example, if we are given that 𝑟 = 5 and 𝑥 = 45, then we have
45 𝑠
=
360 10𝜋
450𝜋 5𝜋
So 360𝑠 = 450𝜋, and therefore 𝑠 = 360
= 4
.

In this particular example we can use a little shortcut. Just note that a 45
1
degree angle gives 8 of the total degree measure of the circle, and
1 10𝜋 5𝜋
therefore the arc length is of the circumference. So 𝑠 = = .
8 8 4

(3) We can also use the following ratio to find the area 𝑎 of sector 𝐴𝑂𝐵.
𝑥 𝑎
=
360 𝐴
For example, if again we are given that 𝑟 = 5 and 𝑥 = 45, then we have

134
[Link]

45 𝑎
=
360 25𝜋
1125𝜋 25𝜋
So 360𝑎 = 1125𝜋, and therefore 𝑎 = 360
= 8
.

Again, we can take a shortcut in this example and just divide the area of
25𝜋
the circle by 8 to get 𝑎 = 8 .

108. * A homeowner drew a sketch of their triangle-shaped garden as


shown above. Although the sketch was not drawn accurately to
scale, the triangle was labelled with the proper dimensions.
What is the value of sin 𝑥 ?
* Since two of the angles of the triangle have equal measure, the triangle
is isosceles. It follows that the median and altitude from the vertex of the
triangle are the same.

In the figure on the left we drew the altitude from the vertex angle. Since
this is also the median, we get a right triangle with a leg of length
20
2
= 10 ft.

135
[Link]

We can now find the length of the other leg by using the Pythagorean
Theorem, or better yet, by noticing that we have a multiple of the
Pythagorean triple 5, 12, 13. Since 10 = 5 ⋅ 2 and 26 = 13 ⋅ 2, it follows
that the length of the other leg is 12 ⋅ 2 = 24 ft.
OPP 24 𝟏𝟐
Finally, we have sin 𝑥 = HYP = 26 = 𝟏𝟑 ≈ . 𝟗𝟐𝟑

Note: See problem 60 for more information on medians and altitudes in


isosceles triangles.

LEVEL 5: PASSPORT TO ADVANCED MATH


9𝑥 2 25
109. If the expression 3𝑥+5 is written in the equivalent form 3𝑥+5 + 𝑘,
what is 𝑘 in terms of 𝑥 ?
(A) 9𝑥 2
(B) 9𝑥 2 + 5
(C) 3𝑥 − 5
(D) 3𝑥 + 5
9𝑥 2 25
* Quick algebraic solution: We have 3𝑥+5 = 3𝑥+5 + 𝑘, so that
9𝑥 2 25 9𝑥 2 −25 (3𝑥+5)(3𝑥−5)
𝑘 = 3𝑥+5 − 3𝑥+5 = 3𝑥+5
= 3𝑥+5
= 3𝑥 − 5, choice (C).

110. The figures above show the graphs of the functions 𝑓 and 𝑔. The
function 𝑓 is defined by 𝑓(𝑥) = 2|𝑥 + 2| and the function 𝑔 is
defined by 𝑔(𝑥) = 𝑓(𝑥 + ℎ) + 𝑘, where ℎ and 𝑘 are constants.
What is the value of |ℎ − 𝑘|?

136
[Link]

* Notice that to get the graph of 𝑔 we shift the graph of 𝑓 3 units to the
right, and 2 units up. Therefore 𝑔(𝑥) = 𝑓(𝑥 − 3) + 2. So ℎ = −3 and
𝑘 = 2. Therefore |ℎ – 𝑘| = | − 3 − 2| = | − 5| = 𝟓.
Notes: (1) To translate the graph of a function up 2 units we add 2 to the
original function.
(2) To translate the graph of a function right 3 units we replace 𝑥 by
𝑥 − 3 in the original function.
Review of basic transformations: Let 𝑦 = 𝑓(𝑥), and 𝑘 > 0. We can
move the graph of 𝑓 around by applying the following basic
transformations.
𝑦 = 𝑓(𝑥) + 𝑘 shift up k units
𝑦 = 𝑓(𝑥) − 𝑘 shift down k units
𝑦 = 𝑓(𝑥 − 𝑘) shift right k units
𝑦 = 𝑓(𝑥 + 𝑘) shift left k units
𝑦 = −𝑓(𝑥) reflect in x-axis
𝑦 = 𝑓(−𝑥) reflect in y-axis.
111. Let the function 𝑞 be defined by 𝑞(𝑥) = 𝑎(𝑥 − ℎ)2 , where ℎ is a
positive constant, and 𝑎 is a negative constant. For what value of
𝑥 will the function 𝑞 have its maximum value?
(A) −ℎ
(B) −𝑎
(C) 𝑎
(D) ℎ
Solution by picking numbers: Let’s substitute ℎ = 1 and 𝑎 = −1, so
that 𝑞(𝑥) = −(𝑥 − 1)2 . If we put this in our graphing calculator we see
that the maximum occurs when 𝑥 = 𝟏. Substituting our chosen values for
ℎ and a into each answer choice yields
(A) −1
(B) 1
(C) −1
(D) 1
We can therefore eliminate choices (A) and (C). Let’s try changing ℎ to
2, and leaving 𝑎 = −1, so that 𝑞(𝑥) = −(𝑥 − 2)2 . Again checking this in
our graphing calculator we get 𝑥 = 𝟐. Substituting the new values for ℎ
and a into answer choices (B) and (D) yields

137
[Link]

(B) 1
(D) 2
Choice (B) does not give the correct answer this time so we can eliminate
it. Thus, the answer is choice (D).
Remark: If we were a bit more careful about how we picked our original
numbers we could have avoided having to do it twice. It is actually easy
to see that choosing ℎ = 1 and 𝑎 = −1 will make some of the answer
choices come out to the same number.
Finding the maximum without a graphing calculator: Let’s use our
second guess (since it turned out to be better).
Method 1: The standard form for a quadratic equation is
𝒚 − 𝒌 = 𝒂(𝒙 − 𝒉)𝟐 or equivalently 𝒚 = 𝒂(𝒙 − 𝒉)𝟐 + 𝒌
The graph is a parabola with vertex at (ℎ, 𝑘). The parabola opens
upwards if 𝑎 > 0, and downwards if 𝑎 < 0.
𝑞(𝑥) = −(𝑥 − 2)2 is the standard form for a quadratic function whose
graph is a parabola that opens downwards and has vertex (2, 0). Thus the
maximum occurs at 𝑥 = 2.
Note: In this problem, ℎ = 2 and 𝑘 = 0. We substituted these values into
the standard form for a quadratic equation (either version) and we got
𝑞(𝑥) = −(𝑥 − 2)2 .
Method 2: The general form for a quadratic equation is
𝒚 = 𝒂𝒙𝟐 + 𝒃𝒙 + 𝒄
𝑏
The graph is a parabola whose vertex has 𝑥-coordinate − . The
2𝑎
parabola opens upwards if 𝑎 > 0, and downwards if 𝑎 < 0.
𝑞(𝑥) = −(𝑥 − 2)2 = −(𝑥 2 − 4𝑥 + 4) = −𝑥 2 + 4𝑥 − [Link]
𝑏 4
− 2𝑎 = − 2(−1) = 2.

Thus the maximum occurs at 𝑥 = 2 (it’s a maximum because the


parabola opens downwards).
Method 3: For those who know calculus we can use a derivative. The
derivative of 𝑞(𝑥) = −(𝑥 − 2)2 is 𝑞 ′ (𝑥) = −2(𝑥 − 2). This is zero
when 𝑥 = 2.

138
[Link]

The derivative is positive for 𝑥 < 2 and negative for 𝑥 > 2 which means
that the function is increasing for 𝑥 < 2 and decreasing for 𝑥 > 2. Thus,
there is a maximum (both relative and absolute) at 𝑥 = 2.
Note: Any of these last 3 methods can be used from the original equation
(without picking numbers). For example, using the first method we have
the following:
* 𝑞(𝑥) = 𝑎(𝑥 − ℎ)2 is in standard form and thus has a graph that is a
parabola with (ℎ, 0) for its vertex. Since 𝑎 < 0 the parabola opens
downwards. Thus the maximum occurs when 𝑥 = ℎ, choice (D).
1 𝑥−6 𝑥+5
112. Give one possible solution to the equation 𝑥 2 +𝑥 − 𝑥+1 = 𝑥 2 +𝑥 .

* Algebraic solution: 𝑥 2 + 𝑥 = 𝑥(𝑥 + 1) is the lcd of the fractions that


appear in the equation. So let’s begin by multiplying each term by
𝑥(𝑥 + 1):
1 𝑥−6 𝑥+5
𝑥(𝑥 + 1) ⋅ − 𝑥(𝑥 + 1) ⋅ = 𝑥(𝑥 + 1) ⋅ 2
𝑥2 +𝑥 𝑥+1 𝑥 +𝑥
1 − 𝑥(𝑥 − 6) = 𝑥 + 5
1 − 𝑥 2 + 6𝑥 = 𝑥 + 5
0 = 𝑥 2 − 5𝑥 + 4
0 = (𝑥 − 1)(𝑥 − 4)
So the two possible solutions are 𝟏 or 𝟒.
Notes: (1) The lcd (least common denominator) of a set of fractions is the
lcm (least common multiple) of the denominators.
In this problem the denominators are 𝑥 2 + 𝑥 = 𝑥(𝑥 + 1) and 𝑥 + 1. So
the lcd is 𝑥 2 + 𝑥 = 𝑥(𝑥 + 1).
(2) Multiplying each side of a rational equation by the lcd of the fractions
that appear in the equation gives an equation without any fractions.
(3) Using the distributive property, we see that multiplying each side of
an equation by an expression is the same as multiplying each term by that
expression.

139
[Link]

1 1
(4) 𝑥(𝑥 + 1) ⋅ 𝑥 2 +𝑥 = 𝑥(𝑥 + 1) ⋅ 𝑥(𝑥+1) = 1
𝑥−6
−𝑥(𝑥 + 1) ⋅ = −𝑥(𝑥 − 6) = −𝑥 2 + 6𝑥
𝑥+1
𝑥+5 𝑥+5
𝑥(𝑥 + 1) ⋅ 𝑥 2 +𝑥 = 𝑥(𝑥 + 1) ⋅ 𝑥(𝑥+1) = 𝑥 + 5

(5) Remember to check that neither solution is extraneous at the end. To


do these we need only check that 1 and 4 do not make any denominator
zero in the original equation.
(6) If a graphing calculator were allowed for this question, an alternative
would be to put each side of the equation into your graphing calculator
and then to use the intersect feature (see the graphical solution to problem
73 to see how to do this).
113. If 𝑥 2 − 8𝑥 = 209, and 𝑥 < 0, what is the value of |𝑥 + 3| ?
* Algebraic solution: Let’s attempt to factor the equation. We begin by
subtracting 209 from each side to get 𝑥 2 − 8𝑥 − 209 = 0. We now factor
the left hand side to get (𝑥 − 19)(𝑥 + 11) = 0.
So 𝑥 = 19 or 𝑥 = −11. Since we are given 𝑥 < 0, we use 𝑥 = −11.
Finally we have |𝑥 + 3| = |−11 + 3| = |−8| = 𝟖.
Notes: (1) There are several ways to solve a quadratic equation. A few
are by (i) factoring, (ii) completing the square, (iii) using the quadratic
formula, (iv) guessing and checking, (v) creating a table of values in your
calculator, (vi) using the graphing features of your calculator.
(2) Whenever you solve a quadratic equation by factoring or by using the
quadratic formula, you need to bring everything over to one side of the
equation first , leaving 0 on the other side.
(3) It can seem very difficult at first to find the factors of 209, but the
following trick can help you find the factors a little easier. Since we have
152 = 225 > 209, it follows that if 209 can be factored, then it has a
factor less than 15. Furthermore, since every positive integer can be
factored as a product of primes, it follows that we need only check prime
numbers less than 15. So we can simply check 209 for divisibility by 2, 3,
5, 7, 11, and 13.
We can use standard divisibility tricks to eliminate 2, 3, and 5 right away.
You may want to try 11 next since it is pretty easy to divide by 11. In this
case we have 209 ÷ 11 = 19. So 209 = 11 ⋅ 19.

140
[Link]

If we happen to be allowed to use a calculator for this problem, then we


could divide 209 by these numbers very quickly.
(4) The zero property of the real numbers says that when you have a
product equal to zero, one of the factors must be zero.
In this problem, since (𝑥 − 19)(𝑥 + 11) = 0, we must have 𝑥 − 19 = 0
or 𝑥 + 11 = 0.
(5) Some students may actually find it easier to solve the given quadratic
equation by completing the square. In this case we do not change the
form of the equation:
𝑥 2 − 8𝑥 = 209
We take half of −8, which is −4, and square this number to get 16. We
add 16 to each side of the equation to get 𝑥 2 – 8𝑥 + 16 = 209 + 16. This
2
is equivalent to (𝑥– 4) = 225. We now apply the square root
property to get 𝑥 − 4 = ±15. So 𝑥 = 4 ± 15. This yields the two
solutions 4 + 15 = 19, and 4 – 15 = −11.
(6) We can also solve the given quadratic equation using the quadratic
formula. For this procedure we need to first subtract 209 from each side
to get 𝑥 2 − 8𝑥 − 209 = 0.
−𝒃±√𝒃𝟐 −𝟒𝒂𝒄 8±√64+836 8±√900 8±30
𝒙= = = = = 4 ± 15.
𝟐𝒂 2 2 2

As in the previous solution we get x = 19 or x = −11.


Solution by guessing and checking: If we are allowed to use our
calculator, there are several other ways to solve the given equation for 𝑥.
One way is to simply guess and check negative values for 𝑥. For
example if we substitute −3 in for 𝑥 on the left hand side of the equation
we get (−3)^2 − 8(−3) = 33, which is too small. So let’s try 𝑥 = −8
next: (−8)^2 − 8(−8) = 128. This is still too small, but we’re heading
in the right direction. We try 𝑥 = −11 to get (−11)^2 − 8(−11) = 209.
It works. So 𝑥 = −11 and we have |𝑥 + 3| = |−11 + 3| = |−8| = 𝟖.
Solution by creating a table of values: Another way to use our
calculator to solve this problem is to create a table of values. We press
the Y= button, and enter the following.
Y1 = X ^ 2 – 8X – 209

141
[Link]

Now press TABLE (which is 2ND GRAPH).


Since we want a negative value for 𝑥, we start scrolling through negative
integers until we see 0 in the Y1 column. This happens when 𝑥 = −11.
Finally we have |𝑥 + 3| = |−11 + 3| = |−8| = 𝟖.
Solution by graphing: As in the previous solution press the Y= button,
and enter the following.
Y1 = X ^ 2 – 8X – 209
This time press ZOOM 6 to graph the parabola in a standard window. Uh
oh – there’s nothing there! Don’t worry – we probably just need to zoom
out. Press the WINDOW button, and change Xmin to −100, Xmax to
100, Ymin to −100, and Ymax to 100. Then press 2nd TRACE (which is
CALC) 2 (or select ZERO).
Now move the cursor just to the left of the negative x-intercept and press
ENTER. Now move the cursor just to the right of the negative x-intercept
and press ENTER again. Press ENTER once more, and you will see that
the x-coordinate of the negative x-intercept is −11.
Finally we have |𝑥 + 3| = |−11 + 3| = |−8| = 𝟖.
Remark: The choices made for Xmax, Ymin and Ymax were just to try
to ensure that the negative x-intercept would appear in the viewing
window. Many other windows would work just as well.
114. Let 𝑥 ∴ 𝑦 be defined as the sum of all integers between 𝑥 and 𝑦.
For example, 1 ∴ 4 = 2 + 3 = 5. What is the value of
(60 ∴ 900) − (63 ∴ 898)?
* We write out each sum formally, line them up so that the numbers
match up, and subtract term by term.
61 + 62 + 63 + 64 + … + 897 + 898 + 899
64 + … + 897 _
61 + 62 + 63 + 0 + … + 0 + 898 + 899
So the answer is 61 + 62 + 63 + 898 + 899 = 1983.

142
[Link]

LEVEL 5: PROBLEM SOLVING AND DATA


115. * A survey was conducted among a randomly chosen sample of
250 single men and 250 single women about whether they
owned any dogs or cats. The table below displays a summary of
the survey results.

Dogs Cats
Both Neither Total
Only Only
Men 92 27 5 126 250
Women 75 43 34 98 250
Total 167 70 39 224 500

According to the table, which of the following statements is


most likely to be false?
(A) The probability that a woman is a cat owner is greater
than the probability that a cat owner is a woman
(B) The probability that a dog owner is male is greater than
the probability that a randomly chosen person is a cat
owner.
(C) The probability that a woman does not own a dog or cat
is greater than the probability that a man owns at least
one dog and one cat.
(D) The probability that a cat owner is a woman is greater
than the probability that a man owns a dog.
* We are being asked to use the table to compute conditional
probabilities. Let’s name the events we will be referring to in the
obvious way. For example, 𝑀 will stand for “the person is a Man,” and 𝐷
will stand for “the person owns Dogs Only.” As an example, the
probability that a person owns cats only will be written 𝑃(𝐶), and the
probability that a woman owns cats only will be written 𝑃(𝐶|𝑊). The
expression 𝑃(𝐶|𝑊) is a conditional probability and should be read “the
probability the person owns cats only given that the person is a woman,”
or more succinctly as “the probability a woman owns cats only.”

143
[Link]

Let’s start with choice (C) and first compute the probability that a woman
does not own a dog or cat. Note that we are given that the person is a
woman. So we restrict our attention to the second row (the row labeled
98
“Women”), and we see that this probability is 𝑃(𝑁|𝑊) = 250 = .392.

We next compute the probability that a man owns at least one dog and
5
one cat as 𝑃(𝐵|𝑀) = 250 = .02.

Since . 392 > .02, we can eliminate choice (C).


Let’s try choice (B) next and first compute the probability that a dog
92+5 97
owner is male. This is 𝑃(𝑀|𝐷 ∪ 𝐵) = 167+39 = 206 ≈ .47.

We next compute the probability that a person is a cat owner. This is


70+39 109
𝑃(𝐶 ∪ 𝐵) = 500 = 500 = .218.

Since . 47 > .218, we can eliminate choice (B).


Let’s try choice (A) and first compute the probability that a woman is a
43+34 77
cat owner. This is 𝑃(𝐶 ∪ 𝐵|𝑊) = 250 = 250 = .308.

We next compute the probability that a cat owner is a woman. This is


43+34 77
𝑃(𝑊|𝐶 ∪ 𝐵) = 70+39 = 109 ≈ .706.

Since . 308 < .706, the answer is choice (A).


Notes: (1) For completeness, let’s do the computations for choice (D).
The probability that a cat owner is a woman is 𝑃(𝑊|𝐶 ∪ 𝐵) ≈ .706 and
92+5
the probability that a man owns a dog is 𝑃(𝐷 ∪ 𝐵|𝑀) = 250 = .388.

(2) 𝑋 ∪ 𝑌, read “𝑋 union 𝑌” is the event consisting of all outcomes in 𝑋


or 𝑌. For example, 𝐷 ∪ 𝐵 is the event consisting of people who own dogs
only or both dogs and cats. In other words, 𝐷 ∪ 𝐵 is the event consisting
of people who own dogs.
(3) 𝐵 is the event consisting of people who own at least 1 dog and 1 cat.
(4) 𝑋 ∩ 𝑌, read “𝑋 intersect 𝑌” is the event consisting of all outcomes
common to both 𝑋 and 𝑌. For example, 𝐷 ∩ 𝑀 is the event consisting of
people who are men and own dogs, or equivalently, males that have dogs
only.

144
[Link]

(5) 𝑃(𝑋|𝑌) is the probability of 𝑋 given 𝑌, and can be computed with the
𝑃(𝑋∩𝑌)
formula 𝑃(𝑋|𝑌) = 𝑃(𝑌)
.

𝑃(𝑁∩𝑊)
For example, 𝑃(𝑁|𝑊) = 𝑃(𝑊)
.

𝑃(𝑁 ∩ 𝑊) can be found in the table by looking at the entry in the column
98
for Neither and the row for Women. So 𝑃(𝑁 ∩ 𝑊) = 500.
250
We also have 𝑃(𝑊) = 500.
𝑃(𝑁∩𝑊) 98 250 98 500 98
It follows that 𝑃(𝑁|𝑊) = 𝑃(𝑊)
= 500 ÷ 500 = 500 ⋅ 250 = 250 = .392.

In practice, we can “forget” the 500’s and just put 98 over 250.
116. Suppose that the average (arithmetic mean) of 𝑎, 𝑏, and 𝑐 is ℎ,
the average of 𝑏, 𝑐, and 𝑑 is 𝑗, and the average of 𝑑 and 𝑒 is 𝑘.
What is the average of 𝑎 and 𝑒?
(A) ℎ − 𝑗 + 𝑘
3ℎ+3𝑗−2𝑘
(B) 2
3ℎ−3𝑗+2𝑘
(C)
2
3ℎ−3𝑗+2𝑘
(D)
5

* Solution by changing averages to sums and trying a simple


operation: We have that 𝑎 + 𝑏 + 𝑐 = 3ℎ, 𝑏 + 𝑐 + 𝑑 = 3𝑗, and
𝑑 + 𝑒 = 2𝑘. If we subtract the second equation from the first, and then
add the third equation we get 𝑎 + 𝑒 = 3ℎ − 3𝑗 + 2𝑘. So the average of 𝑎
𝑎+𝑒 3ℎ – 3𝑗 + 2𝑘
and 𝑒 is 2
= 2
, choice (C).

Solution by picking numbers: Let’s choose values for 𝑎, 𝑏, 𝑐, 𝑑, and 𝑒,


say 𝑎 = 1, 𝑏 = 2, 𝑐 = 3, 𝑑 = 4, and 𝑒 = 6. Then ℎ = 2, 𝑗 = 3, 𝑘 = 5
and the average of 𝑎 and 𝑒 is 3.5. The answer choices become
(A) 4
(B) 2.5
(C) 3.5
(D) 1.4
Since (A), (B) and (D) came out incorrect, the answer is choice (C).

145
[Link]

1 1 1 2 3
, , ,𝑥 ,𝑥
𝑥3 𝑥2 𝑥
117. If −1 < 𝑥 < 0, what is the median of the five numbers in the
list above?
1
(A)
𝑥3
1
(B) 𝑥2
1
(C) 𝑥
3
(D) 𝑥
1
* Solution by picking a number: Let’s choose x = − .
2
1 1 1 1
Then we have = (−2)3 = −8, = (−2)2 = 4, = −2, 𝑥 2 = and
𝑥3 𝑥2 𝑥 4
1 3 1
𝑥3 = (− 2) = − 8.

Now let’s place them in increasing order.


1 1
−8, −2, − 8, 4, 4
1
The median is − 8 which is 𝑥 3 , choice (D).

Note: If we are allowed to use a calculator for this problem, we can do all
the computations in our calculator and get the following decimals:
1 1 1
= −8 =4 = −2 𝑥 2 = 0.25 𝑥 3 = −0.125
𝑥3 𝑥2 𝑥

Here they are in increasing order:


−8, −2, −0.125, 0.25, 4
The median is −0.125 which is 𝑥 3 , choice (D).

146
[Link]

Questions 118 - 119 refer to the following information.


A biologist places a colony consisting of 5000 bacteria into a
petri dish. After the initial placement of the bacteria at time
𝑡 = 0, the biologist measures and estimates the number of
bacteria present every half hour. This data was then fitted by an
exponential curve of the form 𝑦 = 𝑐 ⋅ 2𝑘𝑡 where 𝑐 and 𝑘 are
constants, 𝑡 is measured in hours, and 𝑦 is measured in
thousands of bacteria. The scatterplot together with the
exponential curve are shown below.

118. According to the scatterplot, the biologist’s measurements


indicate that the number of bacteria present quadrupled in 6
hours, and the exponential curve passes through the
corresponding data point at time 𝑡 = 6. The exponential
function also agrees with the initial number of bacteria.
Compute 𝑐𝑘.
* Since there are 5000 bacteria present at time 𝑡 = 0, we have 𝑐 = 5. So
𝑦 = 5 ⋅ 2𝑘𝑡 .
We are given that 𝑦 = 4 ⋅ 5 = 20 when 𝑡 = 6, so that 20 = 5 ⋅ 26𝑘 .
Dividing each side of this equation by 5 yields 4 = 26𝑘 . So we have
2 1 1
22 = 26𝑘 , and therefore 2 = 6𝑘. So 𝑘 = 6 = 3, and 𝑐𝑘 = 5 ⋅ 3 = 𝟓/𝟑.

Notes: (1) We can also grid in the decimals 1.66 or 1.67.

147
[Link]

(2) Since there are 5000 bacteria present at time 𝑡 = 0, we see that the
point (0,5) is a data point. Since the exponential function agrees with the
initial number of bacteria, we have that the point (0,5) is also on the
exponential curve. So 5 = 𝑐 ⋅ 2𝑘⋅ 0 = 𝑐. So 𝑐 = 5.
This computation is not really necessary, because 𝑐 is always the initial
amount in the exponential function 𝑦 = 𝑐 ⋅ 2𝑘𝑡 .
(3) After 6 hours, the biologist measured that 20,000 bacteria were
present. Since the exponential curve matches the data point at 6 hours, we
see that the point (6,20) is both a data point and a point on the
exponential curve.
In particular, 20 = 5 ⋅ 2𝑘⋅6 = 5 ⋅ 26𝑘 .
(4) A common mistake is to write 5 ⋅ 26𝑘 = 106𝑘 . The 5 CANNOT be
combined in any way with the 2 as this would violate the usual order of
operations.
To eliminate the 5, we divide each side of the equation by 5:
20 5⋅26𝑘
5
= 5

4 = 26𝑘
(5) The expressions 4 and 26𝑘 both have a common base of 2. Indeed,
4 = 22 . So we have 22 = 26𝑘 .
(6) When two expressions have the same base, the exponents must be
equal. In this case, since 22 = 26𝑘 , we must have 2 = 6𝑘.
119. Suppose that the data was fitted with a quadratic function of the
form 𝑡 2 + 𝑏𝑡 + 𝑐 instead of an exponential function. Assume
that the quadratic function agrees with the scatterplot at times
𝑡 = 0 and 𝑡 = 6. What is the 𝑡-coordinate of the vertex of the
graph of the quadratic function?
* Since there are 5000 bacteria present at time 𝑡 = 0, we have 𝑐 = 5. So
𝑦 = 𝑡 2 + 𝑏𝑡 + 5.
According to the scatterplot, 𝑦 = 20 when 𝑡 = 6, so that we have
20 = 62 + 6𝑏 + 5 = 36 + 6𝑏 + 5 = 41 + 6𝑏. It follows that
21 7
6𝑏 = 20 − 41 = −21, and therefore 𝑏 = − 6 = − 2.

148
[Link]

The 𝑡-coordinate of the vertex of the graph of the quadratic function is


7
−(− ) 7 7 1 𝟕
2
2⋅1
= 2 ÷ 2 = 2 ⋅ 2 = 𝟒 or 𝟏. 𝟕𝟓.

Notes: (1) Since the quadratic function agrees with the initial number of
bacteria, we have that the point (0,5) is on the quadratic curve. So
5 = 02 + 𝑏(0) + 𝑐 = 𝑐. Therefore 𝑐 = 5.
This computation is not really necessary, becasue 𝑐 is always the
𝑦-coordinate of the 𝑦-intercept of the graph of the quadratic equation.
𝑦 = 𝑎𝑡 2 + 𝑏𝑡 + 𝑐.
(3) The general form for a quadratic function is
𝒚 = 𝒂𝒙𝟐 + 𝒃𝒙 + 𝒄.
The graph of this function is a parabola whose vertex has x-coordinate
𝒃

𝟐𝒂
The parabola opens upwards if 𝑎 > 0 and downwards if 𝑎 < 0.
7
It follows that the graph of 𝑦 = 𝑡 2 − 2 𝑡 + 5 has a vertex with
7
(− ) 7
2
𝑡-coordinate − 2(1) = 4.

(4) We can also find the vertex by putting the quadratic function into
standard form by completing the square. See problem 90 for more details.
120. * John, a United States resident, is on vacation in Spain and is
trying to decide if he should use his own credit card from the
U.S., or to purchase a prepaid credit card for 500 euros in Spain.
The bank that issues John’s U.S. credit card converts all
purchase prices at the foreign exchange rate for that day, and an
additional fee of 6% of the converted cost is applied before the
bank posts the charge.
If John decides to purchase the prepaid card, he can use this card
spending dollars at the exchange rate for that day with no fee,
but he loses any money left unspent on the card.
Suppose that John does decide to buy the prepaid card. What is
the least number of the 500 euros John must spend for the
prepaid card to have been the cheaper option? Round your
answer to the nearest whole number of euros.

149
[Link]

* If we let 𝑑 be the cost of the 500 euro card in dollars, and we let 𝑒 be
John’s total purchases on the prepaid card in euros, we need
𝑑
𝑑 < 1.06 ( )𝑒
500
1.06
We can divide each side of this inequality by 𝑑 to get 1 < 𝑒 ( 500 ), and
500 500
then multiply each side of this last equation by 1.06
to get 1.06
< 𝑒, or
equivalently 𝑒 > 471.698 .
So the least number of the 500 euros John must spend is 𝟒𝟕𝟐.
Notes: (1) Since John is paying 𝑑 dollars for the prepaid 500 euro card, it
𝑑
follows that the exchange rate on this particular day is 500.

(2) Since John’s total purchases on the prepaid card was 𝑒 euros, it
𝑑
follows that John spent ( ) 𝑒 dollars.
500

(3) If John spent the 𝑒 euros on his U.S. credit card instead, he would
𝑑
have paid 1.06 (500) 𝑒 dollars because of the extra 6% fee.
𝑑
(4) So 𝑑 is the cost of the 500 euro prepaid card, and 1.06 ( ) 𝑒 is the
500
cost in dollars of spending 𝑒 euros using the U.S. credit card.
Since we want the cost of the prepaid card to be cheaper, we have the
𝑑
inequality 𝑑 < 1.06 ( ) 𝑒.
500

(5) Since 𝑑 is a factor on each side of the inequality, we can divide each
side by 𝑑 to eliminate this variable. Be careful here! When performing
this type of division it is important that we know that 𝑑 cannot be zero.
1
(6) After dividing out the 𝑑’s, we are left with 1 < 1.06 (500) 𝑒, which
1.06
we can rewrite as 1 < ( 500 ) 𝑒.
1.06
(7) We can solve the inequality 1 < ( ) 𝑒 for 𝑒 by multiplying by the
500
1.06 1.06 500
reciprocal of 500
. The reciprocal of 500
is 1.06.

150
[Link]

SUPPLEMENTAL PROBLEMS
QUESTIONS

LEVEL 1: HEART OF ALGEBRA


1. A caterer is hired to provide food for a private party consisting
of 20 businessmen. She will be paid $80 per hour and an
additional $40 tip if she serves all the food on time. If the caterer
serves all the food on time, which of the following expressions
can be used to determine how much the caterer earns, in dollars?
(A) 40𝑥 + (80 + 20), where 𝑥 is the number of businessmen
(B) (80 + 20)𝑥 + 40, where 𝑥 is the number of businessmen
(C) 40𝑥 + 80, where 𝑥 is the number of hours
(D) 80𝑥 + 40, where 𝑥 is the number of hours
2. If Edna drove 𝑠 miles in 𝑡 hours, which of the following
represents her average speed, in miles per hour?
𝑠
(A)
𝑡
𝑡
(B)
𝑠
1
(C) 𝑠𝑡
(D) 𝑠𝑡

3. A supermarket sells protein cookies individually and in packs of


12. During a certain week, the supermarket sold a total of 315
protein cookies, of which 51 were sold individually. Which
equation shows the number of packs of cookies sold during that
week?
315
(A) 12
+ 51
315
(B) − 51
12
315+51
(C) 12
315−51
(D) 12

151
[Link]

4. Which of the following mathematical expressions is equivalent


to the verbal expression “A number, 𝑐, squared is 52 more than
the product of 𝑐 and 11”?
(A) 2𝑐 = 52 + 11𝑐
(B) 2𝑐 = 52𝑐 + 11𝑐
(C) 𝑐 2 = 52 − 11𝑐
(D) 𝑐 2 = 52 + 11𝑐
3𝑥
5. For what value of 𝑥 is − 11 = 16 ?
7

6. If 18𝑥 + 42𝑦 = 66, what is the value of 3𝑥 + 7𝑦?

LEVEL 1: GEOMETRY AND TRIG


7. In the standard (𝑥, 𝑦) coordinate plane, what is the slope of the
line segment joining the points (3,–5) and (7,2) ?
7
(A) − 4
3
(B) − 4
3
(C)
4
7
(D) 4

8. How many figures of the size and shape above are needed to
completely cover a rectangle measuring 60 inches by 40 inches?
(A) 400
(B) 600
(C) 800
(D) 1000

152
[Link]

9. In the figure above, one side of a triangle is extended. Which of


the following is true?
(A) 𝑦 = 75
(B) 𝑧 = 75
(C) 𝑧 − 𝑦 = 75
(D) 𝑦 + 𝑧 = 75

10. The dimensions of the right triangle above are given in meters.
What is tan B ?
𝑐
(A) 𝑏
𝑎
(B) 𝑏
𝑎
(C) 𝑐
𝑏
(D) 𝑎

153
[Link]

11. In ∆𝑃𝑄𝑅 above, which of the following trigonometric


12
expressions has value ?
13

(A) tan 𝑃
(B) cos 𝑃
(C) sin 𝑅
(D) cos 𝑅

12. What is the perimeter of polygon 𝑃𝑄𝑅𝑆𝑇 shown above, in


inches?

LEVEL 1: PASSPORT TO ADVANCED MATH


13. For the function 𝑓(𝑥) = 5𝑥 2 − 7𝑥, what is the value of 𝑓(−3)?
(A) −66
(B) −24
(C) 24
(D) 66

154
[Link]

14. If 3𝑥 2 + 9𝑥 = 84, what are the possible values for 𝑥 ?


(A) −4 and 7
(B) −7 and 4
(C) −7 and −4
(D) −7 and −12
15. A rectangle has area 𝐴, length 𝑙, and width 𝑤. Which of the
following represents 𝑤 in terms of 𝐴 and l ?
𝐴
(A) 𝑤 = 2𝑙
𝐴
(B) 𝑤 = 𝑙
2𝐴
(C) 𝑤 = 𝑙
√𝐴
(D) 𝑤 =
𝑙
1
16. For which nonnegative value of 𝑏 is the expression 2−𝑏2
undefined?
(A) 0
(B) √2
(C) 2
(D) 8
𝑓(𝑥) = 7𝑥 − 3
𝑔(𝑥) = 𝑥 2 − 2𝑥 + 6
17. The functions 𝑓 and 𝑔 are defined above. What is the value of
𝑓(11) − 𝑔(4)?

𝑥 𝑝(𝑥) 𝑞(𝑥) 𝑟(𝑥)


1 4 5 9
2 –2 6 –9
3 –5 –9 4
4 –3 –10 –7
5 –5 0 –5

18. The table above gives some values of the functions 𝑝, 𝑞, and 𝑟.
At which value 𝑥 does 𝑞(𝑥) = 𝑝(𝑥) + 𝑟(𝑥)?

155
[Link]

LEVEL 1: PROBLEM SOLVING AND DATA

19. Ten 25 year old men were asked how many hours per week they
exercise and their resting heart rate was taken in beats per
minute (BPM). The results are shown as points in the scatterplot
above, and the line of best fit is drawn. What is the resting heart
rate, in BPM, of the man represented by the data point that is
farthest from the line of best fit?
(A) 60
(B) 66
(C) 67
(D) 75

156
[Link]

20. A survey was conducted among a randomly chosen sample of


100 males and 100 females to gather data on family size. The
data are shown in the table below.

Do not
Have
have Total
siblings
siblings
Men 75 25 100
Women 63 37 100
Total 138 62 200

Which of the following is closest to the percent of those


surveyed who have siblings?
(A) 31
(B) 63
(C) 69
(D) 75
21. For which of the following lists of 5 numbers is the average
(arithmetic mean) greater than the median?
(A) 3, 3, 5, 8, 8
(B) 2, 3, 5, 6, 7
(C) 3, 3, 5, 7, 7
(D) 3, 4, 5, 6, 7
22. A 770 gallon tank is filled to capacity with water. At most how
many 14 ounce bottles can be filled with water from the tank? (1
gallon = 128 ounces)
23. Running at a constant speed, an antelope traveled 150 miles in 6
hours. At this rate, how many miles did the antelope travel in 5
hours?
24. What is the median of the following 9 test grades?
95, 72, 81, 96, 62, 98, 82, 76, 82

157
[Link]

LEVEL 2: HEART OF ALGEBRA


𝑥+48
25. Which of the following is equal to 12
?
𝑥+24
(A) 6
(B) 𝑥 + 4
(C) 4𝑥
𝑥
(D) 12
+4

1 1
𝑥− 𝑦=7
3 6
1 1
𝑦− 𝑥=8
5 5
26. Which of the following ordered pairs(𝑥, 𝑦) satisfies the system
of equations above?
(A) (−36, −57)
(B) (12,43)
101 307
(C) ( , )
5 5
(D) (82,122)
𝑇 = 25 + 3𝑐
27. The equation above is used to model the number of chirps, 𝑐,
made by a certain species of cricket in one minute, and the
temperature, 𝑇, in degrees Fahrenheit. According to this model,
what is the meaning of the number 3 in the equation.
(A) If a cricket chirps three more times in one minute, then
the temperature, in Fahrenheit, will be one degree higher.
(B) If a cricket chirps three fewer times in one minute, then
the temperature, in Fahrenheit, will be one degree higher.
(C) If a cricket chirps one more time in one minute, then the
temperature, in Fahrenheit, will be three degrees higher.
(D) If a cricket chirps one fewer time in one minute, then the
temperature, in Fahrenheit, will be three degrees higher.

158
[Link]

28. Gina subscribes to a cell phone service that charges a monthly


fee of $60.00. The first 500 megabytes of data is free, and the
cost is $0.15 for each additional megabyte of data used that
month. Which of the following functions gives the cost, in
dollars, for a month in which Gina uses 𝑥 megabytes of data,
where 𝑥 > 500.
(A) 60 + 15𝑥
(B) 0.15𝑥 − 15
(C) 0.15𝑥 − 440
(D) 60 + 0.15𝑥
50𝑡 + 3𝑐 = 300
29. Robert is playing blackjack at a casino. The equation above can
be used to model the number of chips, 𝑐, that Robert still has in
his possession 𝑡 hours after he begins playing. What does it
mean that 𝑡 = 0, 𝑐 = 100 is a solution to this equation?
(A) Robert is losing 10 chips per hour.
(B) It would take 100 hours for Robert to have 300 chips.
(C) Robert can play for 100 hours before losing all his chips.
(D) Robert begins playing with 100 chips.
30. At a pet store, each frog is priced at $1 and each salamander is
priced at $8. Jeff purchased 14 amphibians at the store for a total
price of $42. How many frogs did Jeff purchase?

LEVEL 2: GEOMETRY AND TRIG


31. If each base of a trapezoid is reduced by 50% and the height of
the trapezoid is quadrupled, how would the area of the trapezoid
change?
(A) The area would be multiplied by 4.
(B) The area would be multiplied by 2.
(C) The area would not change.
(D) The area would be cut in half.

159
[Link]

32. In the figure above, line l is parallel to line 𝑘. Transversals 𝑚


and 𝑛 intersect at point 𝑃 on l and intersect 𝑘 at points 𝑅 and 𝑄,
respectively. Point 𝑌 is on 𝑘, the measure of ∠𝑃𝑅𝑌 is 140°, and
the measure of ∠𝑄𝑃𝑅 is 100°. How many of the angles formed
by rays l, 𝑘, 𝑚, and 𝑛 have measure 40° ?
(A) 4
(B) 6
(C) 8
(D) 10
33. In the 𝑥𝑦-plane, the point (0, 2) is the center of a circle that has
radius 2. Which of the following is NOT a point on the circle?
(A) (0, 4)
(B) (−2, 4)
(C) (−2, 2)
(D) (0, 0)

𝑡
34. Given Δ𝐶𝐴𝑇 above, which of the following is equal to 𝑐 ?

(A) cos 𝐶
(B) cos 𝑇
(C) tan 𝐶
(D) tan 𝑇

160
[Link]

35. The figure above shows a 14-foot ramp forms an angle of 63°
with the vertical wall it is leaning against. Which of the
following is an expression for the horizontal length, in feet, of
the ramp?
(A) 14 cos 63°
(B) 14 sin 63°
(C) 14 tan 63°
(D) 14 cot 63°

36. The figure above shows a right triangle whose hypotenuse is 7


feet long. How many feet long is the longer leg of this triangle?
(A) 3.5
(B) 14
7√3
(C) 2
7√3
(D)
6

LEVEL 2: PASSPORT TO ADVANCED MATH


37. The expression (3𝑏 − 2)(𝑏 + 5) is equivalent to:
(A) 3𝑏 2 − 7
(B) 3𝑏 2 − 10
(C) 3𝑏 2 − 2𝑏 − 7
(D) 3𝑏 2 + 13𝑏 − 10

161
[Link]

𝑥 11
38. The value of 𝑥 that will make 3 − 2 = − 4 a true statement lies
between which of the following numbers?
(A) −3 and −2
(B) −2 and −1
(C) −1 and 0
(D) 0 and 1
39. If 𝑔(𝑥 − 3) = 5𝑥 + 1 for all values of 𝑥, what is the value of
𝑔(−2) ?
40. What is the value of 𝑐 − 2 if (3𝑐 − 7) − (3 − 2𝑐) = 5 ?
2𝑏2 −8𝑎 2
41. The operation ∎ is defined as 𝑎 ∎ 𝑏 = 𝑏+2𝑎 where 𝑎 and 𝑏
are real numbers and 𝑏 ≠ −2𝑎. What is the value of
(−2) ∎ (−1) ?
42. Let a function of 2 variables be defined by
𝑔(𝑥, 𝑦) = 𝑥𝑦 + 3𝑥𝑦 2 − (𝑥 − 𝑦 2 ), what is the value of
𝑔(2, −1)?

LEVEL 2: PROBLEM SOLVING AND DATA


43. Exactly 52% of the marbles in a jar are red. Which of the
following could be the total number of marbles in the jar?
(A) 20
(B) 21
(C) 25
(D) 28

162
[Link]

44. The line graph above shows the average number of days that it
snows at least 0.1 inch in Buffalo, NY from January to May.
According to the graph, approximately what was the greatest
decrease in the number of snow days from one month to the next
month?
(A) 2
(B) 3
(C) 4
(D) 6

45. The histogram above shows the distribution of the weights, in


pounds, of 18 cats in a shelter. Which of the following could be
the median weight of the 18 cats represented in the histogram?
(A) 10 pounds
(B) 11 pounds
(C) 13.5 pounds
(D) 16 pounds

163
[Link]

46. Jeff has taken 6 of 10 equally weighted math tests this semester,
and he has an average score of exactly 82.0 points. How many
points does he need to earn on the 7th test to bring his average
score up to exactly 83 points?
47. A group of 286 parents is to be divided into committees with 3
or more parents on each committee. If each committee must
have the same number of parents and every parent must be on a
committee what is the maximum number of committees
possible?

48. * The graph above shows the frequency distribution of a list of


randomly generated integers between 0 and 6. What is the mean
of the list of numbers?

LEVEL 3: HEART OF ALGEBRA


49. Michael needs a printing job completed. Photoperfect Print Shop
charges a fixed fee of $3 for the print job and 5 cents per page.
Bargain Printing charges a fixed fee of $2 for the print job and 7
cents per page. If 𝑝 represents the number of pages being
printed, what are all values of 𝑝 for which Photoperfect Print
Shop’s total charge is less than Bargain Printing’s total charge.
(A) 𝑝 < 20
(B) 20 ≤ 𝑝 ≤ 35
(C) 35 ≤ 𝑝 ≤ 50
(D) 𝑝 > 50

164
[Link]

𝑎𝑏
50. Which of the following is equivalent to ( 𝑐 )(𝑐𝑏 − 𝑎) ?
𝑏
(A) 𝑎𝑏 2 − 𝑐
𝑎2 𝑏
(B) 𝑎𝑏 2 − 𝑐
𝑎𝑏 𝑎2 𝑏
(C) 𝑐
− 𝑐
𝑎𝑏
(D) 𝑐
− 𝑎2 𝑏𝑐

51. A block is sliding down a ramp that drops 3 centimeters in


elevation for every 5 centimeters along the length of the ramp.
The top of the ramp, where the back edge of the block is initially
placed, is at 60 centimeters elevation, and the block is sliding at
10 centimeters per second down the ramp. What is the elevation
of the ramp, in centimeters, at the point where the back of the
block passes 𝑡 seconds after being released?
3
(A) 60 − 5 𝑡
(B) 60 − 3𝑡
(C) 60 − 6𝑡
(D) 60 − 9𝑡
52. It costs (𝑠 + 𝑡) dollars for a box of brand A cat food, and
(𝑞 − 𝑟) dollars for a box of brand B cat food. The difference
between the cost of 15 boxes of brand A cat food and 7 boxes of
brand B cat food is 𝑘 dollars. Which of the following equations
represents a relationship between 𝑠, 𝑡, 𝑞, 𝑟, and k ?
(A) 105(𝑠 + 𝑡)(𝑞 − 𝑟) = 𝑘
(B) |7(𝑞 − 𝑟) + 15(𝑠 − 𝑡)| = 𝑘
(C) |15(𝑠 + 𝑡) + 7(𝑞 − 𝑟)| = 𝑘
(D) |15(𝑠 + 𝑡) − 7(𝑞 − 𝑟)| = 𝑘
53. Tickets for a concert cost $4.00 for children and $6.00 for
adults. 850 concert tickets were sold for a total cost of $3820.
How many children’s tickets were sold?
1
54. * If (√𝑥)𝑦 = 5, what is the value of 𝑥 2𝑦 ?

165
[Link]

LEVEL 3: GEOMETRY AND TRIG


55. Which of the following is an equation of the line in the 𝑥𝑦-plane
that passes through the point (0, −7) and is perpendicular to the
line 𝑦 = −6𝑥 + 2?
(A) 𝑦 = −6𝑥 + 7
(B) 𝑦 = −6𝑥 + 14
1
(C) 𝑦 = − 6 𝑥 + 6
1
(D) 𝑦 = 6 𝑥 − 7

56. Line 𝑘 contains the point (4,0) and has slope 5. Which of the
following points is on line 𝑘?
(A) (1, 5)
(B) (3, 5)
(C) (5, 5)
(D) (7, 5)

57. The 2 triangles in the figure above share a common side. What
is cos (𝑥 + 𝑦) ?
(Note: cos (𝑥 + 𝑦) = cos 𝑥 cos 𝑦 – sin 𝑥 sin 𝑦 for all 𝑥 and 𝑦.)
4
(A) 5
3+4√24
(B) 5
4√24 − 3
(C) 25
3−4√24
(D) 25

58. The sum of the areas of two squares is 85. If the sides of both
squares have integer lengths, what is the least possible value for
the length of a side of the smaller square?

166
[Link]

59. In the xy -plane above, the area of triangle 𝑂𝐴𝐵 is 32. What is
the value of 𝑏?

60. Triangle 𝐴𝐵𝐶 has the same area as a rectangle with sides of
̅̅̅̅ is 10, what is the length
lengths 5 and 7. If the length of 𝐴𝐶
̅̅̅̅
of 𝐵𝐷?

LEVEL 3: PASSPORT TO ADVANCED MATH


7(𝑎 + 𝑏) = 2(𝑏 − 𝑎)
61. If (𝑎, 𝑏) is a solution to the equation above and 𝑎 ≠ 0, what is
𝑏
the ratio 𝑎 ?
9
(A) − 5
8
(B) − 5
(C) 8
(D) 11

167
[Link]

ℎ(𝑥) = −2(𝑥 2 − 5𝑥 + 3) + 7(𝑐 − 𝑥)


62. In the polynomial ℎ(𝑥) defined above, 𝑐 is a constant. If ℎ(𝑥) is
divisible by 𝑥, what is the value of 𝑐 ?
6
(A) − 7
(B) 0
6
(C)
7
(D) 6
63. For all 𝑥, (𝑥 2 − 3𝑥 + 1)(𝑥 + 2) = ?
(A) 𝑥 3 − 𝑥 2 − 5𝑥 + 2
(B) 𝑥 3 − 𝑥 2 − 5𝑥 − 2
(C) 𝑥 3 − 𝑥 2 + 5𝑥 + 2
(D) 𝑥 3 + 𝑥 2 − 5𝑥 + 2
64. For all numbers 𝑎 and 𝑏, let 𝑎∀𝑏 = 𝑎2 − 3𝑎𝑏 2. What is the
value of |5∀(2∀1)|?
9(2𝑘)3
65. If 𝑘 ≠ 0, what is the value of (3𝑘)3
?

66. For any real numbers 𝑟 and 𝑠 such that 𝑟 ≠ 𝑠, let 𝑟 ⋯ 𝑠 be


𝑟−𝑠
defined by 𝑟 ⋯ 𝑠 = 𝑟+𝑠. If 𝑟 − 𝑠 = 63 and 𝑟 ⋯ 𝑠 = 9, what is
the value of 𝑟 ?

LEVEL 3: PROBLEM SOLVING AND DATA


67. Which scatterplot shows a nonlinear positive association? (Note:
A positive association between two variables is one in which
higher values of one variable correspond to higher values of the
other variable.)

168
[Link]

68. The average (arithmetic mean) age of the people in a certain


group was 35 years before one of the members left the group
and was replaced by someone who is 12 years older than the
person who left. If the average age of the group is now 37 years,
how many people are in the group?
69. A mixture is made by combining a red liquid and a blue liquid
so that the ratio of the red liquid to the blue liquid is 17 to 3 by
weight. How many liters of the blue liquid are needed to make a
420 liter mixture?

70. A bus driver drove at an average speed of 45 miles per hour for
3 hours while the bus consumed fuel at a rate of 15 miles per
gallon. How many gallons of fuel did the bus use for the entire
3-hour trip?

TEST GRADES OF STUDENTS IN MATH CLASS


Test Grade 75 82 87 93 100
Number of students
5 7 10 3 1
with that grade

71. The test grades of the 26 students in a math class are shown in
the chart above. What is the median test grade for the class?
72. A certain exam lasts a total of 4 hours. Each part of the exam
requires the same amount of time and 10 minute breaks are
included between consecutive parts. If there is a total of 4 breaks
during the 4 hours, what is the required time, in minutes, for
each part of the test?

LEVEL 4: HEART OF ALGEBRA


5
=6
√𝑥 − 7
73. For 𝑥 > 7, which of the following equations is equivalent to the
equation above?
(A) 25 = 36(𝑥 − 7)
(B) 25 = 6(𝑥 − 7)
(C) 25 = 6(𝑥 − √7)
(D) 5 = 36(𝑥 − 7)

169
[Link]

𝑥 = 36𝑧
𝑦 = 36𝑧 2 + 5
74. If 𝑧 > 0 in the equations above, what is 𝑦 in terms of 𝑥?
1
(A) 𝑦 = 36 𝑥 2 + 4
1 2
(B) 𝑦 = 𝑥 +5
36
1 2
(C) 𝑦 = 36
𝑥 + 36
1 2
(D) 𝑦 = 6
𝑥 +4

75. A small hotel has 15 rooms which are all occupied. If each room
is occupied by either one or two guests and there are 27 guests
in total, how many rooms are occupied by two guests?
𝑥 + 2𝑦 = 3
2𝑥 − 𝑦 = 8.5
76. If (𝑥, 𝑦) is a solution to the above system of equations, what is
the value of x – y ?
77. If 𝑥𝑦 = 22, 𝑦𝑧 = 10, 𝑥𝑧 = 55, and 𝑥 > 0, then 𝑥𝑦𝑧 =
78. Last month Joe the painter painted many rooms. He used 3 coats
of paint on one third of the rooms he painted. On two fifths of
the remaining rooms he used 2 coats of paint, and he only used 1
coat of paint on the remaining 24 rooms. What was the total
number of coats of paint Joe painted last month?

LEVEL 4: GEOMETRY AND TRIG


79. A cylinder has volume 𝑉, height ℎ, and base diameter 𝑑. Which
of the following represents 𝑑 in terms of 𝑉 and ℎ ?

(A) 𝑑 = √𝑉𝜋ℎ
𝑉
(B) 𝑑 = √
𝜋ℎ

2𝑉
(C) 𝑑 = √
𝜋ℎ

4𝑉
(D) 𝑑 = √𝜋ℎ

170
[Link]

80. When each side of a given square is lengthened by 3 inches, the


area is increased by 45 square inches. What is the length, in
inches, of a side of the original square?
(A) 3
(B) 4
(C) 5
(D) 6

81. Triangle 𝑃𝑄𝑅 above is equilateral with 𝑃𝑄 = 44. The ratio of


𝑆𝑇 to 𝑇𝑈 is 8: 3. What is the length of ̅̅̅̅
𝑆𝑄 ?
(A) 6
(B) 16
(C) 16√3
(D) 32
82. A 7 foot ladder is leaning against a wall such that the angle
relative to the level ground is 70°. Which of the following
expressions involving cosine gives the distance, in feet, from the
base of the ladder to the wall?
7
(A) cos 70°

cos 70°
(B) 7

1
(C) 7 cos 70°

(D) 7 cos 70°

171
[Link]

83. The length of each side of an equilateral triangle will be doubled


to create a second triangle. The area of the second triangle will
be how many times the area of the original triangle?
84. In triangle 𝐷𝐸𝐹, 𝐷𝐸 = 𝐷𝐹 = 10 and 𝐸𝐹 = 16. What is the area
of the triangle?

LEVEL 4: PASSPORT TO ADVANCED MATH

85. What is the maximum value of the function graphed on the


𝑥𝑦-plane above, for −4 ≤ 𝑥 ≤ 4 ?
(A) −4
(B) 3
(C) 4
(D) ∞
86. Let ‖𝑥‖ be defined as the sum of the integers from 1 to 𝑥,
inclusive. Which of the following equals ‖21‖ − ‖20‖?
(A) ‖1‖
(B) ‖5‖
(C) ‖6‖
(D) ‖21‖
87. In the standard (𝑥, 𝑦) coordinate plane, what are the coordinates
of the center of the circle whose equation is
𝑥 2 − 8𝑥 + 𝑦 2 + 10𝑦 + 15 = 0 ?
(A) (4,5)
(B) (4, −5)
(C) (−4,5)
(D) (−5, −4)

172
[Link]

𝑥𝑎𝑥𝑏
88. If (𝑥 𝑐 )𝑑 = 𝑥 2 for all 𝑥 ≠ 0, which of the following must be true?

(A) 𝑎 + 𝑏 − 𝑐𝑑 = 2
𝑎+𝑏
(B) 𝑐𝑑
=2
(C) 𝑎𝑏 − 𝑐𝑑 = 2
(D) 𝑎𝑏 − 𝑐 𝑑 = 2
89. In the equation 𝑥 2 − 𝑏𝑥 + 𝑐 = 0, 𝑏 and 𝑐 are integers. The
solutions of this equation are 2 and 3. What is 𝑐 − 𝑏?
3𝑥 2 + 19𝑥 = 14
90. If 𝑎 and 𝑏 are distinct solutions of the equation above, what is
the value of −3𝑎𝑏 ?

LEVEL 4: PROBLEM SOLVING AND DATA

91. The scatterplot above shows the numbers of incidences of


melanoma, per 100,000 people from 1940 to 1970. Based on the
data shown in the figure, which of the following values is
closest to the range of the number of incidences of melanoma
between 1945 and 1950?
(A) 50,000
(B) 100,000
(C) 170,000
(D) 360,000

173
[Link]

92. * The graph above shows the amount of carbohydrate supplied


by four different foods, I, II, III, and IV, as a percentage of their
total weights. The cost of 20 ounces of foods I, II, III, and IV,
are $4.00, $3.50, $3.00, and $2.75, respectively. Which of the
four foods supplies the most carbohydrate per dollar?
(A) I
(B) II
(C) III
(D) IV
93. In Dr. Steve’s math class, 12 students play the piano and 17
students play the guitar. If a total of 19 students play only one of
these two instruments, how many students play both
instruments?

174
[Link]

94. * 743 children from the United States, aged 6 through 11, were
tested to see if they were overweight. The data are shown in the
table below.

Not
Overweight Total
overweight
Ages 6-8 31 286 317
Ages 9-11 163 263 426
Total 194 549 743

Based on the data, how many times more likely is it for a 6, 7, or


8 year old to NOT be overweight than it is for a 9, 10, or 11 year
old to NOT be overweight? (Round the answer to the nearest
tenth.)
95. * Jessica has two cats named Mittens and Fluffy. Last year
Mittens weighed 12 pounds, and Fluffy weighed 19 pounds.
Fluffy was placed on a diet, and his weight decreased by 20%.
Mittens weight has increased by 20%. By what percentage did
Mitten’s and Fluffy’s combined weight decrease, to the nearest
tenth of a percent?
SURVEY RESULTS

96. The circle graph above shows the distribution of responses to a


survey in which a group of people were asked how often they
donate to charity. What fraction of those surveyed reported that
they donate at least yearly?

175
[Link]

LEVEL 5: HEART OF ALGEBRA


97. If 𝑥 2 + 𝑦 2 = 𝑘 2 , and 𝑥𝑦 = 8 − 4𝑘, what is (𝑥 + 𝑦)2 in terms
of 𝑘?
(A) 𝑘−4
(B) (𝑘 − 4)2
(C) 𝑘 2 − 4𝑘 + 8
(D) (𝑘 − 2)2 + 4
𝑦 ≤ 2𝑥 + 2
𝑦 ≥ −3𝑥 − 3

98. A system of inequalities and a graph are shown above. Which


section or sections of the graph could represent all of the
solutions to the system?
(A) Section I
(B) Section IV
(C) Sections II and III
(D) Sections I, II, and IV
99. For how many integers 𝑛 is (7𝑛 − 26)(5𝑛 + 11) a negative
number?
(A) None
(B) Two
(C) Four
(D) Six

176
[Link]

100. If 𝑎 and 𝑏 are positive integers, which of the following is


equivalent to (5𝑎)3𝑏 − (5𝑎)2𝑏 ?
(A) 5𝑏 (𝑎3 − 𝑎2 )
(B) (5𝑎)2𝑏 [(5𝑎)3𝑏 − 1]
(C) (5𝑎)2𝑏 (25𝑎 − 1)
(D) (5𝑎)2𝑏 [(5𝑎)𝑏 − 1]
101. If |−3𝑎 + 15| = 6 and |−2𝑏 + 12| = 4, what is the greatest
possible value of 𝑎𝑏?
102. * A cheetah ran 12 miles at an average rate of 50 miles per hour
and then traveled the next 12 miles at an average rate of 43 mile
per hour. What was the average speed, in miles per hour, of the
cheetah for the 24 miles?

LEVEL 5: GEOMETRY AND TRIG

103. In the figure above, arc 𝐴𝐵𝐶 is one quarter of a circle with
center 𝐸 and radius 8√2. If the length plus the width of
rectangle 𝐵𝐷𝐸𝐹 is 16, then the area of the shaded region is
(A) 32𝜋 − 32
(B) 32𝜋 − 16
(C) 32𝜋 − 8
(D) 32𝜋 + 16

177
[Link]

104. A ladder rests against the side of a wall and reaches a point that
is ℎ meters above the ground. The angle formed by the ladder
and the ground is 𝜃°. A point on the ladder is 𝑘 meters from the
wall. What is the vertical distance, in meters, from this point on
the ladder to the ground?
(A) (ℎ − 𝑘) tan 𝜃°
(B) (ℎ − 𝑘) cos 𝜃°
(C) ℎ − 𝑘 sin 𝜃°
(D) ℎ − 𝑘 tan 𝜃°
2
105. * In the 𝑥𝑦 plane, line 𝑘 has equation 𝑦 = 9 𝑥 + 5, and line n
1
has equation 𝑦 = 4 𝑥 + 𝑏. If the lines intersect at the point with
2
coordinates (𝑎, 3) , what is the value of 𝑏 ?

106. If the length of a rectangle is increased by 40%, and the width of


the same rectangle is decreased by 40%, then the area of the
rectangle is decreased by 𝑥%. What is the value of 𝑥?

107. * In the figure above, 𝐴𝐵 is the arc of a circle with center 𝑂. If


the length of arc 𝐴𝐵 is 7𝜋, what is the area of region 𝑂𝐴𝐵 to the
nearest integer?

108. A sphere with volume 36𝜋 cubic inches is inscribed in a cube so


that the sphere touches the cube at 6 points. What is the surface
area, in square inches, of the cube?

178
[Link]

LEVEL 5: PASSPORT TO ADVANCED MATH


109. * Jonathon wants to place a rectangular fence around the border
to his backyard. The width of the fence will be 350 inches more
than 5 times the length of the fence. What will be the perimeter
of Jonathon’s fence if the area of the fence is 64,680 square
inches?
(A) 854 inches
(B) 1274 inches
(C) 1708 inches
(D) 2548 inches
110. If 𝑎, ℎ, and 𝑘 are nonzero constants, and the parabola with
equation 𝑦 = 𝑎(𝑥 − ℎ)2 + 𝑘, in the 𝑥𝑦-plane, passes through
the points (ℎ, 5) and (0,2), which of the following must be true?
3
(A) ℎ2 = −
𝑎
3
(B) 𝑎2 = − ℎ
3
(C) 𝑎=−

(D) 𝑎ℎ = −3
111. If 𝑥 − 3 is a factor of 𝑎𝑥 2 − 𝑎2 𝑥 + 12, where 𝑎 is a positive
constant, what is the value of 𝑎 ?

112. The xy-plane above shows the two points of intersection of the
graphs of a linear function and a quadratic function. The
leftmost point of intersection has coordinates (𝑎, 𝑏) and the
rightmost point of intersection has coordinates (𝑐, 𝑑). If the
vertex of the graph of the quadratic function is at (2, −27), what
is the value of 𝑏 − 𝑑 ?

179
[Link]

113. *An arrow is launched upward with an initial speed of 70 m/s


(meters per second). The equation 𝑣 2 = 𝑣02 − 2𝑔ℎ describes
the motion of the arrow, where 𝑣0 is the initial speed of the
arrow, 𝑣 is the speed of the arrow as it is moving up in the
air, ℎ is the height of the arrow above the ground, 𝑡 is the
time elapsed since the arrow was projected upward, and 𝑔 is
the acceleration due to gravity (approximately 9.8 m/s 2).
What is the maximum height from the ground the arrow will
rise to the nearest meter?
114. Let 𝑓 be a linear function such that 𝑓(5) = −2 and 𝑓(11) = 28.
𝑓(9)−𝑓(7)
What is the value of ?
2

LEVEL 5: PROBLEM SOLVING AND DATA


Questions 115 - 116 refer to the following information.
743 children from the United States, aged 6 through 11, were
tested to see if they were overweight. The data are shown in the
table below.

Not
Overweight Total
overweight
Ages 6-8 31 286 317
Ages 9-11 163 263 426
Total 194 549 743

115. In 2014 the total population of children between 6 and 11 years


old, inclusive, in the United Sates was about 74.3 million. If the
test results are used to estimate information about children
across the country, which of the following is the best estimate of
the total number of children between 9 and 11 years old in the
United States who were overweight in 2014?
(A) 3,100,000
(B) 16,300,000
(C) 19,400,000
(D) 42,600,000

180
[Link]

116. * According to the table, which of the following statements is


most likely to be true about children between 6 and 11 years old,
inclusive, in the United Sates?
(A) The probability that a 6-8 year old is overweight is
greater than the probability that an overweight child aged
6-11 is less than 9 years old.
(B) The probability that a 6-11 year old is overweight is
greater than the probability that a 9-11 year old is not
overweight.
(C) The probability that an overweight 6-11 year old is at
least 9 years old is greater than the probability that a 6-11
year old is not overweight.
(D) The probability that a 6-8 year old is overweight is
greater than the probability that a 9-11 year old is not
overweight.
1 1 1
, , , 𝑥, 𝑥 2 , 𝑥 3
𝑥3 𝑥2 𝑥
117. If −1 < 𝑥 < 0, what is the median of the six numbers in the list
above?
1
(A)
𝑥
(B) 𝑥 2
𝑥 2 (𝑥+1)
(C) 2
𝑥(𝑥 2 +1)
(D)
2

118. A group of students takes a test and the average score is 72. One
more student takes the test and receives a score of 88 increasing
the average score of the group to 76. How many students were
in the initial group?
119. Jason ran a race of 1600 meters in two laps of equal distance.
His average speeds for the first and second laps were 11 meters
per second and 7 meters per second, respectively. What was his
average speed for the entire race, in meters per second?

120. A scatterplot includes the points (1,0), (2,0), (3,0), and


(0, −6). The data is fitted with a cubic curve whose equation
has the form 𝑦 = 𝑥 3 + 𝑏𝑥 2 + 𝑐𝑥 + 𝑑. If the curve passes
through all four of the given points, find the value of 𝑏 + 𝑐.

181
[Link]

ANSWERS TO
SUPPLEMENTAL PROBLEMS

LEVEL 1: HEART OF ALGEBRA


1. D
2. A
3. D
4. D
5. 63
6. 11

LEVEL 1: GEOMETRY AND TRIG


7. D
8. C
9. D
10. D
11. D
12. 34

LEVEL 1: PASSPORT TO ADVANCED MATH


13. D
14. B
15. B
16. B
17. 60
18. 4

LEVEL 1: PROBLEM SOLVING AND DATA


19. A
20. C
21. A
22. 7040
23. 125
24. 82

182
[Link]

LEVEL 2: HEART OF ALGEBRA


25. D
26. D
27. C
28. B
29. D
30. 10

LEVEL 2: GEOMETRY AND TRIG


31. B
32. C
33. B
34. D
35. B
36. C

LEVEL 2: PASSPORT TO ADVANCED MATH


37. D
38. A
39. 6
40. 1
41. 6
42. 3

LEVEL 2: PROBLEM SOLVING AND DATA


43. C
44. D
45. C
46. 89
47. 26
48. 3.2

183
[Link]

LEVEL 3: HEART OF ALGEBRA


49. D
50. B
51. C
52. D
53. 640
54. .001 or .002

LEVEL 3: GEOMETRY AND TRIG


55. D
56. C
57. D
58. 2
59. 8
60. 7

LEVEL 3: PASSPORT TO ADVANCED MATH


61. A
62. C
63. A
64. 35
65. 8/3, 2.66, or 2.67
66. 35

LEVEL 3: PROBLEM SOLVING AND DATA


67. C
68. 6
69. 63
70. 9
71. 87
72. 40

184
[Link]

LEVEL 4: HEART OF ALGEBRA


73. A
74. B
75. 12
76. 9/2 or 4.5
77. 110
78. 116

LEVEL 4: GEOMETRY AND TRIG


79. D
80. D
81. B
82. D
83. 4
84. 48

LEVEL 4: PASSPORT TO ADVANCED MATH


85. B
86. C
87. B
88. A
89. 1
90. 14

LEVEL 4: PROBLEM SOLVING AND DATA


91. C
92. C
93. 5
94. 1.5
95. 4.5
96. 3/4 or .75

185
[Link]

LEVEL 5: HEART OF ALGEBRA


97. B
98. B
99. D
100. D
101. 56
102. 46.2

LEVEL 5: GEOMETRY AND TRIG


103. A
104. D
105. 5.54
106. 16
107. 252
108. 216

LEVEL 5: PASSPORT TO ADVANCED MATH


109. C
110. A
111. 4
112. 36
113. 250
114. 5

LEVEL 5: PROBLEM SOLVING AND DATA


115. B
116. C
117. D
118. 3
119. 8.55 or 8.56
120. 5

186
[Link]

About the Author


Steve Warner, a New York native, earned his Ph.D. at Rutgers University in Pure
Mathematics in May, 2001. While a graduate student, Dr. Warner won the TA
Teaching Excellence Award.

After Rutgers, Dr. Warner joined the Penn


State Mathematics Department as an
Assistant Professor. In September, 2002, Dr.
Warner returned to New York to accept an
Assistant Professor position at Hofstra
University. By September 2007, Dr. Warner
had received tenure and was promoted to
Associate Professor. He has taught
undergraduate and graduate courses in
Precalculus, Calculus, Linear Algebra,
Differential Equations, Mathematical Logic,
Set Theory and Abstract Algebra.

Over that time, Dr. Warner participated in a five year NSF grant, “The MSTP
Project,” to study and improve mathematics and science curriculum in poorly
performing junior high schools. He also published several articles in scholarly
journals, specifically on Mathematical Logic.

Dr. Warner has over 15 years of experience in general math tutoring and over
10 years of experience in AP Calculus tutoring. He has tutored students both
individually and in group settings.

In February, 2010 Dr. Warner released his first SAT prep book “The 32 Most
Effective SAT Math Strategies.” The second edition of this book was released in
January, 2011. In February, 2012 Dr. Warner released his second SAT prep book
“320 SAT Math Problems arranged by Topic and Difficulty Level.” Between
September 2012 and January 2013 Dr. Warner released his three book series
“28 SAT Math Lessons to Improve Your Score in One Month.” In June, 2013 Dr.
Warner released the “SAT Prep Official Study Guide Math Companion.” In
November, 2013 Dr. Warner released the “ACT Prep Red Book – 320 Math
Problems With Solutions.” Between May 2014 and July 2014 Dr. Warner
released “320 SAT Math Subject Test Problems arranged by Topic and Difficulty
Level.” for the Level 1 and Level 2 tests. In November, 2014 Dr. Warner released
“320 AP Calculus AB Problems arranged by Topic and Difficulty Level,” and in
January, 2015 Dr. Warner released “320 AP Calculus BC Problems arranged by
Topic and Difficulty Level,”

187
[Link]

BOOKS BY DR. STEVE WARNER

188
[Link]

CONNECT WITH DR. STEVE WARNER

189

You might also like